examen ginecologia o_20196

94
Universidad La Salle. Facultad Mexicana de Medicina. Curso de Extensión Universitaria para la Preparación del Examen Nacional para Aspirantes a Residencias Médicas. Examen de Ginecología y Obstetrícia. 1.- Se trata de paciente femenino de 29 años de edad con tumor anexial de 6 cm líquido, dolor abdominal, fiebre, leucorrea, con historia de cervicovaginitis de repetición y dispareunia crónica, última menstruación hace una semana. El diagnóstico más probable de ésta paciente es: a) Cistoadenoma b) Enfermedad pélvica inflamatoria c) Embarazo ectópico d) Quiste de ovario La EIP puede cursar con los siguientes síntomas: Dolor abdominal bajo (incluyendo dolor anexial, dispareunia). Es el síntoma más frecuente (95%) Aumento del flujo vaginal, flujo de características anormales (74%) Sangrado anormal (intermestrual, poscoital) (45%) Síntomas urinarios (35%) Vómitos (14%) Es posible la ausencia de síntomas Y en ella podemos encontrar estos signos: Dolor a la movilización del cuello, dolor anexial en la exploración vaginal bimanual (99%) En el examen con espéculo observamos cervicitis y descarga endocervical purulenta (74%) Fiebre (> 38º C) (menos del 47%). Masa pélvica: sugiere abceso tuboovárico (ATO) Peritonitis CRITERIOS CLINICOS PARA EL DIAGNOSTICO DE SALPINGITIS a. Dolor abdominal con o sin rebote. b. Sensibilidad a la movilización del cérvix. c. Sensibilidad anexial. Los tres criterios anteriores son necesarios para establecer el diagnóstico, con uno o más de los siguientes:

Upload: carlos-mario-de-la-fuente-villarreal

Post on 22-Jan-2018

98 views

Category:

Health & Medicine


0 download

TRANSCRIPT

Universidad La Salle. Facultad Mexicana de Medicina.

Curso de Extensión Universitaria para la Preparación del Examen Nacional para Aspirantes a Residencias Médicas.

Examen de Ginecología y Obstetrícia. 1.- Se trata de paciente femenino de 29 años de edad con tumor anexial de 6 cm líquido, dolor abdominal, fiebre, leucorrea, con historia de cervicovaginitis de repetición y dispareunia crónica, última menstruación hace una semana. El diagnóstico más probable de ésta paciente es: a) Cistoadenoma b) Enfermedad pélvica inflamatoria c) Embarazo ectópico d) Quiste de ovario

La EIP puede cursar con los siguientes síntomas:

Dolor abdominal bajo (incluyendo dolor anexial, dispareunia). Es el síntoma más frecuente (95%)

Aumento del flujo vaginal, flujo de características anormales (74%) Sangrado anormal (intermestrual, poscoital) (45%) Síntomas urinarios (35%) Vómitos (14%) Es posible la ausencia de síntomas

Y en ella podemos encontrar estos signos:

Dolor a la movilización del cuello, dolor anexial en la exploración vaginal bimanual (99%)

En el examen con espéculo observamos cervicitis y descarga endocervical purulenta (74%)

Fiebre (> 38º C) (menos del 47%). Masa pélvica: sugiere abceso tuboovárico (ATO) Peritonitis

CRITERIOS CLINICOS PARA EL DIAGNOSTICO DE SALPINGITIS

a. Dolor abdominal con o sin rebote. b. Sensibilidad a la movilización del cérvix. c. Sensibilidad anexial.

Los tres criterios anteriores son necesarios para establecer el diagnóstico, con uno o más de los siguientes:

a. Extendido de Gram de endocérvix positivo, para diplococos gram negativos intracelulares

b. Temperatura mayor de 38°C c. Leucocitosis (mayor de 10.000 por c.c.) d. Material purulento (positivo para leucocitos) en la cavidad peritoneal obtenido

por culdocentesis o laparoscopia.

Establecido el diagnóstico clínico de EPI, se debe hacer la definición del estado clínico y anatómico de la patología pélvica:

a) No complicada (limitada a trompas u ovarios)

1) Sin peritonitis pélvica

2) Con peritonitis pélvica

b) Complicada (masa inflamatoria o absceso que compromete trompa (s) u ovario (s)

1) Sin peritonitis pélvica

2) Con peritonitis pélvica

o Beigi RH, Wiesenfeld HC. Pelvic inflammatory disease: new diagnostic criteria and treatment. Obstet Gynecol Clin Norh Am. 2003; 30 (4): 777 – 93

o Center for Disease Control. Guidelines for treatment of sexually transmited diseases. MMWR Recomm Rep. 2002 May 10;51(RR-6):1-78

o Center for Disease Control. Guidelines for prevention and management (MMWR. 40: 1 - 25 1991) Pelvic inflammatory disease: guidelines for prevention and management. MMWR Recomm Rep. 1991 Apr 26;40(RR-5):1-25.

o Hager WD, Eschenbach DA, Spence MR, Sweet RL. Criteria for diagnosis and grading of salpingitis. Obstet Gynecol. 1983 Jan;61(1):113-4.

o Prodigy Guidance. Pelvic inflammatory disease. [Internet]. UK : NHS, Department of Health; 2003. [Acceso 18 de Junio de 2005]. Disponible en:

o Ross J. Pelvic inflammatory disease. Clin Evid. 2004 Dec;(12):2259-65. o Royal College of Obstetricians and Gynecologists. Pelvic Inflammatory Disease.

Guideline nº 32. [Internet]. RCOG; Mayo 2003. [Acceso 18 de Junio de 2005].

2.-Femenino de 25 años con antecedentes de G/2, P/1, C/1 acude al servicio de consulta externa, refiere que presenta una secreción transvaginal bastante líquida, de baja viscosidad, maloliente de color amarillo y gris, espumoso. El tratamiento de elección para esta entidad es: a) Metronidazol 500 mg. VO c/12 por 7 a 10 días b) Ampicilina 1g VO c/ 6 hrs. c) Clotrimazol + Metronidazol 500 mg VO c/12 hrs. por 7 a días d) Clindamicina 300 mg VO cada 12 hrs. x 5 días.

Referencias bibliográficas: 1. Secretaría de Salud. Norma Oficial Mexicana NOM -039-SSA2-2002, Para la prevención y control de las infecciones de transmisión sexual. D.O.F. 19 de Septiembre 2003. 2. Kettler H, White K, Hawkes S. Mapping the landscape for sexually transmitted infections: key findings and recommendations. Geneva, TDR (TDR/STI/ IDE/04.1). 3. CDC. Trends in Reportable Sexually Transmitted Diseases in the United States. CDC, National Report. 2004 4. Distribución de los casos nuevos de enfermedades por mes Estados Unidos Mexicanos 2004. Sistema Único de Información para la Vigilancia Epidemiológica/Dirección General de Epidemiología/SSA 5. Aral S O. Sexual risk behavior and infection: epidemiological considerations. Sex. Transm. Inf. 2004;80:8-12 3.- Femenino de 25 años, con embarazo de término, sin antecedentes de control prenatal. G 3. C-1. Se ingresa al servicio de obstetricia por presentar actividad uterina regular y dolorosa. Ef.: Deambulante, tranquila, adecuada coloración de tegumentos, abdomen con fondo uterino a 32 cm. con producto único vivo en situación transversa dorso inferior FCF 144, al tacto vaginal cérvix dilatado a 3 cm. y membranas íntegras. Se realiza cesárea con retención de placenta e invasión a vejiga. Estudio de gabinete de elección previo al evento obstétrico es:

a) prueba sin estrés b) perfil biofísico c) simple de abdomen d) ultrasonido doppler

PLACENTA:

Se divide en: Placenta acreta. Placenta increta. Placenta percreta.

ACRETA: Las vellosidades se adhieren al miometrio.

INCRETA: Penetran más de la mitad del espesor del miometrio.

PERCRETA: Atraviesa todo el espesor del miometrio, llegando a la serosa, incluso atravesándola y adhiriéndose a órganos vecinos.

Factores:

Endometriósis previa. Tumores submucosos. (Miomas) Cicatríz uterina previa. (Cesárea, miomectomía) Implantación baja. (Placenta previa) Malformaciones placentarias. (Placenta extracorial) Legrado enérgico previo. Extracción manual previa de una placenta.

Diagnóstico Pre-parto

Ultrasonografía Döppler.

Resonancia Magnética. Diagnóstico transparto-:

Placenta retenida por más de 20 minutos. Imposibilidad para encontrar un plano de separación placentaria cuando se intenta su

extracción manual. Hemorragia incontrolable después de la pseudoextracción. El diagnóstico histopatológico corrobora el diagnóstico clínico. Escenario menos deseable.

Tratamiento:

Histerectomía Obstétrica. Constituye una cirugía no planeada y secundaria al hallazgo del acretismo

placentario con sangrado incohercible.

Cesárea-Histerectomía. (Con diagnóstico previo) Cirugía planificada ante un correcto diagnóstico prenatal.

Recomendación ACOG:

Maduración pulmonar intrauterina. Inyectar al cordón umbilical 50 mg de metrotexate. Ligar el cordón en el nacimiento placentario y dejar la placenta in-situ. Embolización inmediata de arterias uterinas bilaterales, así como de ramas de

la división anterior de la arteria iliaca interna con alcohol polivinílico. Continuar con 5 dosis I.M. de 50 mg de metrotexate y cuantificar niveles de

βhCG. Programar Histerectomía Total Radical Abdominal y/o Cistectomía parcial y/o

resección pared anterior recto. Lee et al. Conservative Management of Placenta Percreta. Obstet Gynecol, 112(2):421-424

4.- Mujer que acude al servicio de ginecología por referir ciclos opso-menorreicos desde el inicio de su menarquia, ha incrementado 15 Kg. de lo que pesaba habitualmente, se aprecia una gran cantidad de acné, pero además refiere depilarse el área del bigote cada semana, y cree que esto le sensibiliza la piel para que aumente el acné. El diagnóstico más probable en esta paciente es: a) Sx. De Asherman b) Sx. Stein Leventhall c) Sx. Amenorrea Galactorrea d) Sx. Karman Síndrome de Ovario Poliquístico (SOP) es uno de los más comunes trastornos endocrinos que afectan a las mujeres alrededor del 5% al 10% de las mujeres en edad reproductiva (12-45 años) y se piensa que es una de las principales causas de la infertilidad femenina. Las características principales son la obesidad, anovulación (dando lugar a la menstruación irregular) o amenorrea, acné, y las cantidades excesivas o los efectos de androgénicos (masculinizantes) hormonas. Los síntomas y la severidad del síndrome varían mucho entre las mujeres. Si bien las causas son desconocidas, resistencia a la insulina, la diabetes y la obesidad están fuertemente correlacionadas con el SOP.

Bulun SE, Adashi EY. The physiology and pathology of the female reporductive axis. In: Kronenberg HM, Melmed S, Polonsky KS, Larsen PR, eds. Williams Textbook of Endocrinology. 11th ed. Philadelphia, Pa: Saunders Elsevier; 2008:chap 16.

5.- Se trata de mujer de 43 años, G- 4, P-3, A-1, con diagnóstico de anemia ferropénica, de 9.5 g/dl, refiere ciclos menstruales de 31,32 x 8,9 días de duración, acompañados de coágulos, los cuales aparecieron después del nacimiento de su segundo hijo hace 13 años. E.F.: Buen estado general, TA 130/80, genitales con evidencia de sangrado activo, al tacto vaginal se detecta útero de consistencia firme voluminosa, irregular, aproximadamente de 12 cm. anexos libres. El diagnóstico más probable es:

a) Adenomiosis uterina. b) Cáncer cervicouterino. c) Miomatosis uterina. d) Hiperpalsia adenomatosa de endometrio.

MIOMATOSIS UTERINA

Definición:

Tumor benigno que se origina en el miometrio, por lo que su componente histológico predominante es el tejido muscular y, en menor medida, el conectivo y fibroso. El único tratamiento efectivo es el quirúrgico; sin embargo, sólo requieren ser tratados aquellos que producen síntomas.

Evaluación y Diagnóstico:

Historia:

1. El síntoma más frecuente suele ser la hemorragia uterina. 2. Los síntomas principales están relacionados con el crecimiento del tumor. 3. La paciente puede notar una masa en hipogastrio o abdomen inferior.

4. La masa se puede asociar a dolor pélvico, o manifestaciones por compresión de órganos o estructuras vecinas.

5. Puede haber alteración de la fertilidad.

Examen Físico:

1. Se debe realizar con la vejiga y el recto vacuo. 2. El hallazgo primordial es el aumento de volumen y consistencia del útero, el cual puede

ser simétrico (nódulos submucosos) o irregular (nódulos intramurales o subserosos).

Exámenes Auxiliares:

1. Papanicolau cérvico–vaginal: Indicado siempre; permite descartar neoplasia epitelial cervical o cáncer infiltrante de cérvix.

2. Ultrasonido transabdominal y transvaginal: Indicado siempre; permite evaluar la localización, tamaño y número aproximado de miomas.

3. Hemoglobina, hematocrito: Indicado cuando hay historia de sangrado; orienta en la severidad del sangrado y anemia.

4. Hemograma y VSG: Indicado cuando hay historia de fiebre; si es anormal sugiere infección o necrosis del mioma (puede ser apropiado descartar infección de otro órgano o sistema).

5. Grupo sanguíneo y factor Rh: Si hay anemia severa o en el preoperatorio. 6. Perfil de coagulación (tiempo de protrombina y de tromboplastina parcial, recuento de

plaquetas): Si hay historia de sangrado exagerado. 7. Gonadotrofina coriónica (sub–unidad � sérica): Permite descartar posible embarazo en

casos de ciclos irregulares, retraso menstrual o útero de consistencia blanda.

Diagnóstico Diferencial:

1. Embarazo. 2. Tumor de ovario. 3. Enfermedad inflamatoria pélvica, complejo inflamatorio anexial pélvico. 4. Endometriosis. 5. Adenomiosis. 6. Tumor extragenital: colon, retroperitoneo.

Referencias Bibliográficas:

1. Hillard PA. Benign Diseases of the Female Reproductive Tract: Symptoms and Signs. En: Berek JS, Adashi EY, Hillard PA, eds. Novak's Gynecology. Baltimore: Williams and Wilkins, 1996:331-97.

1. Hutchins FL, Greenber MD. Miomas Uterinos: Diagnóstico e Indicaciones de Tratamiento. Clinicas de Ginecología y Obstetricia. Temas Actuales. 1995;5:609-14.

2. Davis KM, Sclass WD. Tratamiento Médico para Miomatosis Uterina. Clinicas de Ginecología y Obstetricia. Temas Actuales. 1995;5:671-81.

1. Selwyn P, Oskowitz MB. Leiomyomata Uteri. En: Friedman EA, ed. Gynecological Decision Making. St. Louis: Mosby, 1983:148-9.

1. Diaz Huamán V. Tumores Benignos del Aparato Reproductor Femenino. En: Ludmir A, Cervantes R, Castellano C, eds. Ginecología y Obstetricia, Prevención - Diagnóstico - Tratamiento. Lima: Concytec, 1996:907-25.

6.- Femenino de 34 años que cursa con 38.5 semanas de gestación que ingresa al servicio con trabajo de parto. En el transcurso de trabajo de parto durante la dilatación presenta dolor intenso y brusco. A la exploración usted observa metrorragia escasa y aumento del tono uterino a la palpación abdominal que resulta muy doloroso. El diagnóstico más probable es: a) Placenta previa. b) Rotura de vasos previos. c) Crioamnionitis hemorrágica. d) Desprendimiento de placenta. Fisiopatología de la hemorragia La hemorragia es el signo fundamental que domina el cuadro clínico de la placenta previa. Para explicar su mecanismo existen distintas teorías: Mecanismo de Jacquemier: Se produce crecimiento armónico de la placenta y del útero hasta la semana 26, 28. Después el segmento inferior crece más deprisa y favorece el despegamiento lo que origina la hemorragia en el embarazo. Mecanismo de Schroeder: Las contracciones uterinas en el parto traccionan del segmento inferior hacia arriba y empujan al feto hacia abajo despegando la placenta. Mecanismo de Pinard: Explica las hemorragias gestacionales y del parto. El estiramiento de las membranas de la zona de menor radio (orificio interno cervical) como consecuencia de las contracciones tira de la placenta y la desprenden. Mecanismo de Bartholomew: Explica la hemorragia en los casos de placenta previa central. La zona placentaria que reviste el orificio interno es un área isquémica ya que no recibe vasos deciduales. A este nivel disminuye la presión sanguínea, por lo que la sangre tiende a dirigirse hacia esta zona y escapa por la cara materna. En el alumbramiento también puede haber una hemorragia importante producida por un doble mecanismo: -desprendimiento parcial antes de la expulsión en los casos de placenta oclusiva. -atonía uterina en la zona de inserción después de expulsada la placenta y vascularización anómala. Manifestaciones clínicas de la placenta previa en el embarazo - Síntomas: Principalmente la hemorragia. Toda hemorragia vaginal acontecida en el tercer trimestre debe hacer pensar en una placenta previa. Las hemorragias suelen ser espontáneas, no acompañadas de dolor, de sangre roja y se presentan de forma intermitente, con intervalos variables entre las mismas. Progresivamente se van haciendo más frecuentes y más graves. La primera hemorragia suele aparecer en forma inesperada generalmente nocturna, cesando en menos de media hora. Las hemorragias ulteriores son más graves y más precoces. - Signos: La consecuencia fundamental es la anemia materna que depende de la cuantía de la hemorragia (la sangre es de origen materna ya que procede de espacios intervellosos). - Exploración: Exploración general para valorar la existencia de signos de anemia. Exploración obstétrica: valorar el tamaño del útero (adecuado para la edad gestacional), es blando e indoloro. A menudo la estética fetal está alterada (transverso, oblicuo, nalgas). No debe efectuarse tacto vaginal cuando haya existido hemorragia en embarazo avanzado por el riesgo de infección y de despegamiento y aumentar así la hemorragia. Auscultación fetal normal. Exploración ecográfica: es una técnica fundamental en el diagnóstico de la placenta previa. Permite determinar la localización placentaria y la variedad de la placenta previa. En general, la placenta puede identificarse a partir de la 9ª semana. No obstante, a lo largo de la gestación

por crecimiento uterino se produce un cambio en sus relaciones con el útero ("emigración placentaria", imagen de desplazamiento). Así el diagnóstico de certeza de límites placentarios solo puede establecerse hacia la semana 34. Siempre será necesario hacer una adecuada identificación del orificio cervical interno (más fácil con sonda transvaginal). Añadiendo al estudio Doppler color se observa la vascularización y las zonas que sangran. 7.- Se trata de femenino de 24 años de edad gesta 1, tuvo un parto vaginal espontáneo con un producto con peso de 4,350 g. después de 5 minutos de tracción suave del cordón umbilical se expulsó la placenta, que parece estar intacta. Se inició el masaje del fondo uterino y se pidió a la enfermera que administrara 20 unidades de oxitocina en 100 ml de solución Ringer lactato. Después de una inspección cuidadosa del canal del parto se observa una laceración de segundo grado y una laceración de 2 cm en la pared vaginal izquierda que se intentó reparar. En la exploración física se encuentra un fondo uterino blando y atónico. Los signos vitales son: temperatura 37.1°C, TA 164/92, FC 130x’, FR 18 X’. El tratamiento más adecuado en éste caso es: (a) Oxitocina 10 unidades directas en goteo intravenoso (b) Metilergonovina 0.2 mg IM (c) Prostaglandina F 0.25 mg IM (d) Legrado Morgan M, Siddighi S. Ginecología y obstetricia, National Medical Series. 5° edición. Mc Graw Hill. Pp. 28. La atonía uterina es la causa más común de hemorragia puerperal. El masaje energético y la oxitocina diluida no han sido útiles para interrumpir la hemorragia y por tanto el siguiente paso es agregar un fármaco uterotónico. La metilergonovina está contraindicada porque la paciente se encuentra hipertensa a pesar de la hemorragia intensa, el siguiente fármaco es la prostaglandina. La administración de oxitocina no diluida, 10 UI por vía IV podría causar hipotensión grave. La exploración manual podría ser apropiada si se sospecha laceración como causa de hemorragia. El legrado es apropiado para la hemorragia puerperal tardía, cuando se sospecha retención de los productos de la concepción. 8.- Se trata de paciente femenino de 20 años de edad, que acude a consulta refiriendo presentar 6 meses sin regla. Se manifiesta preocupada por su sobrepeso, ha estado a dieta y ha perdido 6 kg en 8 meses. Actualmente pesa 46 Kg. con talla de 1,65 mts. Signos vitales dentro de sus parámetros normales. Se realiza prueba de embarazo con resultado negativa. La causa más probable de su amenorrea es: a) Disgenesia gonadal. b) Hipogonadismo hipogonadotropo. c) Síndrome de ovario poliquístico. d) Adenoma hipofisario. Sigue siendo práctica y eficaz la diferenciación de los hipogonadismos en hipergonadotrópicos (con fallo de función en la gónada) e hipogonadotrópicos (con fallo en hipotalámo y/o hipófisis). La deficiencia puede ser total o parcial y afectar a una o ambas funciones gonadales (producción de esteroides y germinal). Las causas son muy variadas. En la tabla 1 se han recopilado las principales de origen hipogonadotrópico, aunque algunas de ellas raramente se observan durante la infancia (p. ej., hiperprolactinemia) y en la tabla 2 las de origen hipergonadotrópico. Dentro del apartado de “idiopático” poco a poco se van dilucidando las

causas y, así, con la ayuda de la biología molecular se encuentran mutaciones del gen del receptor de GnRH, de LH o alteraciones del gen DAX 1. Por otra parte debe tenerse en cuenta que en diferentes síndromes puede aparecer un hipogonadismo hipogonadotrópico (tabla 3). MANIFESTACIONES CLÍNICAS Y ANALÍTICAS La ausencia de datos clínicos de hipogonadismo durante la infancia es la regla. Tan sólo la presencia de micropene o criptorquidia en algún varón puede hacer sospechar de la existencia de un hipogonadismo, pero ni es patognomónica (se da también en el déficit de GH y en otras situaciones no hormonalmente deficitarias), ni su ausencia excluye la deficiencia gonadal. Por lo tanto, y si se descarta la sospecha del hipogonadismo por otras características asociadas (p. ej., diversos síndromes, alteraciones evidentes: tumoraciones o radioterapia hipotálamo-hipofisaria), sólo el retraso de aparición de la pubertad será la primera indicación de una deficiencia de estas características. Sin embargo, aquí se produce el principal problema para alcanzar un diagnóstico temprano al resultar muy difícil distinguirlo de una variante normal del desarrollo: el retraso constitucional del crecimiento y adolescencia. Respecto al hipogonadismo hipogonadotrópico los estudios hormonales son escasamente discriminatorios en la época previa a la pubertad por lo que el momento en que se realiza el diagnóstico, y por lo tanto el inicio del tratammiento, suele ser tardío. La ausencia de caracteres sexuales secundarios, junto con un tamaño infantil de las gónadas (por palpación testicular o ecografía ovárica), es suficiente para valorar una situación prepuberal. En todo caso, la confirmación, según el sexo, de bajos niveles de testosterona circulante, o de estrógenos (directamente o por citología vaginal/vesical), permiten una sencilla, fácil y rápida comprobación. No debe olvidarse que en relación a la causa del hipogonadismo (p. ej., una tumoración hipofisacualquier edad, es objetivar una anosmia (síndrome de Kallmann), más aún si es parcial. En los casos de hipogonadismo hipergonadotrópico puede encontrarse en ciertos periodos a lo largo de la infancia niveles elevados de gonadotrofinas basales y que tras estímulo con GnRH muestran una respuesta más elevada de lo normal. Sin embargo, lo más habitual es encontrarse con niveles propios de la infancia al realizar las determinaciones basales. Recientemente se ha demostrado por medio de una técnica muy sensible que no era cierta la pretendida ausencia de estradiol en las niñas, por lo que si se consigue generalizar su empleo será un arma valiosa de diagnóstico durante la infancia. TABLA 1. Causas de hipogonadismo hipogonadotrópico Panhipopituitarismo Idiopático Tumoración hipofisaria o hipotalámica Misceláneo Enfermedad granulomatosa Traumatismo Vasculitis Infarto Hemocromatosis Deficiencia aislada de gonadotrofinas Síndrome de Kallman y variantes Hipogonadismo hipotalámico idiopático Deficiencia aislada de LH Deficiencia aislada de FSH Hipoplasia adrenal congénita ligada al cromosoma X Defectos de desarrollo de la línea media Síndromes con diversas malformaciones (tabla 2) Prader-Willi Laurence-Moon-Bardet-Biedl Alteraciones sistémicas Deficiencias nutritivas Enfermedades crónicas Ponderales: desnutrición severa y obesidad mórbida Yatrogenia Farmacológica Radioterápica Quirúrgica Hiperprolactinemia

Ejercicio físico de alta competición Retraso constitucional de la pubertad (?) BIBLIOGRAFÍA SELECCIONADA:

Henkin RI, Bartter FC Studies on olfactory thresholds in normal man and in patients with adrenal cortical insufficiency: The role of adrenal cortical steroids and of serum sodium concentration. J Clin Invest 1966; 45: 1631-1639. 2. Schwanzel-Fukuda M, Abraham S, Crossin KL, Edelman GM, Pfaff DW. Immunocytochemical demonstration of neural cell adhesion molecule (NCAM) along the migration route of luteinizing hormone-releasing hormone (LHRH) neurons in mice J Comp Neurol 1992; 321: 1-18. 3. Schwanzel-Fukuda M, Bick D, Pfaff DW: Luteinizing hormone-releasing hormone (LHRH)-expressing cells do not migrate normally in an inherited hypogonadal (Kallmann) syndrome. Mol Brain Res 1989; 6: 311-326.

9.- Se trata de paciente femenino de 18 años de edad la cual acude a consulta por presentar amenorrea acompañada de profundas alteraciones del olfato. De los siguientes el diagnóstico más probable es: a) Síndrome de amenorrea-galactorrea. b) Amenorrea de causa uterina. c) Síndrome de ovario poliquístico. d) Amenorrea por alteración hipotalámica.

• AMENORREA HIPOTALAMICA. (Hipogonadotrópica) – Psicógena. (Stress emocional) – Anorexia nerviosa. (Deficiencia nutricional) – Ejercicio excesivo. (Carrera de fondo, natación, gimnasia, ballet) – Fármacos. (Fenotiazina, reserpina, bloqueadores ganglionares,

anticonceptivos) – Pseudociésis. – Síndrome de Kallman (Deficiencia selectiva de gonadotropinas y anosmia).

El ejemplo clásico de la alteración hipotalámica que lleva a desórdenes del ciclo menstrual es el Síndrome de Kallman. Lectura Recomendada: Etiopatogenia de la amenorrea hipotalámica funcional Interacción de las respuestas hormonales del Sistema Nervioso Central y Neuropéptidos Periféricos Revista Argentina de Endocrinología y Metabolismo Copyright © 2008 por la Sociedad Argentina de Endocrinología y Metabolismo Vol 45 • No. 2

10.- Una mujer de 27 años G/4, P/0 con 6 SDG acude a su primera visita prenatal. Su historia obstétrica pasada es importante porque tiene tres pérdidas de producto en el segundo trimestre. Refiere que en las tres ocasiones al presentarse al hospital presentaba dilación cervical completa. No recuerda haber tenido contracciones dolorosas. Niega antecedentes médicos y quirúrgicos. El examen físico es normal incluyendo un examen pélvico que muestra un cervix largo y cerrado. Después de una larga discusión con la paciente ella pide que se le practique un cerclaje durante este embarazo. El momento más apropiado para realizarlo es: a) Inmediatamente b) 13 a 17 semanas c) 24 a 28 semanas d) 32 a 36 semanas El cerclaje cervical tiene sus indicaciones en la profilaxis y tratamiento de la incompetencia cervical. La incompetencia o insuficiencia cervical representa un 10% de las causas de parto pretérmino y está asociada a una importante morbimortalidad neonatal. Las modificaciones cervicales en el segundo trimestre de gestación son causa de parto prematuro y pueden deberse a: 1) Incompetencia cervical. 2) Pérdida de tejido conectivo tras una cirugía cervical (conización). 3) Defectos congénitos como la hipoplasia cervical tras exposición a dietilestilbestrol. 4) Infección intrauterina. Hasta un 51.5% de las pacientes con clínica compatible con incompetencia cervical enmascaran un cuadro de infección intraamniótica subclínica. Diferenciamos tres tipos de cerclaje: 1. El cerclaje se considera profiláctico o electivo (o primario) cuando se realiza de forma electiva por historia previa de incompetencia cervical antes de evidenciar cambios en el cerviz y generalmente suele realizarse entre las 13 y 17 semanas de gestación. 2. El cerclaje terapéutico secundario que se realiza tras la detección, en el seguimiento obstétrico, de modificaciones en el cérvix antes de las 26 semanas de gestación. Se realiza en pacientes con un riesgo potencial de parto pretérmino. 3. El cerclaje terapéutico terciario, en caliente, de rescate o “emergent cerclage” que se realiza en pacientes que presentan la membrana amniótica visible a través del orificio cervical externo o en vagina. GUIA CLÍNICA: INDICACIONES DEL CERCLAJE Unitat de Prematuritat. Servei de Medicina Maternofetal. Institut Clínic de Ginecologia, Obstetrícia i Neonatologia, Hospital Clínic de Barcelona Responsables del protocolo: T.Cobo, M. López, M. Palacio Creación: 24/01/07 Modificaciones: 05/09/07 Última actualización: 17/01/10

11.- Se trata de femenino de 67 años refiere intenso prurito vulvar y sensación quemante, al examen el introito vaginal se encuentra estenótico. De los siguientes es el tratamiento más apropiado es: a) 5-fluoracilo b) Testosterona tópica c) Corticoesteroides fluorados d) Estrógeno tópico

Vulvovaginitis atrófica

El hipoestrogenismo conduce a atrofia de la vagina y el vestíbulo vulvar, que los hace fácilmente irritables y susceptibles a infecciones secundarias. Las pacientes refieren sensación de quemadura, prurito, disuria, hipersensibilidad y dispareunia. Puede encontrarse al examen físico atrofia, fisuras superficiales, y un flujo vaginal acuoso1. Hay disminución del tamaño del introito2, pérdida de la rugosidad y la vagina toma una apariencia lisa y brillante.

Los hallazgos histológicos revelan un epitelio vaginal delgado, disminución de los lechos capilares, y la citología muestra, a medida que la atrofia progresa, aumento de las células basales y disminución o ausencia de las células superficiales2.

Se aconseja evitar el uso de jabones y demás irritantes de la piel. Se pueden utilizar lubricantes simultáneamente con los estrógenos o como terapia única, si hay alguna contraindicación a las hormonas.

El tratamiento con estrógenos por vía sistémica o transvaginal mejora y restaura los signos y síntomas, y una a dos semanas después de iniciar el tratamiento los cambios de atrofia empiezan a mejorar rápidamente, se reduce el pH y se induce maduración vaginal y de la mucosa uretral, reduciendo la frecuencia de las infecciones urinarias3. La dosis y vía de administración debe ser debidamente individualizada4. Contraindicaciones al tratamiento con estrógenos, incluyen: la presencia de tumores estrógenosensibles, falla hepática terminal y antecedentes de tromboembolización relacionada con ellos.

Menopausia y Piel. Parte II: Manifestaciones clínicas dermatológicas durante la menopausia MARÍA ISABEL BARONA C. Docente adjunto. Dermatóloga Universidad del Valle-Cali. 12.- Mujer de 26 años, es atendida en consulta en la clínica de displasias por papanicolau con lesión NIC I. Antecedentes: menarca 14 años, ritmo 30x5 eumenorreica, inicio de vida sexual a los 15 años, 2 parejas sexuales, método de planificación familiar oclusión tubaria bilateral, gestas 3 partos 3, cérvix con lesión acetoblanca con extensión lineal de 2 cm. Para confirmar el diagnóstico se debe realizar: a) Crioterapia de lesión. b) Captura de híbridos. c) Repetir colposcopía d) Biopsia de la lesión.

9.5.2 Las pacientes a quienes se les realizó citología cervical, cuyo resultado es LEIBG (infección por VPH, displasia leve o NIC 1); LEIAG (displasia moderada y grave o NIC 2 y 3) o cáncer deben enviarse a una clínica de colposcopía, para realizar estudio colposcópico. 9.5.3 Si el resultado de la citología es LEIBG, la colposcopía es satisfactoria y sin evidencia de LEIBG, se realizará control citológico en un año (Apéndice Normativo A) 9.5.4 Si la citología es de LEIBG, la colposcopía es satisfactoria y existe evidencia de lesión, se debe tomar una biopsia dirigida.

9.5.4.1 Si la biopsia dirigida es negativa, se realizará nueva colposcopía para verificar el diagnóstico y en caso necesario, tomar nueva biopsia dirigida y revalorar. 9.5.4.2 Si la biopsia dirigida es reportada como LEIBG se podrá dar tratamiento conservador: criocirugía, electrocirugía o laserterapia (sólo si cumple con las condiciones referidas en el Apéndice 1) o se podrá mantener a la paciente en vigilancia en la clínica de colposcopía, con colposcopía y estudio citológico cada seis meses, durante 24 meses. Jueves 31 de mayo de 2007 DIARIO OFICIAL (Primera Sección) 9.5.4.3 Si la biopsia dirigida es reportada como LEIAG (Lesión Intraepitelial Escamosa de Alto Grado) se realizará tratamiento conservador (electrocirugía o laserterapia). En las mujeres posmenopáusicas, dependiendo de las condiciones anatómicas del cérvix, se realizará tratamiento conservador en la clínica de colposcopía o tratamiento quirúrgico (histerectomía extrafascial) en el servicio que corresponda. 9.5.4.4 Si la biopsia dirigida reporta cáncer microinvasor o invasor, la paciente se transferirá a un Servicio o Centro Oncológico para su tratamiento correspondiente. 9.5.4.5 Si la citología reporta LEIBG y la colposcopía es no satisfactoria, se tomará cepillado endocervical (Apéndice Normativo A) 9.6 En caso de colposcopía no satisfactoria, negativa a LEIBG y con cepillado endocervical negativo, se continuará su control en la clínica de colposcopía en seis meses, con colposcopía y citología. 9.6.1.1 Si el cepillado endocervical reporta LEIBG se tratará a la paciente como LEIAG, con métodos conservadores escisionales. Jueves 31 de mayo de 2007 DIARIO OFICIAL (Primera Sección) Modificación a la Norma Oficial Mexicana NOM-014-SSA2-1994, Para la prevención, detección, diagnóstico, tratamiento, control y vigilancia epidemiológica del cáncer cérvico uterino. Al margen un sello con el Escudo Nacional, que dice: Estados Unidos Mexicanos.- Secretaría de Salud. MODIFICACION A LA NORMA OFICIAL MEXICANA NOM-014-SSA2-1994, PARA LA PREVENCION, DETECCION, DIAGNOSTICO, TRATAMIENTO, CONTROL Y VIGILANCIA EPIDEMIOLOGICA DEL CANCER CERVICO UTERINO.

7.3 El resultado del estudio citológico es descriptivo y debe ser informado de la siguiente manera:

a.- Negativo a cáncer.

b.- Negativo con proceso inflamatorio.

c.- Displasia leve (NIC 1).

d.- Displasia moderada (NIC 2).

e.- Displasia grave (NIC 3).

f.- Cáncer del cuello del útero in situ (NIC 3).

g.- Cáncer microinvasor e invasor.

h.- Adenocarcinoma.

i.- Maligno no especificado

13.- Se trata de paciente de 33 años que cursa con 39 SDG; a la exploración física reflejos patelares hiperactivos, inquieta, se reportan cifras de TA 145/95, se realiza laboratorio que reporta proteinuria 2+,. El diagnóstico más probable es: a) Glomerulonefritis aguda

b) Hipertensión esencial c) feocromocitoma d) Preeclampsia

La hipertensión es la complicación médica más común del embarazo , aunque para algunos autores es la segunda complicación médica del embarazo sólo después de la anemia; es más frecuente en jóvenes durante el primer embarazo y en nulíparas de mayor edad, hipertensas previas y diabéticas.

En México, también es la complicación más frecuente del embarazo, la incidencia es de 47.3 por cada 1 000 nacimientos y es además, la primera causa de ingreso de pacientes embarazadas a las unidades de terapia intensiva (debido a hemorragia masiva, para recibir soporte hemodinámico), según la secretaría de salud (2001) la mortalidad por complicaciones del embarazo ocupa el 15º lugar en la mortalidad hospitalaria en general. Además, la tasa de preeclampsia se ha incrementado 40% en el periodo entre 1990 y 1999 y constituye hasta 40% de los partos prematuros iatrogénicos.

.

Preeclampsia

La preeclampsia es un síndrome clínico caracterizado por hipertensión con disfunción orgánica múltiple, proteinuria, edemas.

Es definida como un incremento de al menos 140/90 mmHg después de la semana 20 de gestación, un incremento en la presión sanguínea diastólica de al menos 15 mmHg respecto a un nivel previo a la semana 20 combinado con proteinuria (> 300 mg en 24 horas). Las mediciones de la presión arterial citadas deben ser medidas al menos 2 ocasiones con por lo menos 6 horas de separación. La proteinuria puede ser una toma simple de orina al azar que indique al menos 30 mg/dL 3 ó ++ en dos muestras de orina1 según el tipo de prueba. El criterio del incremento de 30 mmHg en la presión sistólica y/o 15 mmHg en la presión diastólica respecto a valores previos a la semana 20 de gestación ha sido eliminado por ser poco específico15

Myers JE, Baker PN. Hupertensive diseases and eclampsia. Curr Opin Obstet Gynecol 2002; 14: 119-125 2. Tierney, McPhee, Papadakis. Diagnóstico clínico y tratamiento 2003. 38ª ed, México, Manual Moderno, 2003: 770-773 3. Wilson MI, Goodwin TM, Pan VI, Ingles SA. Molecular epidemiology of preeclampsia. Obstet and Gynecol Survey 2003; 58(1):39-66 4. Burrow GM. Complicaciones médicas durante el embarazo. 4ª ed, México, McGraw-Hill panamericana: 1996: 1-25 5. Guyton AC, Hall JE. Embarazo y lactancia en: Tratado de fisiología médica, 10ª ed, México, McGraw-Hill Interamericana 2001: 1135-45 6. Vaticon D. Fisiología de la fecundación, embarazo, parto y lactancia, en: Tresguerres JAF. Fisiología Humana. México, Interamericana McGraw-Hill, 1992: 1086-1109

7. Pridjian G, Puschett JB. Preeclampisa. Part 1: Clinical and Pathophysiologic Considerations. Obstet and Gynecol Survey 2002; 57 (9): 598-618 8. Pridjian G, Puschett JB. Preeclampisa. Part I1: Experimental and Genetic Considerations. Obstet and Gynecol Survey 2002; 57 (9): 619-40 9. IMSS. Embarazo de alto riesgo. Guía diagnóstica terapéutica. Rev Med IMSS 1998; 36(1):45-60 14.- Femenino de 36 años, con presencia de hiper y polimenorreas o pérdidas de sangre contínuas sin conservación del ciclo, se sospecha de miomatosis uterina, éste tipo alteración se presenta con mayor frecuencia en los miomas de localización:

a) Submucoso b) Intramural d) Intraligamentaria e) cervical

Descripción

Los miomas son tumores monoclonales benignos de las células del músculo liso del miometrio. Están compuestos por grandes cantidades de matriz extracelular que contiene colágeno, fibronectina y proteoglicanos. El colágeno tipo I y tipo II es abundante, pero las fibrillas de colágeno son anormales y están desorganizadas, de modo similar a lo que se observa en la formación de queloides.

Los miomas submucosos son los menos frecuentes, constituyendo únicamente el 5% de la totalidad de los miomas, a menudo producen un aumento del sangrado menstrual en forma de hiper y polimenorreas, e incluso hemorragias importantes que exigen tratamiento de urgencia. Las metrorragias también son habituales en este tipo de miomas. Según Novak, el peligro de degeneración sarcomatosa es mucho mayor en los miomas submucosos, e igualmente es causa de dismenorreas más intensas y frecuentes.

BIBLIOGRAFÍA 1. De la Fuente U. Tratado de Obstetricia y Ginecología. Mc Graw-Hill. (Madrid). 1998. Volumen II. 2. DI SAIA S. Tratado de Obstetricia y Ginecología de Danforth. Sexta. Mc Graw-Hill. (Nueva York). 1990. Sexta Edición. 3. González-Merlo J. Tratado de Obstetricia y Ginecología. Salvat Editores S.A. (Barcelona). 1990. Quinta Edición. 4. Aller J., Pages G. Obstetricia Moderna. Tercera Edición. Mc Graw-Hill. (Caracas). 1999. 5. Formación Médica Continuada en Atención Primaria. Marzo 1995. Vo. 5, Nº (3).

15.- Se trata de mujer de 26 años, es atendida en consulta por secreción láctea bilateral hace 4 meses y menstruaciones cada 21 días, en escasa cantidad desde hace 6 meses. Antecedentes: hace 6 meses padece gastritis tratada con cimetidina durante 2 meses y posteriormente ha tomado el medicamento de forma irregular. a.g.o.: menarca 12 años, ritmo 30/7, núbil. e.f.: talla 1.63, peso 54 kg. Al efectuar compresión en glándulas mamarias se produce salida de secreción láctea bilateral, resto normal. El estudio de elección en esta paciente para confirmar el diagnóstico es: a) perfil hormonal ginecológico. b) prolactina sérica. c) radiografía de silla turca. d) ultrasonido mamario. La hiperprolactinemia (hiperprl) es un trastorno frecuente, más en la mujer que en el hombre, y puede ser la manifestación de un adenoma hipofisario (prolactinoma). La medición de prolactina (prl) en la sangre es ahora un procedimiento de rutina en la detección de trastornos de la menstruación y galactorrea que se resuelven efectivamente con el uso de fármacos con actividad dopaminérgica (bromocriptina y cabergolina). La causa más frecuente de hiperprl es por el uso de fármacos que inhiben la acción de dopamina, pero se debe descartar inicialmente hipotiroidismo y un prolactinoma mediante resonancia magnética. Los prolactinomas son fácilmente controlados con dopaminérgicos y excepcionalmente se requiere cirugía. En caso de infertilidad por exceso de prl existe una respuesta favorable con bromocriptina o cabergolina, aunque en la actualidad se prefiere la última por su efectividad y menos efectos indeseables.

Hiperprolactinemia. Guía terapéutica y diagnóstica Arturo Zárate* * Unidad de investigación de endocrinología, diabetes y metabolismo, Centro médico nacional IMSS y hospital ángeles México. 16.- Se trata de femenino de 31 años nuligesta pero con actividad sexual regular, sin método de planificación familiar, con ciclos regulares, sin leucorrea, refiere dispareunia profunda, sangrado intermestrual y dismenorrea secundaria ocacionalmente presenta urgencia urinaria, el método de elección para confirmar el diagnóstico clínico de ésta paciente es:

a) Cultivo de exudado vaginal b) Ultrasonido c) Biopsia de endometrio d) Laparoscopia con biopsias dirigidas

Endometriosis

Examinación bimanual

Dolor en los ligamentos utero‐sacros

Nodularidad en el fondo de saco

Induración del septo rectovaginal

Útero en retroversión fijo

Masa anexial

Endometriosis

Laparoscopiavisualización directa

estadificación

toma de muestras

Estadio I MinimoEstadio II LeveEstadio III ModeradoEstadio IV Severo

Referencias bibliográficas

1. Ruiz V. Endometriosis y fertilidad. Ed. Acosta y Warman, pp. 99

2. López, VH. Palomo E. Incidencias de endometriosis en una población infértil. XXI Congreso nacional de Ginecología y Obstetricia. Guatemala, 1993.

3. El-Eoley, et al. Danazol but not ginadotropin releasing hormone agonists suppresses autoantibodies in endomeriosis. Fertil Steril 1990; 54:725

4. Acosta AA. Buttram VC Jr. Besch PK, Malinak LR, Van Der Heyden J. A.proposed classfication of pelvic endometriosis. Obstet Gynecol 1973;42:19.

5. Buttran VC Jr. Evolution of the revised American Fertility classification of endometriosis. Fert. Steril 1985; 43: 347

6. López VH. Tratamiento médico-quirúrgico de la endometriosis. Simposio El rostro cambiante de la endometriosis panamá 3. 12. 1993.

7. Steinleitner A. Heterolous transplation of activated murine peritonel macrophages inhibitis gamete interaction in vivo; A paradigm fo endometriosis associted subfertility. Fertil Steril 1990; 54:725.

8. Damewood M. Effect of serum from patients with minimal to mild endometriosis on mouse embryo growth. Fertil Steril 1990; 54: 917

9. Proug S. Peritoneal fluid fracctions from patients with endometriosis do not promote two-cell mouse embryo growth. Fertil Steril 1990; 54: 927.

17.- Femenino de 22 años, que presenta una tumoración de 2 cm de diámetro en el cuadrante

ínfero-externo de la mama izquierda, indolora, de consistencia firme, superficie lisa, forma ovoidea, móvil y bien delimitada del parénquima vecino, sin antecedentes de derrame por el pezón, sin “piel de naranja” ni retracción del pezón. El diagnóstico más probable es:

a) Fibroadenoma. b) Carcinoma. c) Ectasia de los conductos mamarios. d) Quiste solitario.

FIBROADENOMA MAMARIO Tumor benigno más frecuente en las mujeres entre los 20 y 35 años. ETIOLOGIA Existen múltiples teorías siendo la más aceptada la hormonal, generalmente son únicos, solo el 20% son múltiples o bilaterales. De tamaño variable hasta de 10 cm. Ocupa el 13.6% de la patología mamaria benigna. CUADRO CLÍNICO Lesión nodular de consistencia dura, de larga evolución y no dolorosa. Normalmente llegan a los 3 cm. De diámetro. Durante la fase tardía del ciclo menstrual el tumor suele presentar un leve aumento de tamaño. Durante la menopausia presentan regresión hasta la calcificación (signo de palomitas de maíz). DIAGNOSTICO Es clínico, se presenta como un tumor bien delimitado, desplazable, no adherido a piel ni a planos profundos, liso o multilobulado en ocasiones. Se localiza frecuentemente en cuadrantes externos. EXAMENES DIAGNOSTICOS ULTRASONIDO MAMARIO .- Identifica un nódulo sólido, bien delimitado de bordes regulares . TRATAMIENTO.- Conservador con vigilancia estrecha dependiendo del tamaño y en caso de ser necesario exéresis del nódulo para estudio histopatológico hospitalgeneral.salud.gob.mx/ BIBLIOGRAFIA

1. Sánchez BC. Tratado de Enfermedades de la glándula mamaria. Ed. Manual Moderno. Cap. 13- 15. 2.- De Vita V. Cancer of the Breast. In Cancer: Principles and Practice of Oncology: Fifth Ed. Philadelphia: Lippincott-Raven, Chapter 36; pp: 1521-1616. 3.-Consenso Nacional Acerca del Tratamiento de Cáncer de Mama. En Tumores de mama: Diagnóstico y Tratamiento. 2ª Ed. McGraw-Hill Interamericana; pp: 119-126. 4.-Eberlein T. Current management of carcinoma of the breast. Ann Surgery 1994; 220: 121-136. 18.- A 26-year-old woman presents with malodorus gray-wellow discharge. You take a wet mount preparation and observe “Clue cells”. The agent of this infection and its treatment is: a) Gardnerella vaginalis / clindamycin b) Gardnerella vaginalis / Ketoconazole c) Trichomona vaginalis / metronidazole / treat the partner d) Candida albicans / nistatin Gardnerella vaginalis fue clasificada como una sola especie y fue establecida como agente causal de la vaginosis (antes conocida como vaginitis inespecífica). El cuadro clínico que presenta es caracterizado por una secreción blanca o blanco-grisácea que se percibe generalmente después de la relación sexual con olor fétido aminado (pescado). El diagnóstico certero es la base para evitar posibles complicaciones como la enfermedad inflamatoria pelviana y las complicaciones del embarazo. El tratamiento se basa principalmente en los fármacos como: metronidazol y clindamicina, debido a su efectividad y espectro, pero como todos se deben emplear con adecuada prudencia debido a su toxicidad. Además de que se deben corregir o modificar los factores predisponentes, ya que esta patología va en aumento convirtiéndose por su frecuencia en un problema de salud pública. Bibliografía: 1. Hernández F. Gardnerella vaginalis mobiluncus en la etiología de la vaginosis bacteriana. Rev Costarricense Ciencias Médicas 1998; 19: 57-61. 2. Hansen EA. Gardnerella. Rev Ginecol 2005; 25: 99. 3. Espinosa I, Lorenzo M, Bentancourt A, Riverón Y, Romero M. Caracterización bioquímica y antigénica de diferentes aislamientos de Gardnerella vaginalis. Rev Cubana Invest Biomed 2005; 24: 22-7. 4. Taylor F. Vaginal flora morphotypic profiles and assessment of bacterial vaginosis in women at risk for HIV infection.Infect Dis Obstet Gynecol 2004; 12: 121-6. 19.- Una mujer embarazada, puede afectar al feto y hacerlo contraer lesiones importantes durante el embarazo o al salir al exterior (atravesando el canal de parto), sí la gestante se encuentra afectada de la siguiente patología:

a) Tricomonas b) Gardenerella c) Herpes genital d) Gonococos

Herpes genital

La prevalencia de herpes simplex genital o tipo 2 (VHS-2) en mujeres embarazadas varía entre 7 y 33% en distintas series. La prevalencia ha experimentado un sostenido aumento durante los últimos años. Se estima que aproximadamente 1 a 3% de las mujeres adquiere cada año la infección. En el caso de parejas discordantes, la tasa de adquisición aumenta hasta 10 a 30% anual. La adquisición durante el embarazo es ~2%.

La transmisión al hijo ocurre principalmente cuando la mujer embarazada adquiere una infección primaria. La transmisión es de 30 a 50% cuando la infección primaria ocurre cerca del momento del parto. La mayor transmisión (85%) ocurre durante el parto. Sin embargo, también puede ocurrir transmisión intrauterina (5-8%) y post-natal (8-10%). Los factores que inciden en la transmisión son: infección primaria mucho mayor eficiencia que infección recurrente, parejas discordantes, títulos de anticuerpos maternos y procedimientos obstétricos invasores, (los que están absolutamente contraindicados).

Las manifestaciones en la mujer embarazada son principalmente bajo la forma de herpes genital localizado, muy raramente ocurre diseminación cutánea y visceral, situación de elevada mortalidad (50%). La infección en el niño, si ocurre en las primeras 20 semanas del embarazo, puede provocar aborto en 25%, malformaciones cerebrales, cicatrices, corioretinitis, RCIU. Si ocurre después de las 20 semanas, puede causar parto prematuro, RCIU, o herpes neonatal. Esta condición clínica tiene tres formas de presentación, las dos primeras de elevada mortalidad y secuelas: herpes diseminado y encefalitis herpética o infección localizada en piel, ojo y boca.

Pass R, Weber T, Whitley RJ. Herpesvirus infections in pregnancy. Recommendations from the International Herpes Management Forum. Management Strategies Workshop and 7th Annual Meeting.

Whitley R J. Varicella - Zoster virus. Mandell, Douglas and Bennett's Principles and Practice of Infectious Diseases. Mandell G, Bennett J, Dolin R, editors. Fifth edition, 2000 Churchill Livingstone, Philadelphia, pp: 1586-98.

Abarca K, Cohen B J, Vial P. Seroprevalence of parvovirus B19 in urban Chilean children and young adults, 1990 and 1996. Epidemiol Infect 2002; 128: 59-62. 20.- Femenino de 20 años, atendida en sala de urgencias ginecoobstétricas, Antecedente: cursa embarazo de 38 SDG. Exploración Física: en trabajo de parto. Repentinamente presenta sangrado profuso transvaginal y dolor abdominal. A la paciente se le coloca un monitor fetal externo. El tono uterino está incrementado y hay desaceleraciones variables y ocasionales de la FCF que llegan a 90.

El manejo apropiado en esta paciente es:

a) Tocolisis con un agonista beta adrenérgico b) Continuar el monitoreo de la madre y el producto c) Amniotomía d) Cesárea

El manejo de éstas pacientes es estricto y urgente: 1- Ingreso inmediato de la paciente en vigilancia a sala de partos. 2- Venopuntura. Si existen signos de shock, instaurar preferentemente dos vías, una central y otra periférica. 3- Solución de Ringer. 4- Monitoréo de signos vitales cada 15’. 5- Sonda vesical y monitoréo de diuresis. Diuresis de 50 ml/h asegura perfusión periférica correcta.

Evitar diuresis inferiores a 30 ml/h. 6- Monitoréo contínuo de LCF. 7- Laboratorio de urgencia: hemograma, ionograma, urea y creatinina, coagulograma. 8- Sangre como mínimo 4U para reserva (concentrado de glóbulos rojos o sangre entera). 9- Ecografía obstétrica para localizar y medir el hematoma y descartar otras causas de sangrado. Dependiendo del grado de desprendimiento realizar: 1- Desprendimiento leve: sin compromiso materno ni fetal y el embarazo es de pretérmino, conducta expectante. Uteroinhibición y maduración pulmonar fetal. 2- Desprendimientos moderados a severos: si hay compromiso materno o fetal u óbito fetal, terminar inmediatamente la gestación. 3- Independientemente de la vía del parto, es preceptiva la práctica de la amniotomía, para reducir la presión intraamniótica y disminuir el paso de tromboplastina hística a la circulación y la extravasación de sangre al miometrio. Las principales complicaciones son: 1- Hipotensión, shock hipovolémico. 2- Coagulación intravascular diseminada. 3- Necrosis cortical y necrosis tubular aguda renal.

Obstetricia. Scwarcz, Sala, Duverges. 7ª edic. Edit. El Ateneo. (Biblioteca Fac. Med. UNNE).

21.- Mujer de 27 años a quien después de estudios de laboratorio y exploración física se le diagnostica hirsutismo asociado a ovario poliquístico, el siguiente fármaco que usted elige para su tratamiento por ser el más adecuado es:

a) Clomifeno b) Estrógenos c) Corticoide d) Acetato de ciproterona

El acetato de ciproterona parece ser más efectivo que otros fármacos para el hirsutismo en mujeres causado por la producción ovárica excesiva de andrógenos

Una de las causas de hirsutismo (crecimiento piloso excesivo) en mujeres es la hiperproducción de andrógenos a partir del ovario. Varios fármacos pueden utilizarse para contrarrestar los efectos del andrógeno. El acetato de ciproterona es un fármaco antiandrogénico. Los efectos adversos informados con su uso fueron aumento de peso, depresión, fatiga, síntomas mamarios y disfunción sexual. La revisión de los ensayos encontró que el acetato de ciproterona parece ejercer un efecto en el hirsutismo similar a otros fármacos utilizados para el tratamiento del hirsutismo por exceso de andrógenos. No existen pruebas suficientes para comparar los efectos adversos de las opciones de tratamiento.

Van der Spuy ZM, le Roux PA. Acetato de ciproterona para el hirsutismo (Revisión Cochrane traducida). En: La Biblioteca Cochrane Plus, número 4, 2007. Oxford, Update Software Ltd. Disponible en: http://www.update-software.com. (Traducida de The Cochrane Library, 2007 Issue 4. Chichester, UK: John Wiley & Sons, Ltd.).

22.- Mujer de 36 años nulípara tras 3 años de relaciones sexuales sin contracepción, desde hace 1 año inició presentando dismenorrea, dispareunia y sangrado vaginal intermenstrual. El diagnóstico más probable de ésta paciente es: a) Insuficiencia luteínica. b) Enfermedad inflamatoria pélvica. c) Dismenorrea funcional. d) Endometriosis.

La endometriosis consiste en la aparición y crecimiento de tejido endometrial fuera del útero, sobre todo en la cavidad pélvica como en los ovarios, detrás del útero, en los ligamentos uterinos, en la vejiga urinaria o en el intestino. Es menos frecuente que la endometriosis aparezca fuera del abdomen como en los pulmones o en otras partes del cuerpo.

La endometriosis es una enfermedad relativamente frecuente, que puede afectar a cualquier mujer en edad fértil, desde la menarquia hasta la menopausia, aunque algunas veces, la endometriosis puede durar hasta después de la menopausia. La endometriosis altera la calidad de vida de las mujeres que la padecen, afectando a sus relaciones de pareja, familiares, laborales y de reproducción.

Síntomas

Los síntomas clásicos son la dismenorrea, dolor pélvico, dispareunia, sangrados

intermestruales y en muchos casos, esterilidad.

El dolor no tiene que ver con el tamaño y la severidad de la lesión; generalmente cuanto

menor es la lesión mayor dolor produce. El dolor se agrava con las menstruaciones y en los

casos en que la lesión ocupa el fondo de saco de Douglas, puede dar dispareunia. Existe un

aumento de la PGF2 alfa y PGE2 y un aumento de las contracciones uterinas que podría

deberse a un depósito de endometrio en la cavidad peritoneal.

La esterilidad debido a la endometriosis podría deberse a distintas causas de acuerdo a la

severidad de la patología. En los casos de endometriosis severa puede haber un factor

tuboperitoneal con adherencias y alteración en la anatomía de la pelvis que interfiera con el

transporte del esperma y el óvulo. En los casos de endometriosis leve hay varios mecanismos

propuestos que justifican su relación con la infertilidad: foliculogénesis alterada, fase lútea

inadecuada, fagocitosis espermática, mala calidad ovocitaria, embriotoxicidad y alteración a

nivel de la implantación.. La producción de prostaglandinas por el endometrio ectópico puede

afectar la motilidad tubaria, la foliculogénesis y la función del cuerpo lúteo. Puede haber un

aumento de la activación de los macrófagos peritoneales en la endometriosis que cause la

fagocitosis de los espermas o la secreción de citoquinas que pueden ser tóxicas para el

embrión. Según algunos investigadores habría un 60% de las mujeres con endometriosis que

presentan un síndrome de Folículo Luteinizado no roto (LUF) en el cual el folículo no se rompe

en la ovulación y el óvulo queda atrapado.

Referencias bibliográficas

1. Ruiz V. Endometriosis y fertilidad. Ed. Acosta y Warman, pp. 99

2. Lópes,VH. Palomo E. Incidencias de endometriosis en una población infértil. XXI Congreso nacional de Ginecología y Obtetricia. Guatemala, 1993.

3. El-Eoley, et al. Danazol but not ginadotropin releasing hormone agonists suppresses autoantibodies in endomeriosis. Fertil Steril 1990; 54:725

4. Acosta AA. Buttram VC Jr. Besch PK, Malinak LR, Van Der Heyden J. A.proposed classfication of pelvic endometriosis. Obstet Gynecol 1973;42:19.

5. Buttran VC Jr. Evolution of the revised American Fertility classification of endometriosis. Fert. Steril 1985; 43: 347

6. López VH. Tratamiento médico-quirúrgico de la endometriosis. Simposio El rostro cambiante de la endometriosis panamá 3. 12. 1993.

7. Steinleitner A. Heterolous transplation of activated murine peritonel macrophages inhibitis gamete interaction in vivo; A paradigm fo endometriosis associted subfertility. Fertil Steril 1990; 54:725.

8. Damewood M. Effect of serum from patients with minimal to mild endometriosis on mouse embryo growth. Fertil Steril 1990; 54: 917

9. Proug S. Peritoneal fluid fracctions from patients with endometriosis do not promote two-cell mouse embryo growth. Fertil Steril 1990; 54: 927.

23.-Se trata de femenino de 37 años de edad, a quien se realiza diagnóstico de placenta percreta, el tratamiento e elección en esta patología es:

a) Histerectomía. b) Ergonovina a dosis altas. c) Hemostasia con puntos transfictivos. d) Taponamiento uterino.

La placenta anormalmente adherida es poco común y tiene importancia clínica por su morbimortalidad, a consecuencia de hemorragia, perforación, invasión y lesión de las vías urinarias. Esta adherencia anormal está asociada con la implantación placentaria sobre cicatrices de cesárea previa, incisiones uterinas o legrados. La placenta percreta consiste en la penetración del tejido placetario a través de toda la pared uterina, traspasando la serosa de la misma. La identificación de esta anormalidad antes del parto es posible mediante métodos de imagen (escala de grises por ultrasonido, ecografía Doppler color pulsado o resonancia magnética nuclear). El tratamiento conservador se acompaña de elevada morbilidad en muchos casos, por lo que el tratamiento quirúrgico se convierte en el definitivo. La

literatura sugiere un aumento previsto en la incidencia de esta condición con base en el incremento del número de cesáreas, por lo que la histerectomía postcesárea será una decisión que enfrentarán los especialistas con mayor frecuencia.

Perucca E, Domínguez C, Yahng Ch, García R. Placenta previa percreta con invasión vesical. Rev Chil Obstet Ginecol 1997; 62(3): 206-10.

2. Abbas F, Talati J, Wasti S et al. Placenta percreta with bladder invasion as a cause of life threatening hemorrhage. J Urol 2000; 164: 1270-4.

3. Perucca E, Cazenave H, Barra A, Ochoa N, Villagrán G, Espinoza R, Estay R, Bustamante R, Siebert A. Placenta previa percreta con invasión vesical. Rev Chil Obstet Ginecol 2002; 67(5): 364-7.

4. Price F, Resnik E, Heller K, Christopherson W. Placenta previa percreta involving de urinary bladder. A report of two cases and review of the literature. Obstet Gynecol 1991; 78(3): 508-11.

24.- Se trata de paciente femenino de 37 años de edad, G-3, C-2. Es ingresada a hospital presentando cefalea, acúfenos, fosfenos y epigastralgia en barra con embarazo de 34 semanas. E.F T/A 160/110, FC 84 x´, FR 18 x´, no presenta fiebre, somnolienta, sin agregados cardioventilatorios, hepatalgia. F.U. de 25 cm. Producto único vivo. FCF 110 lpm, genitales sin pérdidas ni modificaciones cervicales. Laboratorio: hb 9.8 g/dl, plaquetas de 54 mil, TP 11 seg TPT 27, TGO 160 ng/dl TGP 160 ng/dl, hiperbilirrubinemia indirecta, albuminuria 300 mg/dl, Ácido Úrico de 8.1 mg/dl, creatinina de 1.5 mg/dl. El diagnóstico más probable es:

a) Sindrome de hellp. b) Sx anticuerpos antifisfolípidos c) Púrpura trombocitopénica trombótica d) Hígado graso

DEFINICIÓN: • Es una complicación de la preeclampsia en la cual además de la Hipertensión Arterial y proteinuria hay presencia de anemia hemolítica, enzimas hepáticas elevadas y recuento bajo de plaquetas MANIFESTACIONES CLINICAS:

• Malestar general, fatiga y molestias inespecíficas 90% • Cefalea 70% • Epigastralgia 64% • Vómito 22% • Fosfenos 15% • Visión Borrosa 11% • Acùfenos 3% • Ictericia • Anemia no explicada • Oliguria

BIBLIOGRAFIA: Sibai baha, El síndrome HELLP. Universidad de Valencia , revista quincenal de Obstetricia clínica y ginecología, Octubre 2003. V. Cararach, Síndrome de HELLP y Repercusiones maternas. X curso intensivo de formación continuada materno fetal. Enero de 2003. Toirac, Abelardo. Síndrome de Weistein HELLP Hospital Ginecoobstetrico Tamara Bunke. Junio 2002 De la Fuente, David. Síndrome HELLP. Medicina Universitária 2003; 5 (19): 101 -9 Andrea G. Witlin, DO, Baha M. Sibai, MD. Diagnosis and Management of women with Hemolysis Elevate Liver Enzymes, and Pletelet Count (HELLP) syndrome. Hospital Physician. Febrero 1999. CIFUENTES B, Rodrigo. Ginecología y obstetricia.

25.- Se trata de femenino de 24 años, G-1, que cursa con embarazo de 37 semanas de gestación, presenta pérdida del estado de alerta posterior a crisis convulsivas tónico-clónicas, signos vitales con T-A 170.120mmhg Fc 95x´, reflejos osteotendinosos aumentados, se aprecia una Fc fetal de 132x´ y edema importante de miembros inferiores, no se aprecian datos de trabajo de parto ni modificaciones cervicales, El diagnóstico más probable es:

a) Pre eclampsia severa b) Crisis epileptica de gran mal c) Hipertensión inducida por el embarazo d) Eclampsia

CUADRO 1. DIAGNÓSTICO* Preeclampsia Leve: Se presenta después de la semana 20 de gestación, durante el parto, o en las primeras 6 semanas después de éste Presión sistólica ≥ a 140 mm Hg o presión diastólica ≥ 90 mm Hg Proteinuria ≥ a 300 mg / orina de 24 hrs o su equivalente en tira reactiva Preeclampsia Severa: Se presenta después de la semana 20 de gestación, durante el parto, o en las primeras 6 semanas después de éste Presión sistólica ≥ a 160 mm Hg o presión diastólica ≥ 110 mm Hg Proteinuria ≥ a 2 gr en orina de 24 horas o su equivalente en tira reactiva Creatinina sérica > a 1.2 mg/dl Trombocitopenia ≤ 150 000 cel/mm3 Incremento de la deshidrogenasa láctica ≥ a 600 UI Elevación al doble de los valores de TGO/AST o TGP/ALT Cefalea, alteraciones visuales o auditivas Epigastralgia Oliguria ≤ a 500 ml en 24 horas 7 Edema agudo de pulmón Dolor en hipocondrio derecho Restricción en el crecimiento intrauterino Oligohidramnios Eclampsia Preeclampsia mas convulsiones sin otra causa. Se presenta después de la semana 20 de gestación, durante el parto, o en las primeras 6 semanas después de éste. Síndrome de HELLP Criterios para establecer el diagnóstico del síndrome de HELLP: Plaquetas < 100 000/mm3 TGO/AST ≥ 70U/L DHL ≥ 600U/LBilirrubina total > 1.2 mg/dl Se presenta después de la semana 20 de gestación, durante el parto, o en las primeras 6 semanas después de éste. Hipertensión Crónica: Se diagnostica cuando existe hipertensión arterial ≥ a 140/90 mm Hg antes de la semana 20 de gestación o si persiste después de doce semanas posteriores al parto. Las pacientes con hipertensión crónica deben ser evaluadas antes del embarazo para determinar la severidad de la hipertensión y facilitar la planeación de un embarazo mediante el cambio de medicamentos y de hábitos higiénicos y dietéticos para evitar complicaciones. Hipertensión Gestacional: Presencia de hipertensión arterial ≥ a 140/90 mm Hg después de la semana 20 de gestación y se mantiene hasta las doce semanas después del parto Ausencia de proteinuria Presencia o no de cefalea, acúfenos y fosfenos Después de 12 semanas de la interrupción del embarazo se revalorará la presencia de hipertensión, si continúa, se reclasifica como hipertensión crónica: es un diagnóstico retrospectivo. 8 Si no hay, se clasifica como hipertensión transitoria. 1. Aagaard-Tillery KM, Belfort MA. Eclampsia: morbidity, mortality, and management. Clin Obstet Gynecol 48:12-23, 2005.

2. Atallah AN, Hofmeyr GJ, Duley L. Calcium supplementation during pregnancy for preventing hypertensive disorders and related problems. Cochrane Database Syst Rev 1:CD001059, 2001. 3. Barton JR, Sibai BM. Diagnosis and management of hemolysis, elevated liver enzymes, and low platelets syndrome. Clin Perinatol 31:807-33, 2004. 4. Baxter JK, Weinstein L. HELLP syndrome: the state of the art. Obstet Gynecol Surv 59:838-45, 2004. 5. Cetin A. Eclampsia. In Mohler III ER, Townsend RR. Advanced therapy in hypertension and vascular disease. Ontario: B.C. Decker Inc. pp. 407-15, 2006. 6. Cetin A. Hemolysis, elevated liver enzymes, and low platelets (HELLP). In Mohler III ER, Townsend RR. Advanced therapy in hypertension and vascular disease. Ontario: B.C. Decker Inc. pp. 416-20, 2006. 7. Chappell LC, Seed PT, Briley AL, Kelly FJ, Lee R, Hunt BJ, Parmar K, Bewley SJ, Shennan AH, Steer PJ, Poston L. Effect of antioxidants on the occurrence of pre-eclampsia in women at increased risk: a randomised trial. Lancet 354:810-16, 1999. 26.- Femenino que cursa con 12 semanas de gestación, refiere tres días con sangrando por genitales, con náuseas continuas y constantes, útero mayor aumentado de tamaño y valores de beta HCG muy elevados, debemos sospechar de: a) Amenaza de aborto. b) Aborto diferido. c) Mola hidatídica. d) Amenaza de aborto en un útero con miomas. La enfermedad trofoblástica gestacional agrupa a diferentes entidades interrelacionadas: mola completa, generalmente diploide con origen cromosómico paterno, mola parcial generalmente triploide, tumor trofoblástico del lecho placentario y coriocarcinoma, con tendencias variables a la invasión local y a las metástasis, cuyo denominador común es la hipersecreción de hCG. El coriocarcinoma es diploide y proviene de ambos progenitores, excluyendo probablemente su origen directo en la mola completa. El tumor trofoblástico del lecho placentario está constituido por trofoblasto mononuclear intermedio no conteniendo vellosidades coriónicas e inmunohistoquimicamente caracterizado por expresar muchas de sus células hPL y unas pocas hCG.

Cuadro clínico

Tras un periodo de amenorrea, y a partir del segundo mes, hay un aumento de los síntomas subjetivos del embarazo, sobre todo náuseas y vómitos o hiperémesis en un 30%. Hay metrorragias irregulares en el 96% de los casos, en principio de escasa cantidad, pero que se va incrementando. Dicha hemorragia no es continua sino que se repite cada dos o tres días, de color roja o negruzca, que se produce por la ruptura de vasos maternos al separarse las vesículas de la decidua.

Ocasionalmente, hay expulsión de restos molares, que lo refiere la paciente como expulsión de vesículas en el 11 % de los casos y que es patognomónico pero aparece tardíamente. También puede presentar mal estado general, dolor difuso en hipogastrio y anemia. Esta última en relación con las metrorragias.

Pueden haber signos y síntomas de hipertiroidismo, como taquicardia, sudoración, y temblores, en el 7%, y es debido a que la fracción β-hCG es similar a la hormona TSH. Existen signos de preeclampsia o hipertensión gestacional del primer trimestre hasta en el 50% de los casos, y signos de insuficiencia respiratoria aguda en el 2% por embolismo pulmonar de las células trofoblásticas, o por la asociación entre hipertiroidismo e hipertensión arterial. Como complicaciones pueden aparecer coagulopatías y metástasis.

A la exploración el útero esta aumentado de tamaño, en el 50% de los casos, por encima de lo que correspondería a la edad gestacional,3 de consistencia blanda, sin signos de actividad fetal a partir de la semana 12 y siempre que se trate de una mola total. También puede que la paciente presente un tamaño uterino menor al esperado para la edad gestacional.3 El cuello está cerrado, con metrorragia en cantidad variable, y raramente se observa la expulsión de vesículas. Pueden haber quistes teca-luteínicos bilaterales en ovario en cerca del 20% de los casos,3 debido al estímulo de la β-hCG.

La enfermedad trofoblástica maligna va a cursar con metrorragias por lo general intensas, hay una elevación de la β-hCG y en la ecografía se objetiva la cavidad uterina con signos de ocupación atípica.

Diagnóstico

Por la clínica, y pruebas complementarias como la determinación de la β-hCG y la ecografía. La determinación de la β-hCG se basa en que el trofoblasto produce la hormona gonadotropina coriónica, presentando cifras elevadas, y su cuantificación va a servir para diagnóstico, valorar el pronóstico, y el seguimiento postratamiento. La ecografía revela un útero aumentado de tamaño que no corresponde con la amenorrea, con ecos en su interior, puntiformes que corresponderían a las vesículas y que asemejan copos de nieve o panal de abeja. No se aprecia saco gestacional ni estructuras fetales y, en ambos ovarios se aprecian quistes teca-luteínicos como formaciones ováricas redondas, econegativas, con múltiples tabiques en su interior

BIBLIOGRAFÍA

1. Mazur MT, Kurman RJ. Gestational trophoblastic disease and related lesions. En: Kurman RJ editor. Blaunstein’s pathology of the female genital tract. 4th ed. New York: Springer-Verlag. 1994, p. 1049-93.

2. Kurman RJ, Young RH, Norris HJ, Main CS, Lawrence WD, Scully RE. Immunocytochemical localization of placental lactogen and chorionic gonadotrophin in the normal placenta and trophoblastic tumors, with emphasis on intermediate trophoblast and the placental site trophoblastic tumor. Int J Gynecol Pathol 1984; 3: 101-21.

3. Berkowitz RS, Golstein DP. The management of molar pregnancy and gestational trophoblastic tumours. En Knapp RC, Berkowitz RS, editores. Gynecologic Oncology, 2nd ed. New York: Mc Graw-Hill 1992, p. 328-38.

4. De Agustín P, Ruiz A, López F, Contreras F. Patología de la enfermedad trofoblástica. Simposio Enfermedad Trofoblástica 1972; 79-98.

5. Salem S. Ultrasound diagnosis of trophoblastic disease. En: Sanders RC, James AE(Jr) editores. Ultrasonography in Obstetrics and Gynaecology. New York: Appleton-Century Crofts: 1977; p. 255-66.

6. Silverberg SG, Kurman RJ. Tumors of the uterine corpus and gestational trophoblastic disease. En: Rosai J, Sobin LJ, editores. Atlas of tumor pathology: tumors of the uterine corpus and gestational trophoblastic disease, fasc. 3, ser. 3. Washington DC: Armed Forces Institute of Pathology; 1992, p. 219-85.

Szulman AE, Surti U The syndromes of hydatiform mole II. Morphologic evolution of the complete and partial mole. Am J Obstet Gynecol. 1978; 132: 20-7. 27.- Mujer que acude al servicio de ginecología por referir ciclos opso-menorreicos desde el inicio de su menarquia, ha incrementado 15 Kg. de lo que pesaba habitualmente, se aprecia una gran cantidad de acné, pero además refiere depilarse el área del bigote cada semana, y cree que esto le sensibiliza la piel para que aumente el acné. En el caso de ovario poliquístico el dato clínico que con más frecuencia les acompaña es:

a) anovulación y esterilidad b) Hirsutismo c) Amenorrea d) Obesidad

El síndrome de ovarios poliquísticos (SOPQ) afecta aproximadamente a un 4% de mujeres en edad reproductiva y se caracteriza por anovulación crónica e hiperandrogenismo. Es la causa más común de infertilidad en mujeres. Se caracteriza clínicamente por acné, alopecia, hirsutismo, irregularidades menstruales e infertilidad. Los hallazgos de laboratorio más frecuentes son: aumento de la hormona luteinizante (LH), aumento de la relación LH/FSH (hormona folículoestimulante), aumento de andrógenos (tanto ováricos como adrenales) y de estrógenos circulantes. Otros hallazgos de laboratorio habituales son una prueba tolerancia oral a la glucosa anormal y alteraciones en el perfil lipídico. Todo esto junto con las imágenes ecocardiográficas características define al síndrome. La terapéutica permite dos grandes enfoques que pueden superponerse: la corrección de las manifestaciones de hiperandrogenismo y el tratamiento de las alteraciones del eje reproductivo (anovulación, esterilidad). Los antiandrógenos están fundamentalmente indicados para tratar los síntomas virilizantes. Las alternativas para inducir la ovulación son numerosas: al citrato de clomifeno y a la antigua resección en cuña se agregan las gonadotrofinas humanas, pulsos de GnRH (hormona liberadora de gonadotrofinas), medidas o fármacos para modificar los niveles de insulina, y finalmente técnicas quirúrgicas endoscópicas para reducir la masa ovárica.

BIBLIOGRAFIA

1. Guzick D.Polycystic ovary syndrome: Symptomatology, pathophysiology, and epidemiology. Am J Ostetric Gynecol 1998; 179 (6): 89-93.

2. Stephen Franks. Polycystic ovary syndrome. N Engl J Med 1995; 333(13): 853-861. 3. Gori J.R., Larusso A. Ginecología de Gori. 2ª Edición. Buenos Aires, Argentina.

Editorial El Ateneo. 2001. 4. Adams J., Polson D. W., Franks S. Prevalence of polycystic ovaries in women with

anovulation and idiopathic hirsutism. Br Med J 1986; 293: 355-9. 5. Copeland L. J . Ginecología. Buenos Aires, Argentina. Editorial Panamericana. 1ª

Edición. 1994. 6. Ehrmann D.A., Rosenfield R.L., Barnes R.B., Brigell D.F., Sheikh Z. Detection of

functional ovarian hyperandrogenism in women with androgen excess. N Engl J Med 1992; 327:157-162.

7. Kahasar-Miller M., Conway Myers B., Boots L., Azziz R. Steroidogenic acute regulatory protein (StAR) in the ovaries of healthy women and those with polycystic ovary syndrome. Am J Obstet Gynecol 2001; 185(6): 1381-7.

8. Pérez Sánchez A. Ginecología. Santiago de Chile. Publicaciones Técnicas Mediterráneo. 3ª Edición. 1995.

9. Velázquez E., Mendoza S., Hamer T., Sosa F., Glucck C. Metformin therapy in women with polycistic ovary syndrome reduces hiperinsulinemia, insulin resistence,

hyperandrogenemia, and systolic blood pressure, while facilitating menstrual regularity and pregnancy. Metabolism 1994 ; 43: 647-655.

28.- Femenino de 29 años. Acude al servicio de urgencias por presentar salida de líquido vaginal. Antecedentes: G2, P1, cursa embarazo de 36 semanas de gestación exploración física: cervix con 10% de borramiento, 1 cm de dilatación y Tarnier positivo. La complicación más frecuente en esta paciente es: a) Corioamnioitis. b) Parto pre término. c) Sepsis neonatal. d) Endometritis. Corioamnioitis: El diagnóstico de la infección intraamniótica (IIA) es básicamente clínico. La corioamnionitis se debe descartar en toda gestante que presente fiebre sin foco aparente, sobre todo si se sospecha o se ha confirmado una rotura de membranas. Los criterios más empleados para el diagnóstico son: fiebre materna y, al menos, 2 de los siguientes signos: taquicardia materna, taquicardia fetal, irritabilidad uterina, leucocitosis materna o líquido amniótico purulento o maloliente. Progresos de obstetricia y ginecología: revista oficial de la Sociedad española de ginecología y obstetricia, ISSN 0304-5013, Vol. 48, Nº. 6, 2005 , pags. 316-317 29.- Mujer de 23 años diagnosticada de E. Ectópico a nivel ampular, con saco gestacional menor de 3 cm., sin actividad cardíaca embrionaria, asintomática y con niveles de B-HCG menores para su edad gestacional. El tratamiento más indicado es: a) Salpingocentesis b) Resección segmentaria c) Tratamiento médico con Metrotexate. d) Histerectomía total con doble anexectomía. TRATAMIENTO

METOTREXATE Ácido 4 amino 10 metil folínico, antagonista del ácido fólico Glucosa hiperosmolar Prostaglandina F2a Actinomicina D Mifespristona

TRATAMIENTO CON METOTREXATO

Mayor éxito:

Embarazo menor a 6 SDG Masa tubaria menor de 3.5cm Feto sin latido cardíaco

PROTOCOLO CON DOSIS ÚNICA DE METOTREXATE Día 0 hGC, Biometría hemática, transaminasas, creatinina, grupo sanguíneo Día 1 hGC Día 4 hGC Día 7 hGC

Disminución de hCG <15%. Segunda dosis Si la hCG declina seguirla semanalmente Si la hCG está en meseta o en ascenso, segunda dosis El raspado endometrial se realiza sólo en pacientes con hCG <2000 mIU/mL al

momento de comenzar el tratamiento Graczykowski JW, Mishell DR. Methotrexate prophylaxis for persistent ectopic pregnancy after conservative treatment by salpingostomy. Obstet Gynecol. 30.- Femenino de 19 años acude a consulta por padecer un cuadro de dolor abdominal de inicio súbito, refiere alteraciones en su ciclo menstrual. La exploración física revela una tumoración dolorosa en el anexo izquierdo. La prueba de embarazo es negativa. La radiografía muestra una masa opaca en la fosa ilíaca izquierda con áreas de calcificación. El diagnóstico clínico más probable es:

a) Cistadenoma mucinoso

b) Teratoma quístico

c) Quiste folicular

d) Cistadenoma seroso

Los tumores de células germinales constituyen casi el 20% de los tumores de ovario y de ellos un 95% son benignos, siendo el tipo más frecuente el teratoma maduro benigno o también denominado quiste dermoide (1).

Aproximadamente el 80% se presentan en mujeres en edad fértil. Se originan a partir de células embrionarias pluripotenciales presentes habitualmente en ovario, testículo, mediastino, retroperitoneo y región sacrocoxígea, esto explica que la coloración de las faneras del quiste coincida con el fenotipo del paciente (2).

El teratoma quístico benigno con relativa frecuencia es un tumor bilateral (del 7 al 25% según los autores) (1) y se caracteriza por una cápsula gruesa, bien formada, revestida por epitelio plano estratificado. Bajo este se pueden encontrar una variedad de apéndices cutáneos que incluyen glándulas sudoríparas, apocrinas y sebáceas. La cavidad se llena de los detritus de éste y sus anexos, que es de color amarillo pálido, grasoso, espeso y suele contener pelo. Otros tejidos que se pueden encontrar son dientes, cartílago, plexos coroideos, falanges, tejido nervioso y en ocasiones tejido tiroideo (struma ovarii) con potencial tirotóxico o de degeneración maligna tiroidea.

La mayor parte de los quistes dermoides son asintomáticos y la forma más frecuente de presentación son el dolor abdominal (48%) y hemorragia uterina anormal o concomitante (15%) o aumento del volumen abdominal (15%). La rotura de un quiste dermoide es rara, entre el 1-1,2% y constituye una urgencia quirúrgica (3).

El tratamiento es quirúrgico, siendo posible la mayor parte de las veces una resección del mismo, respetando el resto del ovario.

La ecografía constituye el modelo diagnóstico de elección y la combinación de ecografía con radiografía simple de abdomen proporciona un diagnóstico más exacto en la mayoría de los casos, siendo la resonancia magnética o la TAC el que aporte el diagnóstico diferencial.

Las Rx de quiste dermoide se caracteriza por una cápsula bastante radioopaca y el líquido oleoso que contiene es radiolúcido, esta conjugación presenta muchas veces una característica de aspécto radiológico de calcificación en la pared del mismo.

Cólico nefrítico, teratoma ovárico y radiografía simple de aparato urinario Romero Pérez P, Martínez Hernández MªC.

Servicio de Urología. Policlínico San Carlos Denia (Alicante).Actas Urol Esp. 2007:31(8):936-937ACTAS UROLÓGICAS ESPAÑOLAS SEPTIEMBRE 2007

31.- Se trata de femenino de 33 años que acude al servicio cursa en éste momento con diagnóstico de preclampsia , el fármaco de elección que se administra en esta patología es:

a) Nifedipina. b) Inhibidores de la enzima convertidora de angiotensina. c) Clonidinas. d) Alfametildopa.

• Prevenir complicaciones a corto plazo de las mujeres con PA elevada que comprometa el bienestar fetal

• Cuando la PAS es mayor o igual a 150 mmHg y la PAD mayor o igual a 100 mmHg.

El propósito es alcanzar cifras de TA alrededor de 140/90. La medicación antihipertensiva se reserva para los casos en que la PAD ≥ 100 mmHg. Se recomienda continuar el tratamiento antihipertensivo previo al embarazo,

exceptuando el uso de IECA. La alfametildopa y la hidralazina vía oral son los fármacos de elección dado su uso

extensivo con seguridad y eficacia y sin efectos colaterales para el feto (excepto hidralazina en lupus).

• ALFA METILDOPA 500-2000 MG/DÍA

• HIDRALAZINA 50-200 MG/DÍA

• LABETALOL 100-400 MG/DIA

• ATENOLOL 50-200 MG/DÍA

• NIFEDIPINA 10-30 MG/DÍA

1. Aagard K, Belfort M. Eclampsia: Morbility, mortality, and management. Clin Obstet Gynecolol. 2005; 48: 12-23. 2. Oyarzún E. Síndrome hipertensivo del embarazo en Oyarzún E. Ed. Embarazo de alto riesgo. Ediciones Universidad Católica de Chile. Santiago. 1997: 157- 175. 3. Roberts J, Redman C. Pre-eclamsia: More than pregnancy induced hypertens 32.- Se trata de femenino de 56 años de edad refiere aumento de volumen a nivel abdominal, con predominio de hemiabdomen inferior con la siguiente sintomatología: plenitud, estreñimiento, se acompaña de USG pélvico que demuestra imagen quística en ovario derecho de 15 por 15 cms. El diagnóstico más probable es:

a) Teratoma quístico. b) Disgerminoma. c) Endometrioma. d) Cistadenoma seroso.

Los Tumores de Ovario son una patología frecuente dentro del contexto de la patología femenina. Por esta causa consultan un grupo elevado de mujeres, tanto las consultas de ginecología como las de Cirugía propiamente dicha. Las edades oscilan desde las tempranas hasta las ya avanzadas, siendo el riesgo de degeneración maligna muy variable y relacionado con le edad. La experiencia de la clínica revela la alta incidencia de tumores de ovario en la etapa del climaterio, comprendida entre los 35 y 65 años de edad 1. El cistoadenoma seroso de ovario (CSO) es un tipo de tumor derivado del epitelio superficial (celómico), formado por áreas quísticas. El cistoadenoma seroso de ovario es el tumor más frecuente de aquellos que provienen del epitelio celómico superficial. Hay tumores pequeños macroscópicamente y tumores masivos que ocupan toda la pelvis e incluso la cavidad abdominal. Estas frecuentes neoplasias quísticas uniloculares están tapizadas por células epiteliales altas, cilíndricas y ciliadas, llenas de un líquido seroso claro y de superficie lisa con abundantes vasos. Las variedades benigna, limítrofe y maligna representan, en conjunto, 30% aproximadamente de todos los tumores del ovario. El riesgo de presentar tumores epiteliales se incrementa con el paso de la edad, ya que pese a que la declinación de la función ovárica marca el envejecimiento gonadal progresivo, el ovario humano nunca pierde su capacidad para generar tumores. Por lo general, cuando es detectado, su tamaño es grande, en donde la imagenología puede ayudarnos a considerar su diagnóstico

1. Capítulo 22 Tumores Benignos de Ovario. En: Novak ER, Jones G., Jokes HW. Tratado de Ginecología. 9 ed. Ciudad de la Habana. Editorial Científico Técnica; 1977.p.432 – 66. 2. MedlinePlus Enciclopedia Médica en Español: Quistes Ováricos. Disponible en: http://vsearch.nlm.nih.gov/vivisimo/cgibin/querymeta?v%3Aproject=medlineplusspanish&spell=spell&query=Quistes+Ov%C3%A1ricos Acceso: Actualizado 20/6/06.

Capítulo XL Tumores Ováricos En: Llusiá Botella J, Núñez Clavero JA. Tratado de Ginecología. Ciudad de la Habana. Editorial Científico Técnica. 1983; T 3.1; p. 751 – 803. 33.- Paciente femenino de 27 años de edad con deseo de un embarazo, antecedentes de G3 A2 - P1 se le realiza una histerosalpingografía, se constata que existe un síndrome de Asherman. Ello significa que se trata de:

a) Útero bicorne b) Endometriosis en la trompa c) Sinequias uterinas d) Insuficiencia istmico cervical

El síndrome de Asherman es una enfermedad ginecológica rara que se caracteriza por la presencia de sinequias (adherencias) intrauterinas que pueden ocasionar amenorrea (ausencia de períodos menstruales regulares) e infertilidad.

En 1894 Heinrich Fritsch describe por primera vez la presencia de sinequias intrauterinas de tipo postraumático, en una paciente que desarrolló una amenorrea secundaria a un curetaje. Posteriormente en 1927 Bass informó de veinte casos de atresia (oclusión de una abertura natural) cervical tras abortos inducidos, pero no fue hasta 1948, cuando Joseph G. Asherman recopiló la información hasta entonces existente y acuñó el nombre con el que se conoce actualmente a la enfermedad.

Asherman describió originalmente dos tipos diferentes de amenorrea secundaria, en función de su etiología (estudio de las causas de las enfermedades): la amenorrea traumática atrética, debida a estenosis del orificio cervical interno y la amenorrea debida a adherencias intrauterinas. Posteriormente ambas entidades se agruparon en una única entidad bajo el nombre de síndrome de Asherman.

Suele presentarse en mayor proporción tras dilataciones y curetajes uterinos de repetición y sobre todo si se realizan durante el embarazo o si existe infección uterina en el momento en el que se realizan estas intervenciones.

Las adherencias intrauterinas pueden producirse debido a cualquier factor que lleve a una destrucción de las paredes del miometrio (capa muscular de la pared del útero). Sin embargo, hay que distinguir entre factores predisponentes, siendo el principal de ellos el embarazo y factores causales, entre los que se encuentran: traumatismos uterinos, intervenciones quirúrgicas que afecten al útero, agentes físicos o químicos e infecciones uterinas por tuberculosis o esquistosomiasis. En cualquier caso, el factor más importante es el trauma uterino en el momento del parto o el puerperio.

El cuadro clínico es muy variable y las manifestaciones clínicas varían con el grado de oclusión de la cavidad uterina y la severidad de las adherencias, pudiendo presentarse: esterilidad cuando la oclusión de la cavidad uterina incluye porciones proximales (más cerca de un centro, tronco o línea media) de las trompas de Falopio o cuando las adherencias impiden la nidación del huevo; las pacientes presentan con frecuencia amenorrea, oligomenorrea (disminución de la frecuencia de las menstruaciones), dismenorrea (menstruación dolorosa) y abortos repetidos.

1. Hysteroscopic treatment of severe Asherman's syndrome and subsequent fertility. Capella-Allouc S; Hum Reprod, 1999 May.

34.- Se trata de femenino de 31 años de edad, la cual inicia con hiperemesis gravídica de difícil control, así como sangrado trasvaginal. Se realiza el diagnóstico de mola hidatiforme. La primera opción de tratamiento indicado en esta patología es:

a) Histerectomía total abdominal. b) Legrado por aspiración. c) Metotrexate y seguimientos radiográficos. d) Legrado uterino instrumental

La enfermedad trofoblástica gestacional agrupa a diferentes entidades interrelacionadas: mola completa, generalmente diploide con origen cromosómico paterno, mola parcial generalmente triploide, tumor trofoblástico del lecho placentario y coriocarcinoma, con tendencias variables a la invasión local y a las metástasis, cuyo denominador común es la hipersecreción de hCG. El coriocarcinoma es diploide y proviene de ambos progenitores, excluyendo probablemente su origen directo en la mola completa. El tumor trofoblástico del lecho placentario está constituido por trofoblasto mononuclear intermedio no conteniendo vellosidades coriónicas e inmunohistoquimicamente caracterizado por expresar muchas de sus células hPL y unas pocas hCG

Tratamiento

Hay que tratar las complicaciones como la hiperémesis, anemia, hipertensión, y alteraciones electrolíticas, coagulopatías, alteraciones cardio-respiratorias y preeclampsia, procediendo a evacuar la mola lo antes posible, con lo que se producirán menos malignizaciones.

La evacuación del contenido uterino se realiza mediante dilatación, y legrado por aspiración. Además se pauta profilaxis antibiótica y oxitócicos.3

La histerectomía, con la mola en su interior, está indicada en pacientes de edad superior a 40 años o en mujeres con más de tres hijos, ya que en ambos grupos se ha demostrado una mayor incidencia de malignización.

. Tras la cirugía, se mide la concentración de gonadotropina coriónica humana para determinar si la extirpación ha sido completa. Si es así, el valor de esta hormona vuelve a la normalidad, en unas 8 semanas, y se mantiene en esos valores. Si una mujer a la que se le ha extirpado una mola queda embarazada, es difícil interpretar un valor alto de gonadotropina coriónica humana, porque podría estar causado tanto por el embarazo como por una parte de la mola que no se ha extirpado. En consecuencia, a las mujeres a las que se les ha extirpado una mola se les recomienda no quedar embarazadas durante un año. Las molas hidatiformes benignas no necesitan quimioterapia, pero las malignas sí. Los fármacos que se usan para este tratamiento son el metotrexato, la dactinomicina o una combinación de ambos.

Silverman L,Romero Zambrano F, Saldaño S. Enfermedad molar. Diagnóstico, tratamiento y seguimiento, 1987. 4- Puertas A, López Fernandez J et al. Enfermedad trofoblástica. Casuística del Hospital Virgen de las Nieves de Granada. Clín Invest Gin Obs 1993; 20: 98-103. 5- Enfermedad trofoblástica gestacional. Propuesta Normativa Perinatológica y Ginecológica de Alto Riesgo. Ministerio de Asuntos Sociales. Tucumán. 1996-1997; 19:171-183. 6- Jones. Enfermedad Trofoblástica Gestacional: qué hemos aprendido en la última década. Am J Gynecol Obstet 1990;162: 1286-1292.

7- Resúmen del simposio Enfermedad Troblástica Gestacional del Segundo Congreso Nacional de AGORA, 1990. 8- Gonzalez Merlo et al. Protocolos de diagnóstico y tratamiento en Obstetricia y Ginecología, Barcelona: editorial Salvat, 7:35-45. 35.- Se trata de femenino de 33 años que ingresa a la sala de urgencias, inicia con convulsiones por presentar preclampsia, decide iniciar tratamiento, el fármaco elección en ésta patología es:

a) Diacepam. b) Fenitoína. c) Donadores de óxido nítrico. d) Sulfato de magnesio.

Manejo de la Preeclampsia 1. Manejo ambulatorio: HTA sin proteinuria significativa, se recomienda el reposo en cama. Monitoreo de TA, peso, presencia de proteínas en orina. Ecografías periódicas para ver el feto y evaluar posibles retardo de crecimiento. 2. Manejo hospitalario: para mujeres con HTA inducida por el embarazo y 2+ o más o proteinuria significativa y en quienes falló el manejo ambulatorio. 3. Laboratorio y evaluación del peso: debe realizarse diariamente. Evaluación de la dinámica fetal. Monitoreo de síntomas como cefalea, alteraciones visuales y dolor epigástrico. 4. El parto es el tratamiento de elección: el cual debe realizarse cuando el feto está maduro pero puede realizarse en forma temprana si la salud de la madre está en peligro o si hay evidencia de distress fetal. El parto está indicado cuando la paciente cumple con los criterios de preeclampsia severa. Betametasona 12.5 mg IM dos veces por día puede estimular la maduración de los pulmones fetales. 5. Terapia antihipertensiva: está indicada sólo si la TA es persistentemente > 160/110 , es importante disminuir la TA hasta una diastólica de 90 a 100 mmHg porque la presión normal podría resultar en hipoperfusión de la placenta. Los diuréticos nunca están indicados, estas pacientes ya son hipovolémicas. Los IECA no deben ser usados durante el embarazo. Las medicaciones de largo plazo, incluyen alfa metildopa, atenolol y labetalol. 6. Terapia anticonvulsivante: A- Profilaxis de las convulsiones: está indicada en todas las pacientes pre-eclámpticas durante el trabajo de parto y el parto y por un mínimo de 24 hs luego del mismo. Algunos mantienen la terapia con magnesio hasta que comienza la diuresis. El Sulfato de Magnesio es la droga de elección. La dosis profiláctica es de 4 a 6 g de sulfato de magnesio IV y continúa con 2 g c/ hora. B- Tratamiento de las convulsiones: Sulfato de Magnesio 1 g/min IV hasta controlar las convulsiones hasta un máximo de 4 a 6 g. El nivel terapéutico es de 4 meq/l. Toxicidad del magnesio: ausencia de reflejo patelar, debilidad muscular, parálisis respiratoria y depresión cardíaca, 10 ml al 10 % de gluconato de calcio puede ser administrada IV. La terapia con sulfato de magnesio continúa por lo menos 24 horas en el post parto, la terapia puede detenerse si la excreción urinaria es > 200 ml/h por cuatro horas consecutivas. C- Prevención: 81 mg de aspirina diarios pueden ser administrados luego del primer trimestre en mujeres con hipertensión crónica o historia previa de preeclampsia, sin embargo la eficacia de esta indicación ha sido cuestionada. Myers JE, Baker PN. Hupertensive diseases and eclampsia. Curr Opin Obstet Gynecol 2002; 14: 119-125 2. Tierney, McPhee, Papadakis. Diagnóstico clínico y tratamiento 2003. 38ª ed, México, Manual Moderno, 2003: 770-773 3. Wilson MI, Goodwin TM, Pan VI, Ingles SA. Molecular epidemiology of preeclampsia. Obstet and Gynecol Survey 2003; 58(1):39-66

4. Burrow GM. Complicaciones médicas durante el embarazo. 4ª ed, México, McGraw-Hill panamericana: 1996: 1-25 5. Guyton AC, Hall JE. Embarazo y lactancia en: Tratado de fisiología médica, 10ª ed, México, McGraw-Hill Interamericana 2001: 1135-45 6. Vaticon D. Fisiología de la fecundación, embarazo, parto y lactancia, en: Tresguerres JAF. Fisiología Humana. México, Interamericana McGraw-Hill, 1992: 1086-1109 7. Pridjian G, Puschett JB. Preeclampisa. Part 1: Clinical and Pathophysiologic Considerations. Obstet and Gynecol Survey 2002; 57 (9): 598-618 8. Pridjian G, Puschett JB. Preeclampisa. Part I1: Experimental and Genetic Considerations. Obstet and Gynecol Survey 2002; 57 (9): 619-40 9. IMSS. Embarazo de alto riesgo. Guía diagnóstica terapéutica. Rev Med IMSS 1998; 36(1):45-60 36.- Se trata de femenino de 47 años que presenta sinusorrragia de 3 meses, sin alteraciones menstruales previas, por lo demás asintomática, el diagnóstico más probable a la especuloscopía es: a) Adenomiosis b) Malformaciones Müllerianas c) Pólipo endometrial d) Pólipo endocervical

Pólipo es toda formación sésil o pediculada que se fija a la cavidad uterina. Las formaciones polipoideas dentro del aparato genital femenino se dividen en endometriales y cervicales. Dentro de los pólipos cevicales se diferencian los ectocervicales y los endocervicales; estos últimos son considerados como los verdaderos por estar constituidos por epitelio cilíndricco. No se conocen con certeza los factores asociados a su histogénesis (teoría más admitida: hiperplasia focal de la mucosa endocervical por estímulo estrogénico) (1). Los pólipos cervicales se consideran la forma más frecuente de tumor benigno de cérvix.

Constituyen de un 3 a un 10% de las consultas ginecológicas; presentan máxima incidencia entre la 4° y 5° década de la vida. Algunos estudios demuestran que hasta en el 56% de casos de mujeres postmenopáusicas se encuentran asociados pólipos cervicales y endometriales; es mucho menor esta frecuencia antes de la menopausia (2). La clínica de presentación más común es la hemorragia intermenstrual, espontánea o durante el coito (3). También pueden ser asintomáticos (hallazgo casual en estudio ecográfico) o, en ocasiones, alcanzar grandes tamaños: llegan a aflorar por el orificio cervical externo; se denominan "pólipos gigantes" cuando superan 2 cm (4). Para el diagnóstico habitualmente se utiliza la ecografía. Los casos sintomáticos se consideran de indicación quirúrgica por su riesgo de degeneración a adenocarcinoma cervical.

Los pólipos cervicales constituyen una patología frecuente en la consulta ginecológica, sobre todo en mujeres mayores de 20 años que han tenido hijos y son poco comunes antes de la menarquia. En la mayoría de los casos, solamente se presenta un pólipo, aunque en ocasiones se pueden encuentrar 2 ó 3. La época más frecuente es al final de su vida reproductiva y después de la menopausia. En las primeras, las lesiones suelen ser más grandes, difíciles de tratar y con frecuencia recidivan. La causa de los pólipos cervicales aún no se ha comprendido completamente, pero con frecuencia son el resultado de una infección crónica, una respuesta local anormal a los niveles de estrógeno o a una congestión local de los vasos sanguíneoscervicales. Clínicamente se manifiesta con sangrado genital fuera de la menstruación, sobre todo durante la relación sexual. Son benignos, no se malignizan pero deben ser extirpados para controlar la sinusorragia.

37.- A 23-year-old nulligravid female has not menstruated in the past 4 months. Previously, her menstrual cycles were regular. She is otherwise well and denies recent onset of stress, change in exercise routine, headaches, visual field alterations, or galactorrhea.. She has a body mass index of 24, blood pressure of 120/78 mm Hg, and does not appear hirsute. No adnexal masses can be palpated. Laboratory investigations reveal a negative beta human chorionic gonadotropin (β-hCG), normal thyroid-stimulating hormone (TSH), and prolactin levels. What is the next best step in the management of this patient? a) Measurement of luteinizing hormone. b) CT scan of the sella turca. c) Prescribe oral estrogen for 21 days followed by 7 days of medroxyprgesterona and reevaluate. d) Preescribe 7 days of medroxyprgesterona and reevaluate. La Prueba de respuesta a progestágenos se basa en la observación de que el tratamiento con progestágeno (acetato de medroxiprogesterona 10 mg por 5 a 6 días) solo induce la menstruación en las mujeres con concentraciones normales de estrógenos circulantes. Una prueba positiva (hemorragia después de concluir el tratamiento con progestágenos) señala cifras normales de producción de estrógenos y una prueba negativa (sin hemorragia por privación), hipogonadismo franco. Danforth, Tratado de Obstetricia y Ginecologia, 9ª Edición, Ed. Mc Graw Hill Interamericana, Pág 668 38.- Se trata de femenino de 27 años, Gesta 1, Para 1. Con dos citologías “lesión de alto grado”, prueba de Schiller positiva y biopsia de cérvix que demuestra carcinoma “In Situ”. La conducta es:

a) Conización. b) Histerectomía total abdominal. c) Histerectomía y salpingooforectomía bilateral. d) Electrocauterización del cérvix.

La conización cervical es el tratamiento de elección en pacientes con cáncer cervicouterino microinvasor y más si existedeseo de fertilidad. Asimismo, la histerectomía extrafasciales un método adecuado en lesiones de 0.5 a 3 mm de invasión.Además se propone que, para pacientes con lesiones de 3.1 a 5 mm de invasión, a partir de la membrana inicial sinfactores de mal pronóstico como invasión vascular y linfática, sean tratadas con histerectomía extrafascial, ya que en aquellas a las que se realizó linfadenectomía pélvica, con este tipo de lesión, no se encontró metástasis a ganglios linfáticos. Resultados del tratamiento en cáncer cervicouterino microinvasor en el Instituto Nacional de Cancerología de México (1980-1999)

1.- Mestwerdt G. Fruhdiagnose des Kollumkarzinoms. Zentralb Gynaekol, 1947 ;69 :326. 2. - Morrow CP, Curtin JP. Surgery for cervical neoplasia. In Gynecologic Cancer Surgery. New York, Churchill Livingstone, 1996, p 472.3. 3. - Burghardt E, Holzer E. Diagnosis and treatment of microinvasive carcinoma of the cervix uteri. J Obstet and Gynecol 1977; 49:641-653. 39.- Se trata de femenino de 35 años la cual presenta un nódulo mamario palpable de aparición brusca. La ecografía revela un nódulo anecogénico, de limites muy precisos, morfología regular y refuerzo posterior, único de 3.5 cms. de diámetro. El diagnóstico más probable es: a) Cáncer. b) Displasia fibrosa. c) Fibroadenoma. d) Quiste Quistes. Los quistes mamarios son fáciles de detectar con la ecosonografía. Pueden ser lesiones únicas o múltiples que se observan como imágenes redondeadas, anecogénicas, de paredes delgadas, contornos bien definidos, con importante reforzamiento acústico posterior y sombras laterales delgadas. Pueden presentar septos intraquísticos y, en ocasiones, se pueden observar ecos internos que sugieren detritus celulares o proceso inflamatorio. Se debe descartar la presencia de lesiones intraquísticas o la coexistencia de otras alteraciones benignas o malignas. En caso de ser sintomáticos, el tratamiento adecuado es la punción y aspiración de la lesión con aguja guiada por palpación o ecosonografía de acuerdo con ell tamaño, profundidad y características del contenido. El uso del ultrasonido garantiza el vaciamiento completo.

REFERENCIAS: Barth V, Prechtel K. Mama normal. En: Barth V, Prechtel K, editores. Atlas de patología de la glándula mamaria. 2da ed. Madrid: Editorial Médica Panamericana, 1991. Bush H, McCredie A. Carcinoma of the breast during pregnancy and lactation. In: Allen HH, Nisker JA. Cancer in pregnancy. New York: Futura Publishing Co. Inc., 1986. Byrd BF, Bayer DS, Robertson JC, Stephenson JE Jr. Treatment of breast tumor associated with pregnancy and lactation. Ann Surg. 1962; 155:940-7. 40.- Femenino de 36 años, es atendida en consulta externa con reporte de papanicolaou que reporta un NIC I, la especuloscopía se observa cérvix con ectropión periorificiario. El agente etiológico más probable causante de esta infección es:

a) neisseria gonorreae. b) clamidya trachomatis. c) virus del papiloma humano. d) treponema pallidum.

9.5.2 Las pacientes a quienes se les realizó citología cervical, cuyo resultado es LEIBG (infección por VPH, displasia leve o NIC 1); LEIAG (displasia moderada y grave o NIC 2 y 3) o cáncer deben enviarse a una clínica de colposcopía, para realizar estudio colposcópico. 9.5.3 Si el resultado de la citología es LEIBG, la colposcopía es satisfactoria y sin evidencia de LEIBG, se realizará control citológico en un año (Apéndice Normativo A) 9.5.4 Si la citología es de LEIBG, la colposcopía es satisfactoria y existe evidencia de lesión, se debe tomar una biopsia dirigida. 9.5.4.1 Si la biopsia dirigida es negativa, se realizará nueva colposcopía para verificar el diagnóstico y en caso necesario, tomar nueva biopsia dirigida y revalorar. 9.5.4.2 Si la biopsia dirigida es reportada como LEIBG se podrá dar tratamiento conservador: criocirugía, electrocirugía o laserterapia (sólo si cumple con las condiciones referidas en el Apéndice 1) o se podrá mantener a la paciente en vigilancia en la clínica de colposcopía, con colposcopía y estudio citológico cada seis meses, durante 24 meses. Jueves 31 de mayo de 2007 DIARIO OFICIAL (Primera Sección) 9.5.4.3 Si la biopsia dirigida es reportada como LEIAG (Lesión Intraepitelial Escamosa de Alto Grado) se realizará tratamiento conservador (electrocirugía o laserterapia). En las mujeres posmenopáusicas, dependiendo de las condiciones anatómicas del cérvix, se realizará tratamiento conservador en la clínica de colposcopía o tratamiento quirúrgico (histerectomía extrafascial) en el servicio que corresponda. 9.5.4.4 Si la biopsia dirigida reporta cáncer microinvasor o invasor, la paciente se transferirá a un Servicio o Centro Oncológico para su tratamiento correspondiente. 9.5.4.5 Si la citología reporta LEIBG y la colposcopía es no satisfactoria, se tomará cepillado endocervical (Apéndice Normativo A) 9.6 En caso de colposcopía no satisfactoria, negativa a LEIBG y con cepillado endocervical negativo, se continuará su control en la clínica de colposcopía en seis meses, con colposcopía y citología. 9.6.1.1 Si el cepillado endocervical reporta LEIBG se tratará a la paciente como LEIAG, con métodos conservadores escisionales.

Jueves 31 de mayo de 2007 DIARIO OFICIAL (Primera Sección) Modificación a la Norma Oficial Mexicana NOM-014-SSA2-1994, Para la prevención, detección, diagnóstico, tratamiento, control y vigilancia epidemiológica del cáncer cérvico uterino. Al margen un sello con el Escudo Nacional, que dice: Estados Unidos Mexicanos.- Secretaría de Salud. MODIFICACION A LA NORMA OFICIAL MEXICANA NOM-014-SSA2-1994, PARA LA PREVENCION, DETECCION, DIAGNOSTICO, TRATAMIENTO, CONTROL Y VIGILANCIA EPIDEMIOLOGICA DEL CANCER CERVICO UTERINO. El agente etiológico del cáncer de cuello uterino es el “papiloma virus humano” (hpv).

Existen lesiones precursoras del cáncer cervical, son las llamadas lesiones intraepiteliales de cuello uterino (también conocidas como sil: squamous intraepithelial lesion) son lesiones que no atravesaron la membrana basal del epitelio y que por lo tanto no pueden invadir ni diseminarse por el resto del cuerpo, como sí lo hace un cáncer invasor. anteriormente a las lesiones intraepiteliales se las llamaba neoplasia intraepitelial cervical (también conocidas como cin: cervical intraepithelial neoplasia), y anteriormente se las llamaba displasias de cuello uterino.

41.- Paciente de 50 años, con mioma uterino de tamaño equivalente a una gestación de 12 semanas, que presenta hipermenorreas y hemoglobinemia de 9 gr%. No se demuestra patología asociada. Se encuentra en espera para la práctica de una histerectomía programada a realizar en 4 meses. En esta paciente está indicado el tratamiento preoperatorio con: a) Estrógenos. b) Inhibidores de la fibrinólisis. c) Análogos de la GnRH. d) Derivados del cornezuelo del centeno. · Análogos de la GnRH: Son derivados de la hormona GnRH en donde se ha realizado una sustitución peptídica en posición 6 y en algunos casos en la 10, obteniendo compuestos hasta unas 200 veces más potentes debido a mayor afinidad por los receptores y a su resistencia a la degradación por peptidasas. Aunque su acción inicial produce un incremento en la producción de FSH-LH (efecto flure-up o llamarada) tras 5- 6 días de exposición contínua, los receptores son internalizados produciéndose un estado de hipogonadismo hipogonadotropo y niveles de estradiol similares a los de la postmenopausia. Los análogos de GnRH están disponibles en distintas fórmulas: administración nasal (varias aplicaciones al día), subcutánea (aplicación diaria) o intramuscular (preparados depot mensuales o trimestrales) (Shaw RW 1999). Marco Filicori y sus colaboradores de la Universidad de Bolonia fueron los primeros en utilizar en 1983 los aGnRH en un estudio que confirmó su eficacia para reducir el tamaño de los miomas uterinos y secundariamente síntomas como alteraciones menstruales, dolor pélvico y síntomas de presión local. Otros autores como Minaguchi H y colaboradores continúan comprobando la efectividad de los análogos de la GnRH en el tratamiento del mioma uterino tras evaluar en el año 2000 seis estudios con un total de 602 pacientes tratadas con nafarelina. La disminución del tamaño se calcula entre un 30-70%, y se ha observado como el mayor porcentaje de reducción ocurre tras el primer mes de tratamiento, no existiendo reducciones o siendo éstas mínimas después del tercer mes (Healy et al 1986; Friedman et al 1989; Matta et al 1989; Williams y Shaw 1990). En miomas pediculados o con gran proporción de calcio o colágeno (hialinización) la repuesta es también

menor. Debe tenerse en cuenta que si después de dos meses de tratamiento no se ha producido un significativo descenso del tamaño del mioma, éste ya no debe ser esperado y debe pensarse en la posibilidad de la existencia de un tumor muscular maligno no diagnosticado (Messia AF et al 1998). Tras finalizar el tratamiento y recuperarse el estado de hipogonadismo, el mioma retorna rápidamente a su tamaño inicial (Friedman AJ et al 1987; Matta WH et al 1989). En casos próximos a la menopausia, la reducción del tamaño del mioma y su sintomatología, permitiría hablar de una solución médica del problema, pero en todo caso la utilización de aGnRH facilitaría la intervención quirúrgica al acortar el tiempo de intervención, la hemorragia y el acceso a localizaciones complicadas como el caso de miomas interligamentarios o situados en istmo o cérvix. En el caso de la cirugía histeroscópica la reducción del tiempo de cirugía permitiría reducir el volumen de fluidos aportado a cavidad uterina y los riegos de absorción e hiponatremia. Los mecanismos de acción por los que los aGnRH actúan son: o Hipoestrogenemia: es necesario mantener la hipoestrogenemia, pues la elevación de sus niveles lleva a un rápido incremento del tamaño del mioma. El crecimiento del mioma es dependiente de los niveles de estrógenos (aumentan de tamaño con el embarazo y se reducen durante la menopausia o el tratamiento con aGnRH, pudiendo volver a crecer durante la THS), pero aunque los estrógenos parecen ser importantes en el crecimiento del mioma, su relación debe ser algo más compleja pues no se han descrito incrementos significativos del tamaño de miomas durante el tratamiento con gonadotrofinas en RA (situaciones con elevados niveles de E2), algunos de ellos no se modifican durante el embarazo o incluso decrecen y se han encontrado crecimientos después del tratamiento con citrato de clomifeno (antiestrógeno). En relación con la hipoestrogenemia podrían estar los cambios inducidos en el flujo vascular uterino (incrementos en el índice de resistencia de las arterias uterinas) que suponen una reducción de la vascularización o las modificaciones de distintos factores de crecimiento. o Cambios histológicos: el tratamiento con aGnRH puede producir degeneración roja, infiltración linfocitaria, y necrosis, así como reducción de la proliferación celular e incremento de la apoptosis. Pero en otras circunstancias no es posible encontrar 7 diferencias. No se ha encontrado una relación entre los cambios histológicos y el porcentaje de reducción del tamaño del útero, y existe una gran variabilidad entre distintas pacientes o entre distintos miomas de una misma paciente, no existiendo pues un patrón histológico característico de respuesta ante el tratamiento con aGnRH. BIBLIOGRAFÍA: 1. Abad L, Abad de Velasco L, Parilla JJ. Etiopatogenia. Papel de las hormonas esteroideas, factores de crecimiento y otras sustancias. Cuad Med Reprod 1999;5(1):15-29. 2. Albano C, Platteau P, Devroey P. Gonadotropin-releasing hormone antagonist: how good is the new hope? Curr Opin Obstet Gynecol 2001;13(3):257-62. 3. Coutinho EM.Treatment of large fibroids with high doses of gestrinone. Gynecol Obstet Invest 1990;30(1):44-47. 4. Chavez NF, Stewart EA. Medical treatment of uterine fibroids. Clin Obstet Gynecol 2001;44(2):327-84. 5. De Leo V, la Marca A, Morgante G. Shortterm treatment of uterine fibromyomas with danazol. Gynecol Obstet Invest 1999;47(4):258-262. 6. Eldar-Geva T, Healy DL. Other medical management of uterine fibroids. Baillieres Clin Obstet Gynaecol 1998;12(2):269-88. 7. Felberbaum RE, Germer U, Ludwig M, Riethmuller-Winzen H, Heise S, Buttge I, Bauer O, Reissmann T, Engel J, Diedrich K. Treatment of uterine fibroids with a slow-release formulation of the gonadotrophin releasing hormone antagonist Cetrorelix.HumReprod 1998;13(6):1660-8

42.- Se presenta paciente de 40 años de edad a su consultorio refiriendo mastalgia que es más severa antes de la menstruación. A la palpación hay nodularidad excesiva, hiperestesia y áreas quísticas que la paciente refiere disminuyen en tamaño después de la menstruación. El diagnóstico más probable es:

a) Fibroadenomas b) Papiloma intraductal c) Mastopatía fibroquística d) Cáncer de mama

Es raro encontrar una mujer mayor de 35 años a quien no le hayan dicho, en un examen físico mamario, ecográfico o mamográfico, que tiene quistes en la mama o que su mama es mastopática. Es el trastorno benigno de la mama más frecuente y consiste en un aumento del tejido mamario, especialmente en las zonas superiores y externas de las mamas, hacia las axilas, que las hace más densas.

La mastopatía fibroquística suele presentarse en ambas mamas, aunque puede ser de diferente intensidad en una que en otra. Puede presentarse a cualquier edad después del inicio de la menstruación, pero es más probable que aparezca entre los 30 años y la menopausia. Raramente se presenta más tarde de esa edad. El origen de este trastorno es funcional y responde a desequilibrios de las hormonas sexuales femeninas y puede condicionar la aparición de quistes mamarios.

Los síntomas pueden fluctuar de leves a severos en una mastopatía fibroquistica mamaria, se acentúan típicamente antes de cada período menstrual y desaparecen inmediatamente después.

Los síntomas abarcan:

Consistencia de protuberancias (como de "guijarros"), irregular y densa del tejido mamario

o generalmente más notoria en la parte superior externa de la mama Molestia en las mamas

o generalmente en ambas mamas o puede ser persistente o puede aparecer y desaparecer

Sensación de llenura en las mamas Sensibilidad y dolor sordo e intenso Sensibilidad y edema premenstrual Secreción ocasional del pezón

Bibliografía:

Jones III HW, Wentz AC. Tratado de Ginecología de Novak. Editorial Interamericana-McGraw Hill. Undécima Edición 1994.

DiSaia-Creasman. Oncología Ginecológica Clínica. Editorial Mosby. Cuarta Edición 1994.

Van Dinh T. Sumario de Patología Ginecológica. Editorial La Prensa Médica Mexicana. 1992.

Pernoll ML. Diagnóstico y Tratamiento Ginecoobstétricos. El Manual Moderno. México. Sexta Edición 1991.

Alvarez-Bravo A. Diagnóstico de los trastornos menstruales y hemorrágicos. En: Alfonso Alvarez Bravo y su obra. Editorial Marketing y Publicidad SA. Tomo I. 1993.

Vázquez E. Aspectos histoquímicos del endometrio humano después del tratamiento con progestágenos sintéticos. Gac Méd Méx 1966; 96: 1277-93.

Huerta MR, Malacara JM, Rivera-Cisneros A, Díaz Cisneros FJ. Síntomas en adolescentes de dos ciudades de México y su asociación con el ciclo menstual. Ginec Obstet Méx 1994; 62: 146-50.

43.- En una consulta prenatal de rutina, una mujer de 28 años de edad, G5 P4, con 28 SDG, refiere que no ha sentido movimiento fetal durante los últimos 2 días. Su embarazo ha sido complicado debido a que padece hipertensión crónica, para lo cual se le recetaron tabletas de alfa-metildopa 2 veces al día. Al examen, su FU es de 30cm, y las maniobras de Leopold demuestran que el feto se encuentra en situación transversa. Su TA es 145/85mmHg. No se encuentra latido cardiaco con el Doppler. El paso más apropiado a seguir en el manejo de la paciente es: a) Realizar un test sin estrés b) Amniocentesis c) USG d) Beta-HCG (cuantitativa) Probable óbito: Sintomatología y diagnóstico Signos funcionales: No se perciben movimientos fetales por 12-24 horas. Disminución o ausencia de síntomas y/o signos como nauseas vómito, hipertensión, albuminuria) Paraclínicos: *ecografía: diagnóstico precoz y exacto: Doppler. *Radiología: hay 3 signos: +deformación del cráneo +curvatura y torsión de la columna +presencia de gas en el feto *líquido amniótico: puede estar meconiado, o sanguinolento Signos locales: en los senos hay secreción calostral, sangrado leve y oscuro por vagina, el feto se vuelve blando a la palpación, fetocardia (-), puede haber detención y/o disminución de la altura uterina, bajo peso corporal, entre otros Bibliografía: 1. OBSTETRICIA, Schwarcz R, editorial El ateneo, 2003. 2. Sociedad española de ginecología y obstetricia, junio 2002. 3. OBSTETRICIA CLÍNICA, Llaca V, edición 2000, capítulo 24; Pág, 315-316. 44.- Femenino de 34 años, es atendida en sala de partos secundario a eutocia, durante la reparación de la episiotomía media hay un marcado incremento en el sangrado transvaginal. El manejo inmediato a realizar en éste caso es:

a) 20 unidades IV de oxcitocina b) 0.2mg Im de metilergonovina c) Masaje y compresión del fondo uterino

d) Empaquetar con gasas

Atonía Uterina a) Compresión bimanual externa. b) Masaje uterino combinado (endouterino y abdominal). c) Medicamentos: uso de oxitócicos. d) Evacuación uterina de coágulos. e) Transfusión de sangre o hemoderivados. f) Evaluar factores de coagulación. g) De persistir el cuadro clínico, proceder a: 1. Ligadura de arteria uterovárica. 2. Sutura de las paredes uterinas (puntos de colchonero). 3. Sutura de Lynch. 4. Ligadura bilateral de arterias hipogástricas. 5. Histerectomía abdominal subtotal. 6. Histerectomía abdominal total.

h) Hemoglobina - hematocrito control seriado.

(1) Guía Clínica Basada en las Evidencias: Manejo de la Hemorrasia Postparto. Centro Latinoamericano de Perinatologia y Desarrollo Humano (CLAP). Novedades del CLAP no 16, abril 2002. (2) OMS/FNUAP/UNICEF/BM. Integrated Management Of Pregnancy And Chidbirth - IMPAC, 2000. (Traducción al Español: Manejo de las complicaciones del embarazo y el parto: Guía para obstetrices y médicos 2002 (3) OutLook. Prevención de la Hemorragia Postparto: Manejo del Tercer Período del Parto. Número 45.- Femenino de 25 años, con embarazo de término, sin antecedentes de control prenatal. G 3. C-1. Se ingresa al servicio de obstetricia por presentar actividad uterina regular y dolorosa. Ef.: Deambulante, tranquila, adecuada coloración de tegumentos, abdomen con fondo uterino a 32 cm. con producto único vivo en situación transversa dorso inferior FCF 144, al tacto vaginal cérvix dilatado a 3 cm. y membranas íntegras. Se realiza cesárea con retención de placenta e invasión a vejiga. La alteración placentaria que presenta esta paciente es:

a) placenta increta b) placenta acreta c) placenta percreta d) placenta marginal

Es la penetración y adherencia anormal de la placenta en la pared uterina.

Se divide en: Placenta acreta. Placenta increta. Placenta percreta.

ACRETA: Las vellosidades se adhieren al miometrio.

INCRETA: Penetran más de la mitad del espesor del miometrio.

PERCRETA: Atraviesa todo el espesor del miometrio, llegando a la

serosa, incluso atravesándola y adhiriéndose a órganos vecinos. Factores:

Endometriosis previa. Tumores submucosos. (Miomas) Cicatríz uterina previa. (Cesárea, miomectomía) Implantación baja. (Placenta previa) Malformaciones placentarias. (Placenta extracorial) Legrado enérgico previo. Extracción manual previa de una placenta.

Diagnóstico transparto-:

Placenta retenida por más de 20 minutos. Imposibilidad para encontrar un plano de separación placentaria cuando se intenta su

extracción manual. Hemorragia incontrolable después de la pseudoextracción. El diagnóstico histopatológico corrobora el diagnóstico clínico. Escenario menos deseable.

Tratamiento:

Histerectomía Obstétrica. Constituye una cirugía no planeada y secundaria al hallazgo del acretismo

placentario con sangrado incohercible.

Cesárea-Histerectomía. (Con diagnóstico previo) Cirugía planificada ante un correcto diagnóstico prenatal.

Recomendación ACOG:

Maduración pulmonar intrauterina. Inyectar al cordón umbilical 50 mg de metrotexate. Ligar el cordón en el nacimiento placentario y dejar la placenta in-situ. Embolización inmediata de arterias uterinas bilaterales, así como de ramas de

la división anterior de la arteria iliaca interna con alcohol polivinílico. Continuar con 5 dosis I.M. de 50 mg de metrotexate y cuantificar niveles de

βhCG. Programar Histerectomía Total Radical Abdominal y/o Cistectomía parcial y/o

resección pared anterior recto. Lee et al. Conservative Management of Placenta Percreta. Obstet Gynecol, 112(2):421-424 46.- Femenino 40 años, G-3 P-2 A-1, se detecta de anemia ferropénica, de 9.5 g/dl, refiere ciclos menstruales de 31,32 x 8,9 días de duración, acompañados de coágulos, los cuales aparecieron después del nacimiento de su segundo hijo hace 12 años. e.f.: con ligera palides de tegumentos, S/V dentro de los parámetros normales, , genitales con evidencia de sangrado activo, al tacto vaginal bimanual se detecta útero de consistencia firme voluminoso, irregular, aproximadamente de 12 cm. anexos libres. El tratamiento para esta paciente es:

a) histerectomía total sin conservar anexos. b) histerectomía total conservando anexos. c) histerectomía vaginal. d) histerectomía radical.

Tratamiento: La cirugía es el tratamiento más común del mioma uterino: Miomectomia: Extirpar sólo el mioma, los tumores únicos y accesibles, la resección histeroscópica de miomas submucosos también es posible realizarlo con electrocoagulación en pacientes con hemorragias (>90%). Histerectomía: La cirugía puede ser abdominal o laparoscópica, la decisión final de la técnica dependerá de la elección del cirujano, en base al caso individual y a su experiencia, el procedimiento puede ser: -histerectomía subtotal. -histerectomía total (remoción cervical). Técnicamente más sencillo en su realización. La incidencia de carcinoma con origen en el muñón cervical es menor al 1% en nuestros días.

Bibliografía: 1. hanafi m. predictors of leiomyoma recurrence after myomectomy. am coll obstet ginecol 2005; 105: 877-880. 2. inclan j, mojarra j. miomectomía histeroscópica. abordaje actual para el manejo de los miomas submucosos. reporte de un caso y revisión de la literatura. bol clin hosp. infant edo son 2001;18: 29-34. 3. goldrath mh, husain m. the hysteroscopic management of endometrial leiomyomatosis. j am assoc gynecol laparosc 1997; 4: 263-267. 4. clement pb, scully re. mullerin adenofibroma of the uterus with invasion of myometrium and pelvic veins. int j gynecol pathol 1990; 9: 363-371. 5. ravina jh, herbreteau d, ciraru-vigneron n, bouret jm, houdart e, aymard a, merland jj. arterial embolization to treat uterine myomata. lancet 1995; 346: 671-672.

6. kunhardt-urquiza e, cruz si, fernández-martínez rl, hernández-zúñiga ve: miomatosis de localización poco frecuente. ginecol obstet mex 1997; 65: 541-544. 47.- Paciente femenino de 18 años diagnosticada de embrazo ectópico mediante ecografía que también muestra gran cantidad de líquido libre en Douglas, con mal estado general. No tiene hijos y conserva la trompa contralateral en perfecto estado. El tratamiento de elección es: a) Expectante y determinación del B-HCG. b) Salpingectomía. c) Legrado de cavidad uterina. d) Alta y revisión en una semana

SANGRADOS 1era MITAD EMBARAZOEMBARAZO ECTOPICO – Tratamiento

• Quirúrgico.– Salpingostomía lineal.– Salpingectomía total.– Ordeñamiento fimbrial.– Legrado uterino instrumental post-laparoscópico.

(Reacción deciduoide)

• Médico.– Metrotexate y Mifepristona (RU 486)

48.- Se trata de femenino de 31 años nuligesta pero con actividad sexual regular, sin método de planificación familiar, con ciclos regulares, sin leucorrea, refiere dispareunia profunda, sangrado intermestrual y dismenorrea secundaria ocacionalmente presenta urgencia urinaria. El diagnóstico clínico más probable es:

a) Enfermedad pélvica inflamatoria b) Absceso tubo ovárico c) Endometritis d) Endometriosis

La endometriosis consiste en la aparición y crecimiento de tejido endometrial fuera del útero, sobre todo en la cavidad pélvica como en los ovarios, detrás del útero, en los ligamentos uterinos, en la vejiga urinaria o en el intestino. Es menos frecuente que la endometriosis aparezca fuera del abdomen como en los pulmones o en otras partes del cuerpo.

La endometriosis es una enfermedad relativamente frecuente, que puede afectar a cualquier mujer en edad fértil, desde la menarquia hasta la menopausia, aunque algunas veces, la endometriosis puede durar hasta después de la menopausia. La endometriosis altera la calidad de vida de las mujeres que la padecen, afectando a sus relaciones de pareja, familiares, laborales y de reproducción.

Síntomas

Los síntomas clásicos son la dismenorrea, dolor pélvico, dispareunia, sangrados

intermestruales y en muchos casos, esterilidad.

El dolor no tiene que ver con el tamaño y la severidad de la lesión; generalmente cuanto

menor es la lesión mayor dolor produce. El dolor se agrava con las menstruaciones y en los

casos en que la lesión ocupa el fondo de saco de Douglas, puede dar dispareunia. Existe un

aumento de la PGF2 alfa y PGE2 y un aumento de las contracciones uterinas que podría

deberse a un depósito de endometrio en la cavidad peritoneal.

La esterilidad debido a la endometriosis podría deberse a distintas causas de acuerdo a la

severidad de la patología. En los casos de endometriosis severa puede haber un factor

tuboperitoneal con adherencias y alteración en la anatomía de la pelvis que interfiera con el

transporte del esperma y el óvulo. En los casos de endometriosis leve hay varios mecanismos

propuestos que justifican su relación con la infertilidad: foliculogénesis alterada, fase lútea

inadecuada, fagocitosis espermática, mala calidad ovocitaria, embriotoxicidad y alteración a

nivel de la implantación.. La producción de prostaglandinas por el endometrio ectópico puede

afectar la motilidad tubaria, la foliculogénesis y la función del cuerpo lúteo. Puede haber un

aumento de la activación de los macrófagos peritoneales en la endometriosis que cause la

fagocitosis de los espermas o la secreción de citoquinas que pueden ser tóxicas para el

embrión. Según algunos investigadores habría un 60% de las mujeres con endometriosis que

presentan un síndrome de Folículo Luteinizado no roto (LUF) en el cual el folículo no se rompe

en la ovulación y el óvulo queda atrapado.

Referencias bibliográficas

1. Ruiz V. Endometriosis y fertilidad. Ed. Acosta y Warman, pp. 99

2. López ,VH. Palomo E. Incidencias de endometriosis en una población infértil. XXI Congreso nacional de Ginecología y Obtetricia. Guatemala, 1993.

3. El-Eoley, et al. Danazol but not ginadotropin releasing hormone agonists suppresses autoantibodies in endomeriosis. Fertil Steril 1990; 54:725

4. Acosta AA. Buttram VC Jr. Besch PK, Malinak LR, Van Der Heyden J. A.proposed classfication of pelvic endometriosis. Obstet Gynecol 1973;42:19.

5. Buttran VC Jr. Evolution of the revised American Fertility classification of endometriosis. Fert. Steril 1985; 43: 347

6. López VH. Tratamiento médico-quirúrgico de la endometriosis. Simposio El rostro cambiante de la endometriosis panamá 3. 12. 1993.

7. Steinleitner A. Heterolous transplation of activated murine peritonel macrophages inhibitis gamete interaction in vivo; A paradigm fo endometriosis associted subfertility. Fertil Steril 1990; 54:725.

8. Damewood M. Effect of serum from patients with minimal to mild endometriosis on mouse embryo growth. Fertil Steril 1990; 54: 917

9. Proug S. Peritoneal fluid fracctions from patients with endometriosis do not promote two-cell mouse embryo growth. Fertil Steril 1990; 54: 927.

49.- Femenino de 39 años con embarazo de 32 semanas. Ingresa al servicio de urgencias por presentar cuadro de 1 día de evolución por sangrado transvaginal leve, rojo brillante, sin contractilidad uterina. A.G.O.: G-3, P-0, C-2. E.F.: IMC: 30 kg/m2, F.U. de 28 cm, fcf presente, especuloscopía cérvix cerrado con huellas de sangrado. USG reporta producto único vivo con fetometría normal, líquido amniótico normal y placenta anterior que cubre parcialmente el orificio cervical interno. El siguiente paso en la atención de esta paciente es:

a) manejo por consulta externa y vigilar sangrado b) manejo por consulta externa con uteroinhibidores c) ingreso a hospital e inductores de maduración pulmonar d) ingreso a hospital y cesárea.

50.- Femenino de 45 años es atendida en consulta externa por presentar desde hace 6 meses flujo transvaginal, mucosanguinolento y sinusorragia importante, refiere dispareunia de 3 meses de evolución. AGO. G-5 P-5. Oclusión tubárica bilateral hace 6 años. E.F.: TA 130/80 MMHG, FC 80 LPM, TEMP. 36.7 ºC. Laboratorio: HB 9.7 G/DL, HTO 37 %, Se reporta Papanicolaou clase V. El siguiente paso para confirmar el diagnóstico es:

a) histeroscopía. b) ecosonografía. c) colposcopía. d) biopsia dirigida.

El examen citológico de papanicolau, realizado como examen de tamizaje en ginecología, permite establecer las características de las células que se descaman del cuello uterino. La técnica de papanicolau ha permitido disminuir la mortalidad e incidencia de cáncer invasor de cuello uterino. Se define examen de papanicolau anormal cuando se presentan alteraciones en el núcleo, citoplasma y/o la relación núcleo/citoplasma de las células examinadas. El

papanicolau anormal, de acuerdo a la clasificación de bethesda abarca desde un diagnóstico citológico de ascus (siglas en inglés de células atípicas de significado no determinado), lesión escamosa intraepitelial (lei) de bajo grado (corresponde a displasia leve y cambios por papiloma virus), lesión escamosa intraepitelial (lei) de alto grado (displasia moderada, displasia severa o carcinoma in situ) y células de cáncer invasor.

4.6. Colposcopía Ideada por Hinselmann en Alemania en 1924, llegó a América por el cono sur. Consiste en la visualización y amplificación del cuello uterino mediante un sistema binocular de lentes, entre 25 y 40 aumentos, lo cual permite la observación de las estructuras del cuello uterino mediante la asociación con imágenes preestablecidas. La colposcopía tiene una mayor sensibilidad que la citología, pero su menor especificidad. de conducir a procedimientos diagnósticos invasivos (biopsias y conizaciones) innecesarios y su mayor costo, son sus principales limitaciones. Combinadas la citología y la colposcopia brindan una seguridad diagnóstica que excede EL 95%. 4.6.1. Indicaciones de la colposcopia La colposcopía está indicada en las siguientes circunstancias (20), (21): - pacientes con citología cervical ( papanicolaou) clase III, IV o V, o sus equivalentes en los otros sistemas de clasificación. - pacientes con citologìa clase II con atipia inflamatoria, escamosa o endocervical, o cuando se informe la presencia de coilocitos. - pacientes con cérvix macroscópicamente normal, pero quienes presentan sinusorragia. - pacientes con cérvix macroscópicamente anormal, en ausencia de carcinoma evidente. - pacientes con citología clase II persistente, pese a tratamiento de posibles causas. (ejemplo: trichomonas). Tamizaje en cáncer ginecológico autores de la guía. Dr. Miguel Bueno Montaño Profesor asociado departamento de ginecología universidad libre ginecólogo centro médico Imbanaco profesor titular de ginecología y obstetricia universidad del valle Dr. Jaime Rubiano universidad del valle Dra. Derry Trujillo 51.- Femenino de 20 años de edad refiere irregularidades menstruales tipo hipo-opso-oligomenorrea desde hace 3 años. Niega tener vida sexual activa y no recuerda su fecha de última menstruación. No hay antecedente de galactorrea ni de uso de hormonales exógenos. Mide 164cm y pesa 60kg. Sin datos de hirsutismo, las mamas, útero y anexos son normales. El ultrasonido pélvico es normal, así como el perfil hormonal. El diagnóstico clínico más probable es:

a) Hiperplasia del endometrio b) Sangrado uterino disfuncional c) Alteración menstrual fisiológica d) Endometriosis

En la práctica médica se utiliza un grupo de términos para hacer referencia a las diversas alteraciones del ciclo menstrual, que requieren precisión por la frecuencia en que son diagnosticados.

Según Schiavon (2000), las alteraciones menstruales más frecuentes son: • oligoamenorrea: episodios de sangrado infrecuentes, irregulares, con intervalo de más de cuarenta días; • polimenorrea: episodios frecuentes pero regulares de sangrado uterino, que ocurren a intervalos menores de veintiún días; • menorragia: sangrado excesivo, tanto en cantidad como en duración, que ocurre con

regularidad y es sinónimo de hipermenorrea; • metrorragia: sangrado generalmente no excesivo, que ocurre a intervalos irregulares; • menometrorragia: sangramiento generalmente excesivo y prolongado, que ocurre a intervalos frecuentes e irregulares; • hipomenorrea: sangrado uterino regular, pero disminuido en cantidad; • sangrado intermenstrual: sangrado uterino generalmente no excesivo, que ocurre entre períodos menstruales regulares.

Las irregularidades menstruales son causa frecuente de consulta en las adolescentes, siendo 95 % de las veces de naturaleza disfuncional, por inmadurez del eje hipotálamo–hipofiso–ovárico (HHO).

Uno de los primeros problemas que hay que plantear ante estas irregularidades menstruales, es la hemorragia uterina disfuncional (HUD). Su definición guarda relación con las características en cantidad y frecuencia que difieren del sangrado menstrual normal.

Con mayor frecuencia se encuentra en forma de sangrados excesivos y prolongados, asociados a ciclos anaovulatorios, en ausencia de una patología o enfermedad existente, aunque raramente la HUD puede presentarse con ciclos ovulatorios.

De forma práctica, consideramos una hemorragia uterina (HU) como anormal cuando el sangrado es excesivo, con cualquier desvío o alteración de su duración, cantidad o intervalo.

El diagnóstico de HUD supone una alteración de origen endocrino (eje HHO); por lo tanto, su diagnóstico impone haber descartado cualquier patología orgánica y sistémica que produzca hemorragia genital. Es un diagnóstico por exclusión.

ETIOPATOGENIA DE LA HUD EN LA ADOLESCENCIA

Los ciclos anovulatorios son más frecuentes en las adolescentes por la inmadurez del eje HHO en el primer año tras la menarquia.

En esos casos de HUD, como ya se mencionó, se producen ciclos anovulatorios que se traducen en una proliferación desorganizada del endometrio por falta de efecto progestagénico. Una vez que el endometrio alcanza un grosor crítico, comienza a descamarse en forma irregular, traduciéndose en un sangrado permanente de cuantía variable.

Las manifestaciones clínicas de la HUD son: • fases de amenorrea de dos a cuatro meses, seguidas de salida de sangre abundante durante tres o cuatro semanas; en oportunidades existe irregularidad completa en el sangrado; • sangrado de más de seis compresas (bien empapadas) al día; • presencia de coágulos; • suele ser indolora; • menstruaciones de más de siete días de duración; • ciclos de menos de veintiún días.

La gravedad de esta hemorragia se clasifica, de acuerdo con el grado de anemia que produzca, en metrorragia leve, moderada o grave: Leve: • metrorragia leve y prolongada, • ciclo menstrual acortado, • hemoglobina y hematocrito normales. Moderada: • metrorragia copiosa prolongada, • ciclo menstrual acortado, • anemia leve (cifras de hemoglobina inferior a diez gramos por litro). Grave: • metrorragia copiosa prolongada,

• ciclo acortado e irregular, • anemia grave (cifras de hemoglobina de ocho gramos por litro o menos).

Ante un sangramiento uterino en estas edades se debe realizar el diagnóstico diferencial con: a) gestación y problemas relacionados con ésta, tales como abortos y gravidez ectópica; b) coagulopatía: 20 % de las adolescentes con hemorragia uterina tienen un defecto de la coagulación. La manifestación más precoz de alteraciones de la coagulación sanguínea puede ser evidenciada por un sangramiento genital anormal, lo cual puede estar relacionado con deficiencias de plaquetas, leucemias, púrpuras, enfermedad de Von Willebrand, deficiencia de protrombina u otros factores de la coagulación; c) malformaciones del aparato genital, traumatismosgenitales, presencia de cuerpos extraños; d) dispositivos intrauterinos; e) tumores uterinos, sarcoma botroides o tumores anexiales; f) hipo o hipertiroidismo; g) insuficiencia renal o hepática.

Como el diagnóstico de HUD es de exclusión, hay que hacer una historia clínica minuciosa, exámenes complementarios y sin falta descartar las otras causas de sangramiento transvaginal.

Se debe precisar con detalle el nivel de desarrollo puberal, la actividad sexual y la presencia de situaciones concomitantes como: a) contacto sexual sin protección contraceptiva; b) uso irregular de anticonceptivos orales o antecedentes de inserción de dispositivos intrauterinos; c) ejercicios físicos extenuantes; d) historia previa de sangrado excesivo, asociado a extracciones dentarias, pequeñas heridas, epistaxis y otras; e) dolencias renales u hepáticas preexistentes.

Ante cualquier demanda de atención por adolescentes con sangramiento genital con las características descritas, se requiere de un examen físico general que incluya exploración general completa, toma de tensión arterial y pulso, búsqueda de exoftalmia, fascie Cushing, visceromegalias o presencia de masas abdominales palpables, edemas parpebrales y de miembros superiores, así como puntos hemorrágicos en epidermis y otros signos de coagulopatía.

El examen ginecológico debe realizarse en todas las adolescentes, con excepción de las que no han tenido actividad sexual y presentan sangramiento leve. Además del examen de sus genitales, hay que efectuar una valoración citológica y microbiológica en particular en quienes presenten manifestaciones clínicas.

En la inspección de los genitales durante el examen de la paciente, es importante evaluar que el sangramiento se origine en lesiones ubicadas en los genitales externos, uretra u hemorroides, así como indagar acerca de la posibilidad de abuso sexual. Si después del tratamiento de la HUD leve continúa el sangrado, se recomienda la realización de ecografía abdominal para precisar el diagnóstico.

En pacientes que ya han tenido relaciones sexuales se debe buscar si el útero tiene características gravídicas, la posibilidad de un aborto en curso, así como la presencia de una masa anexial que permita corroborar la existencia de embarazo o alguna neoplasia benigna o maligna. El examen con espéculo podrá demostrar un cuello hiperémico, sangrante o gravídico; de encontrarse estos hallazgos, se descartaría la etiología disfuncional del sangrado.

Exámenes complementarios básicos que no pueden faltar: • hemograma completo, • coagulograma completo, • orina, • ultrasonido ginecológico abdominal, transvaginal o transrectal según proceda, • ecografía abdominal. Si fuese necesario por los signos identificados en el examen de la

paciente, se deben realizar: • dosificación de FSH, LH, T3, T4, TSH y prolactina si hay sospechas clínicas de otras enfermedades endocrinas concomitantes, • laparoscopia en casos seleccionados por patología de base, • otros, según hallazgos de la historia clínica y la exploración.

52.- Femenino de 55 años, se queja de presión pélvica y una masa en la entrada vaginal. Antecedentes: G.3 P.3, el último con peso al nacer de 4,500 grs. FUR hace tres años. Sin terapia de reemplazo. Historia de tabaquismo positivo a razón de 40 cajetillas año. Actualmente refiere dificultad para evacuar, tos crónica, presenta una orina de 60 cc. En la exploración pélvica, el hallazgo más probable es: a) Rectocele b) Cistocele c) Enterocele d) Uretrocele El Rectocele es una hernia de la pared anterior del Recto hacia la porción posterior de la vagina. La incidencia real de esta “deficiencia anatómica” es desconocida y en muchas ocasiones es un resultado del paso del tiempo. Es un hallazgo muy frecuente del examen perineal, siendo en múltiples ocasiones asintomático. El rectocele puede ser un hallazgo importante del síndrome de Obstrucción Defecatoria (SOD). No debe tomarse como una deficiencia anatómica única, sino como parte importante de un problema anatomo-fisiológico complejo. Un principio importante es la etiología y anatomo-patología del rectocele. Existen varias teorías sin consenso principal. La existencia, deficiencia o alteraciones del septo rectovaginal son controversias importantes. No existe una fascia visceral que separe el recto de la vagina o que forme un septo específico. Existe frontera entre donde termina la pared anterior del recto y donde comienza la pared de la vagina, pero múltiples estudios no han encontrado un septo rectovaginal específico. El septo puede estar formado de una película casi transparente hasta una pared de consistencia fibromuscular fuerte. El rectocele es un hallazgo común. Se presenta en el 80% de las pacientes femeninas y 13% de los masculinos en una defeco grafía (> a 1cm.) Entre más grande es el rectocele, mayor son los síntomas asociados, Dificultad en la evacuación, constipación crónica, dolor rectal y perineal, sensación de masa y en ocasiones sangrado. La necesidad de presión manual para ayudar a la evacuación o para vaciar el rectocele es común en más del 50% de los pacientes. El diagnostico se hace con un simple tacto rectal e inspección vaginal, pero debe recordarse que rara vez es el rectocele un hallazgo aislado. La presencia cistocele, peritoneocele, enterocele u otros prolapsos perineales y problemas funcionales deben ser descartados antes de proponer la reparación quirúrgica del rectocele. Defeco grafía, Pruebas de función fisiológica del piso pélvico, pruebas de función urinaria, evaluación del esfínter anorectal por ultrasonido y hasta la resonancia magnética han sido propuestos antes de la cirugía. Bibliografía: Rectocele: Pathogenesis and surgical managment. Zbar AP, Linemann A, Fritsch H, Beer-Gabel M, Pescatori M. Int J Colorectal Dis. (2003) 18:369-384. Evaluation and Treatment of Women with rectocele. Cundiff GW, Fenner D, Obstetrics and Ginecology 104(6): 1403-1416 Stapled transanal rectal resection to treat obstructed defecation caused by rectal intussusseption and rectocele. Renzi A, Izzo D, Di Sanrno (2—6) 21:661-667 Rectocele repair using biomaterial augmentation. Altman D, Melgren A, Zetterstrom J.

Obstet Gynecol (2005) 60(11)753-760. 53.- Femenino de 22 años de edad, acude al servicio de urgencias por referir malestar general, fosfenos, náusea y vómito. Antecedentes: cursa con embarazo de 36.5 SDG, G2, A1, C0. Exploración física: TA 185/110 mmHg, FC 120 lpm, FR 35 x’, T37.9°, alerta, inquieta, aprecia ictericia, cardiopulmonar sin compromiso, abdomen globoso a expensas de útero grávido, se detecta PUVI, longitudinal, cefálico, dorso a la derecha, FCF 142 x’, al tacto genital cérvix, central, formado, cerrado, extracción de guantes sin evidencia de pérdidas genitales, extremidades inferiores edema +++. La medida terapéutica inicial en esta paciente es: a) Diuréticos. b) Sedantes. c) Antihipertensivos. d) Analgésicos. Referencia: La preeclampsia se define por aumento de la presión arterial y la presencia de proteinuria durante el embarazo.

1) Leve: las pacientes suelen tener unas cuantas manifestaciones, y su presión arterial diastólica es menor de 110 mmHg. En ocasiones hay edema. La cifra de plaquetas es mayor de 100 000/µl.

2) Grave: los síntomas son más notorios y persistentes. La presión arterial casi siempre es con niveles mayores de 160/110 mmHg. Puede haber trombocitopenia que avance hasta coagulación intravascular diseminada.

Se requiere hospitalización para las mujeres con preeclampsia; debe obtenerse una biometría hemática completa con cifra de plaquetas y determinación de electrolitos, que además incluya enzimas hepáticas. Se obtiene una muestra de orina de 24 horas para determinar la depuración de creatinina y proteínas totales al ingreso hospitalario. Se debe controlar la hipertensión arterial, para evitar sufrimiento fetal, así como empeoramiento de la paciente. Bibliografía: 1. McPhee S, Papadakis M, et. al. Diagnóstico Clínico y Tratamiento 2010. Lange, McGraw

Hill, 49ª edición, México, 2010.

2. Sibai BM, Diagnosis, prevention, and management of eclampsia. Obstet Gynecol. 2005. Feb; 105;: 402 – 410.

54.- Femenino de 42 años portadora de DM, acude a consulta quejándose de descarga vaginal prurítica y blanquecina. De los siguientes exámenes de diagnósticos el más útil para identificar el patógeno es: a) Wet prep. b) Tincion de Gram c) PH d) KOH

Prueba del KOH es un procedimiento en el cual hidróxido del potasio (KOH) se utiliza detectar hongos disolviendo las células humanas en una cultura. La diferencia en la composición de la pared de célula de células humanas y de células fungicidas permite que este procedimiento ayude a distinguir las dos células. El KOH desnaturaliza las proteínas en la célula humana; solamente sigue habiendo las células fungicidas ser considerado debajo del microscopio.

1. Bernal B. Fisiología y ecología de la vagina. Rev Chil Obstet Ginecol 1986; 51:56-60.

2. Ibárcena E. Vaginosis bacteriana; diagnóstico y prevalencia. XII Congreso Peruano de Obstetricia y Ginecología 1996; 204-6.

3. Scapini JC, Guzmán CA. Detección de bacilos Gram negativos curvos anaerobios en pacientes con vaginosis. Obstet Ginecol Latinoam 1986; 44: 320-5.

4. Soihet S. El flujo vaginal en la consulta ginecológica. Ginecol Obstet 55.- Mujer de 26 años, es atendida en consulta en la clínica de displasias por papanicolau con lesión NIC I. Antecedentes: menarca 14 años, ritmo 30x5 eumenorreica, inicio de vida sexual a los 15 años, 2 parejas sexuales, método de planificación familiar oclusión tubaria bilateral, gestas 3 partos 3, cérvix con lesión acetoblanca con extensión lineal de 2 cm. Se realiza biopsia de la lesión, en caso de corroborarse el diagnóstico, el siguiente paso en el manejo de esta paciente es realizar:

a) biopsia. b) electrocirugía. c) cepillado de canal. d) ultrasonido endovaginal.

9.5.2 Las pacientes a quienes se les realizó citología cervical, cuyo resultado es LEIBG (infección por VPH, displasia leve o NIC 1); LEIAG (displasia moderada y grave o NIC 2 y 3) o cáncer deben enviarse a una clínica de colposcopía, para realizar estudio colposcópico. 9.5.3 Si el resultado de la citología es LEIBG, la colposcopía es satisfactoria y sin evidencia de LEIBG, se realizará control citológico en un año (Apéndice Normativo A) 9.5.4 Si la citología es de LEIBG, la colposcopía es satisfactoria y existe evidencia de lesión, se debe tomar una biopsia dirigida. 9.5.4.1 Si la biopsia dirigida es negativa, se realizará nueva colposcopía para verificar el diagnóstico y en caso necesario, tomar nueva biopsia dirigida y revalorar. 9.5.4.2 Si la biopsia dirigida es reportada como LEIBG se podrá dar tratamiento conservador: criocirugía, electrocirugía o laserterapia (sólo si cumple con las condiciones referidas en el Apéndice 1) o se podrá mantener a la paciente en vigilancia en la clínica de colposcopía, con colposcopía y estudio citológico cada seis meses, durante 24 meses. Jueves 31 de mayo de 2007 DIARIO OFICIAL (Primera Sección) 9.5.4.3 Si la biopsia dirigida es reportada como LEIAG (Lesión Intraepitelial Escamosa de Alto Grado) se realizará tratamiento conservador (electrocirugía o laserterapia). En las mujeres posmenopáusicas, dependiendo de las condiciones anatómicas del cérvix, se realizará tratamiento conservador en la clínica de colposcopía o tratamiento quirúrgico (histerectomía extrafascial) en el servicio que corresponda. 9.5.4.4 Si la biopsia dirigida reporta cáncer microinvasor o invasor, la paciente se transferirá a un Servicio o Centro Oncológico para su tratamiento correspondiente. 9.5.4.5 Si la citología reporta LEIBG y la colposcopía es no satisfactoria, se tomará cepillado endocervical (Apéndice Normativo A) 9.6 En caso de colposcopía no satisfactoria, negativa a LEIBG y con cepillado endocervical negativo, se continuará su control en la clínica de colposcopía en seis meses, con colposcopía y citología. 9.6.1.1 Si el cepillado endocervical reporta LEIBG se tratará a la paciente como LEIAG, con métodos conservadores escisionales.

Jueves 31 de mayo de 2007 DIARIO OFICIAL (Primera Sección) Modificación a la Norma Oficial Mexicana NOM-014-SSA2-1994, Para la prevención, detección, diagnóstico, tratamiento, control y vigilancia epidemiológica del cáncer cérvico uterino. Al margen un sello con el Escudo Nacional, que dice: Estados Unidos Mexicanos.- Secretaría de Salud. MODIFICACION A LA NORMA OFICIAL MEXICANA NOM-014-SSA2-1994, PARA LA PREVENCION, DETECCION, DIAGNOSTICO, TRATAMIENTO, CONTROL Y VIGILANCIA EPIDEMIOLOGICA DEL CANCER CERVICO UTERINO. 56.- A 38-year-old man comes to the physician because of slowly progressive visual problems that make him “bump into objects” on both sides. He also reports that, while driving, he has trouble switching lanes because he needs to turn his head all the way backward to look for other cars. Ocular examination shows bitemporal field loss with preserved visual acuity.Examination of the fundus is unremarkable. Which of the following is the most likely diagnosis? a) Pituitary adenoma b) Occipital lobe meningioma c) Optic neuritis d) Retinal detachment

. PRESENTACIÓN CLÍNICA

Los tumores hipofisarios se reconocen clínicamente por uno o más de tres patrones de presentación muy constantes:

- Síntomas de hipersecreción hipofisiaria

- Síntomas de hiposecreción hipofisaria

- Síntomas neurológicos

El tercer patrón de presentación es el dominado por los síntomas neurológicos, aislados o coexistentes con una o varias de las alteraciones endocrinológicas antes descritas. Como se ha dicho, una masa hipofisaria progresivamente creciente generará una constelación de signos y síntomas neurológicos que dependerán de la trayectoria del crecimiento y de las estructuras nerviosas vecinas que resulten alteradas. Los síntomas y signos a encontrar son:

a. Cefaleas: Pueden ser un signo precoz y se atribuyen al estiramiento de la duramadre que recubre la hipófisis o del diafragma de la silla turca. Está presente en el 75% de los casos y se localiza más frecuentemente en la región frontal y orbital.

b. Pérdida de visión: Debido a la compresión de los nervios ópticos o del quiasma óptico. El patrón clásico de pérdida visual es una hemianopsia bitemporal a menudo asociada con disminución de la agudeza visual. Posteriormente puede haber una ceguera completa de uno o ambos ojos.

c. Manifestaciones hipotalámicas: Debido a la compresión del hipotálamo por grandes adenomas hipofisarios que provocan:

- Alteraciones del sueño

- Alteraciones de la atención

- Alteraciones de la conducta

- Alteración de la alimentación

- Alteración de las emociones

d. Hidrocefalia obstructiva: Debido a la infiltración de las láminas terminales que hace que el tumor penetre en la región del III ventrículo, ocasionando obstrucción a la salida de LCR.

e. Compromiso de nervios craneales. Debido a la extensión lateral del tumor hacia la región del seno cavernoso, por donde transcurren los nervios craneanos. Así las manifestaciones más características son:

- Ptosis: Por compromiso del III nervio craneal (motor ocular común).

- Dolor o alteraciones sensitivas faciales. Por compromisos de las ramas oftálmica y maxilar superior del Nervio Trigémino.

- Diplopia: Por compromiso del III, IV y VI nervios craneales.

Bibliografía

1. Kovacs K, Horvath E, Vidal S: Classification of pituitary adenomas. J Neurooncol 54 (2): 121-7, 2001.

2. Ironside JW: Best Practice No 172: pituitary gland pathology. J Clin Pathol 56 (8): 561-8, 2003.

3. Scheithauer BW, Kovacs KT, Laws ER Jr, et al.: Pathology of invasive pituitary tumors with special reference to functional classification. J Neurosurg 65 (6): 733-44, 1986.

4. Ezzat S, Asa SL, Couldwell WT, et al.: The prevalence of pituitary adenomas: a systematic review. Cancer 101 (3): 613-9, 2004.

57.- An 18-year-old woman complains of myalgias, a sore throat, and painful mouth sores for 3 days´duration. Her temperature is 38.2 C (100.8 F), blood pressure is 110/80 mm Hg, pulse is 84/min, respirations are 15/min. Her gingival are edematous and erythematous, and there are vesicles on her right upper and lower lips. Her pharynx is mildly erythematous but without exudates, and there is tender mobile cervical lymphadenopathy. Her breath is not fetid, and the dentition is normal. Which of the following is the most likely causal agent? a) Actinomyces israelii b) Herpes simplex virus 1 c) Nocardia asteroids d) Streptococcus pyogenes

Las infecciones por herpes simplex son comunes en la práctica diaria, y con frecuencia el paciente acude a los servicios de urgencia. Estas infecciones son ocasionalmente recurrentes, generalmente dolorosas y asociadas con síntomas sistémicos, por lo cual el médico de urgencias debe estar familiarizado con el cuadro clínico y su manejo.

Existen dos variedades de virus del Herpes simplex (VHS) capaces de causar infección en el hombre: el tipo 1 (VHS-1) y el tipo 2 (VHS-2) que se distinguen entre sí por varias características, incluyendo sus comportamientos clínico y epidemiológico, antigenicidad, composición del ADN y la sensibilidad a diferentes agentes físicos y químicos (Cuadro No.1)

Cuadro No. 1

DIFERENCIAS ENTRE LOS VIRUS HERPES SIMPLEX TIPOS 1 Y 2

Características clínicas VHS-1 VHS-2

Vía de transmisión Síndromes característicos Reactividad a antígenos monoclonales específicos

Oral Oral-facial Ocular Encefalitis Paroniquia VHS-1 específico

Genital Genita PerianaL Neonata Paroniquia l VHS-2 específico

El VHS-1 es de localización primordialmente extragenital, con predilección por los tejidos de origen ectodérmico, mientras que el VHS-2 corresponde al "Herpes progenitalis" descrito por separado, dentro de las infecciones de transmisión sexual.

La región oral es la localización habitual del Herpes simplex 1, el cual es causa frecuente de lesiones orofaciales recurrentes y de otro tipo de enfermedades (encefalitis).

EPIDEMIOLOGIA

El ser humano es el único reservorio natural conocido del virus herpes simplex, aunque algunos animales de experimentación pueden infectarse con facilidad.

La infección primaria del VHS-1 ocurre sobre todo durante la infancia, mientras que el tipo 2 se presenta en la adolescencia y adultos jóvenes activos sexualmente. Las tasas de infección son inversamente proporcionales al estrato socioeconómico.

El principal mecanismo de transmisión es el contacto directo con las secreciones infectadas. El VHS-1 se trasmite por saliva y el VHS-2 por vía genital. Aunque los títulos virales son más altos cuando existen lesiones activas, también es frecuente la liberación viral en infectados asintomáticos. Por lo tanto, la transmisión viral puede efectuarse aún en ausencia de lesiones activas.

La persistencia de la infección y la recurrencia de las lesiones son un fenómeno frecuente tanto para el VHS-1 como para el VHS-2 y por lo común se producen por reactivación endógena. Los factores que la precipitan van desde la luz solar, el viento, traumatismos locales, fiebre, menstruaciones y hasta estrés emocional.

DIAGNOSTICO

Cuadro Clínico. Los cuadros clínicos causados por este virus se suelen dividir en dos grupos: el debido a la infección primaria y el correspondiente a la infección recurrente. En el primer grupo se incluyen la gingivoestomatitis aguda, la vulvovaginitis aguda y la infección herpética del ojo, que puede llegar a queratitis. Las recurrentes se circunscriben al "Herpes labialis", queratitis, blefaritis y queratoconjuntivitis. Todos los cuadros son autolimitados, pero tanto las formas primarias como las recurrentes, se pueden complicar. Una de estas complicaciones es la Encefalitis herpética y el Eczema herpeticum.

Infección primaria. El primer contacto clínico de infección por virus del herpes simple suele ser el más grave. Los enfermos aquejan fiebre, malestar general, artralgias y por último la presencia de un grupo de vesículas sobre una base eritematosa, dolorosa, inflamada y sensible. La gingivoestomatitis es la manifestación más común, cuya gravedad varía desde la erosión de pequeñas áreas a la ulceración extensa de la boca, lengua y encías. La infección puede ser bastante grave como para dificultar la ingesta de alimentos y líquidos (odinofagia). La curación tiene lugar en 7 a 14 días, a menos que las lesiones se sobreinfecten con estafilocos o estreptococos.

Infección recurrente. Generalmente existe prurito, dolor o molestias focales que preceden la aparición de las vesículas. Las vesículas se rompen espontáneamente después de unos cuantos días y sanan en una semana sin dejar secuelas.

LECTURAS RECOMENDADAS

1. Callen JP, Cooper Ma. Dermatologic emergences. Emerg Med. Clin North Am 3:641, 1985

2. Guzmán M. Herpes simple, varicela zoster. En: Medicina Interna. Segunda Edición. Editado por F Chalem, JE Escandón, J Campos, R Esguerra. Fundación Instituto de Reumatología e Inmunología. Editorial Presencia Ltda. Santafé de Bogotá, 1992

3. Guerra Flecha J, Lizarraga Bonelli S. Enfermedades de transmisión sexual: herpes genital Trib Med 79:29, 1989

Jaramillo AC. Infecciones virales de la piel y sus anexos. En: Fundamentos de Medicina. Enfermedades Infecciosas. Cuarta edición. Corporación para Investigaciones Biológicas. CIB. Medellín, 1989 58.- Femenino de 40 años de edad G.3 P.1 C 2, la cual es diagnosticada por miomatosis uterina de pequeños y medianos elementos sintomáticos, sus antecedentes refieren cirugías pélvicas previas, el tratamiento de elección es:

a) Histerectomía total abdominal. b) Progesterona. c) Observación d) Análogos de GnRH.

CUADRO CLINICO La miomatosis uterina muestra manifestaciones clínicas en menos del 50%, de estas las más frecuentes son: 1. Hemorragia uterina anormal. 2. Dolor.

3. Distensión abdominal. 4. Compresión genitouterina. 5. Compresión gastrointestinal. 6. Compresión pélvica. 10. DIAGNOSTICO El diagnóstico se realiza a través de imagenología: 1-Ecografía. 2-TAC 3-Rayos X 4-Histeroscopia. 11. TRATAMIENTO La miomatosis uterina debe ser tratada cuando produzca cualquiera de las manifestaciones clínicas anotadas, toda paciente que se programe para histerectomía debe tener previamente legrado biopsia Lo podemos dividir en Conservador o Radical. 1. CONSERVADOR: Este tratamiento se puede instaurar en pacientes con deseo de preservar el útero. Igualmente se puede subdividir: -Expectante -Quirúrgico: miomectomía -Medico Tratamiento expectante: Esta indicado en pacientes, cuyos síntomas son leves y no deseen o tengan alguna contraindicación médica para tratamiento quirúrgico. En ellas se recomienda controles clínicos y ecográficos cada 6 meses a 1 año. Tratamiento médico. AINES Análogos GnRH: Progestágenos: Andrógenos. Antiandrógenos Tratamiento quirúrgico: MIOMECTOMIA: 1. Criterios del ACOG en pacientes infecundas. Procedimientos:

Vía endoscópica: Laparoscopia: Miomas subserosos sesiles o pediculados < 5cm. Histeroscopia: miomas submucosos

Vía laparotomía

Aquellos miomas que se salgan de las características anteriores. Indicaciones: Hemorragia anormal. Perdida reproductiva. Infecundidad. Dolor.

El tratamiento previo con análogos está indicado cuando se desee disminuir el tamaño del mioma para prevenir sangrado quirúrgico. Contraindicaciones:

Embarazo. Cáncer endometrial. Infecciones. Dificultad técnica.

2. Criterios del ACOG para Miomectomía en pacientes que desean conservar el útero. Procedimiento: Vía endoscópica. Vía abdominal. Vía vaginal. Indicación:

a. Presencia de uno o dos Leiomiomas asintomáticos de tamaño tal que se pueden palpar por vía abdominal y constituyen una preocupación para la paciente.

b. Pacientes ovulatorias con miomas como posible causa de hemorragia uterina excesiva, demostrada por cualquiera de las siguientes circunstancias: Hemorragia profusa: de duración mayor de 8 días. Anemia por pérdida sanguínea aguda o crónica.

2. RADICAL: HISTERECTOMIA Para pacientes post menopáusicas, con paridad satisfecha o sin deseo de preservar el útero. Criterios del ACOG para Histerectomía por miomas. 1. Presencia de 1, 2, o 3 Miomas asintomáticos de tamaño tal que son palpables por vía

abdominal y preocupan a la paciente. 2. Hemorragia uterina excesiva.

Duración mayor de 8 días. Anemia por pérdida sanguínea aguda o crónica. 1. Molestias pélvicas producidas por los miomas: signos compresivos.

Contraindicaciones:

1. Deseo de conservar la fecundidad. 2. Miomas asintomáticos. 3. Contraindicación médica o dificultades técnicas para la cirugía. Guarnaccia M. and Rein M. Traditional Surgical Approaches to Uterine Fibroids Abdominal. Myomectomy and Hysterectomy. Clinical Obstetrics and Gynecology 2001. 44.2. 385-400 - Milad. M and Sankpal R. Laparoscopic Approaches to Uterine Leiomyomas. Clinical Obstetrics and Gynecology. 2001. 44-2. 401-411 - Carlson K. et al. Indications for Hysterectomy. N. Engl. J. Med. 1993. 328(12) 56- 59.- Se trata de femenino de 23 años que cursa con amenorrea secundaria la cual presenta menstruación posterior a la administración de progestágenos, lo más probable en su perfil hormonal es que presente:

a) Estrógenos bajos b) Estrógenos normales c) Progesterona elevada d) Gonadotropinas altas

La presencia de estrógenos se puede establecer de dos maneras: con la prueba de desafío con acetato de medroxiprogesterona (AMP) y el índice de maduración vaginal. Si hay estrógenos, luego de la administración de 10 mg de AMP diarios por 10 días debería presentarse un sangrado por vagina. Y de esta forma corroborar los niveles de estrógenos serian normales. La ausencia de esta metrorragia sugiere una obstrucción o un hipogonadismo. La otra alternativa es realizar el índice de madurez vaginal que es un procedimiento sencillo que puede obtenerse en el consultorio. Usando una espátula como las de Papanicolaou el ginecólogo extrae una muestra de la vagina y la extiende en un portaobjeto. Este portaobjeto se procesa y se estudia en el microscopio. Un extendido maduro tendrá células epiteliales superficiales grandes y numerosas con un núcleo rodeado por una gran cantidad de citoplasma. Por el contrario, una muestra no estrogénica tendrá un mayor número de células parabasales y basales con núcleo grande rodeado por escaso citoplasma.

Speroff L, Glass RH, Kase NG, eds. Clinical Gynecologic Endocrinology and Infertility, 5th ed. Baltimore: Williams & Wilkins; 1994:334-335.

60.- En el servicio de consulta externa recibe un frotis vaginal el cual reporta la presencia al microscopio de células “clave” las cuales son propias de infección por:

a) Cándida albicans b) Gardnerella c) Tricomonas d) Gonococos

La infección por gardnerella (bacteria) tiende a producir una secreción blanca, gris o de color amarillo turbio, con un olor fétido o a "pescado" que aumenta cuando la secreción se vuelve

alcalina, como sucede después del coito o de lavarse con jabón. Puede haber prurito o irritación vulvar, pero por lo general no son muy pronunciados. Al microscopio se aprecia las famosas células clave. Realizando el diagnóstico diferencial.

Células escamosas de capa intermedia alta, algunas de ellas con núcleos picnóticos, y dos de ellas con el citoplasma cubierto por formas cocáceas que borran sus bordes, dándole el aspecto de célula rebozada ó "célula clave". Gardnerella. .

Pernoll M. Enfermedades de trasmisión sexual. En: Manual de Obstetricia y Ginecología. Benson/Pernoll Editores.Editorial Interamericana S A. México DF, 1994

61.- Se trata de paciente femenino de 29 años de edad con tumor anexial de 6 cm líquido, dolor abdominal, fiebre, leucorrea, con historia de cervicovaginitis de repetición y dispareunia crónica, última menstruación hace una semana El manejo más adecuado es: a) Histeroscopía b) Colposcopía c) Laparoscopía d) Histerosalpingografía

CLASIFICACION LAPAROSCOPICA DE LA ENFERMEDAD PELVICA INFLAMATORIA

El stándar de oro para el diagnóstico de EPI es la laparoscopia, ya que además de visualizar directamente los órganos pélvicos, permite la toma de muestras para estudios bacteriológicos.

Los criterios laparoscópicos para el diagnóstico de EPI, se describen en la tabla No.3.

Tabla No. 3

CLASIFICACION LAPAROSCOPICA DE LA EPI

Leve Eritema, edema, las trompas se mueven libremente. No hay exudado purulento.

Moderada Eritema, edema mas marcado, material purulento evidente. No hay movimiento libre de las trompas. La fimbria puede no ser evidente.

Severa Presencia de piosálpinx y/o absceso

DiagnósticoDolor abdominal bajo y/o dolor a la movilización cervical y anexial y:

Fiebre de 38º C o mayor

leucocitosis > 10, 500 mm3, eritrosedimentación y PCR elevada

Bacterias o leucocitos en liquido peritoneal

Masa anexial palpable o por USG

Laparoscopia

Síndrome de Fitz‐ Hugh‐Curtis

EPI

62.- Femenino de 34 años que inicia tratamiento con sulfato de magnesio por presentar eclampsia, se presentan signos de sobre dosificación. El antídoto específico que se debe de utilizar es: a) Gluconato cálcico b) Nitroprusiato. c) Simpaticomiméticos. d) Carbonato sódico.

NIVEL DE PRIMER CONTACTO (ATENCION PRIMARIA)

Se debe instruir a todas las embarazadas que deben acudir inmediatamente a un centro de salud en cualquiera de los siguientes casos:

edema que se desarrolla rápidamente (en pocos días) cefalea severa y persistente dolor en la región abdominal superior visión borrosa

Se debe realizar la medición de la presión arterial y un análisis de orina para la detección de proteinuria a las mujeres que acudan a centros de salud presentando estos síntomas.

Convulsiones

Si se asiste a una mujer con eclampsia en un centro de atención primaria,

1. deben mantenerse las vías respiratorias permeables;

2. se debe colocar a la mujer de costado (posición decúbito lateral izquierda) para evitar la aspiración del vómito u otras secreciones;

3. si es posible, se debe establecer una vía intravenosa;

4. se debe administrar sulfato de magnesio.

Monitoreo de la administración de sulfato de magnesio: Durante el tratamiento con sulfato de magnesio, se recomienda realizar un control cada 4 horas, como mínimo, para detectar la presencia de: Reflejo rotuliano, frecuencia respiratoria superior a 16 por minuto, volumen de orina >100 ml en las 4 horas previas.

- Sobredosis de sulfato de magnesio: Todo centro de salud que utilice sulfato de magnesio debe disponer de ampollas de gluconato de calcio (1 g) como antídoto para la sobredosis de dicho fármaco.

Se sugiere medir la presión arterial y administrar antihipertensivos según corresponda. Convulsiones recurrentes: en caso de convulsiones recurrentes, se administran otros 2

a 4 g de sulfato de magnesio por vía IV en el lapso de 5 minutos, tanto para el régimen IM como el IV; la dosis se determina en función del peso de la paciente.

El sulfato de magnesio es un fármaco usado en el control de las convulsiones eclámpticas, para suprimir o controlar las contracciones uterinas sean estas espontáneas o inducidas, y como broncodilatador luego del uso de beta agonistas y agentes anticolinergicos. También tiene indicación como terapia de reemplazo en la deficiencia de magnesio, como laxante para reducir la absorción de tóxicos del tracto gastrointestinal. El sulfato de magnesio está ganando popularidad como tratamiento de inicio en el manejo de algunas arritmias, particularmente en Torsades de Pointes, y en arritmias secundarias a sobredosis de antidepresivos tricíclicos o toxicidad digitálica. Esta también considerado clase Ila (probable beneficio) para la fibrilación ventricular refractaria y la taquicardia ventricular, luego de la administración de dosis de lidocaina y bretilio.

FARMACODINAMIA

El sulfato de magnesio tiene la capacidad de alterar la excitabilidad de la fibra miometrial, afecta el acoplamiento excitación – contracción y el proceso mismo de contracción, inhibe la entrada de calcio al sarcoplasma y reduce la frecuencia de los potenciales de acción. Inhibe también la liberación de acetilcolina. Por ser estas acciones comunes en las fibras musculares se pueden ver afectadas también la musculatura voluntaria e incluso las fibras miocárdicas.(1)

Bibliografía:

Graves C. Fármacos que contraen o relajan el útero. En: Hardman J, Limbird L, Molinoff P, Ruddon R, Goodman A, eds. Goodman & Gilman. Las Bases Farmacológicas de la Terapéutica. 9 ed. México DF: McGraw-Hill Interamericana; 1996. pp. 1012-3. 63.- Femenino que cursa con 36. 5 semanas de gestación acude al servicio por referir malestar general, fosfenos, nausea y vómito, aprecia moderada ictericia, usted sospecha de un síndrome de HELLP. Las alteraciones de laboratorio que espera encontrar al confirmar el diagnóstico son:

a) Anemia Hemolítica, trombocitopenia, enzimas hepáticas elevadas. b) Trombocitosis, Enzimas hepática elevadas, Anemia hemolítica. c) Anemia hemolitica, Trombocitosis, Fosfatasa Alcalina elevada. d) Trombocitopenia, Leucopenia, Hipertensión Arterial.

DEFINICIÓN:

• Es una complicación de la preeclampsia en la cual además de la Hipertensión Arterial y proteinuria hay presencia de anemia hemolítica, enzimas hepáticas elevadas y recuento bajo de plaquetas

EPIDEMIOLOGIA:

• Se presenta en un 4 a 10% de las preeclámpticas, diagnosticándose anteparto en un 70% de los casos preferentemente antes de las 37 semanas, mientras que el 30% de los casos restantes enferma en los primeros 7 días del puerperio, sobre todo en las 48 h iniciales.

• La proteinuria e hipertensión pueden estar ausentes en un 15 al 20% de los casos. • Incidencia mayor en multigestantes y en edades avanzadas. • Ocurre más frecuentemente cuando se demora la salida del feto y cuando se presenta

desprendimiento de la placenta • Mortalidad materna del 24% y mortalidad perinatal del 30-40%.

CLASIFICACION: Síndrome de HELLP. Clasificación de Mississipi.

CLASE Plaquetopenia LDH AST-ALT 1 Severa <50000 >600 IU/L >70 IU/L 2 Moderada

>50000 >600 IU/L >70 IU/L

<100000 3 Ligera >100000

<150000 >600 IU/L >40 IU/L

<70 IU/L PE severa

Eclampsia (sin HELLP)

>150000 <400 IU/L <40IU/L

MANIFESTACIONES CLINICAS:

• Malestar general, fatiga y molestias inespecíficas 90% • Cefalea 70% • Epigastralgia 64% • Vómito 22% • Fosfenos 15% • Visión Borrosa 11% • Acùfenos 3% • Ictericia • Anemia no explicada • Oliguria

��Si se añade una HEMORRAGIA HEPÁTICA, el paciente puede quejarse de dolor en el HOMBRO DERECHO y EL CUELLO, además de las molestias abdominales.

• Equimosis en los sitios de punciones venosas, petequias en los sitios de presión del

brazo, pero pueden tener pruebas de Rumpel Leed negativas. • En casos severos se pude presentar ascitis como causa de hipertensión portal.

DIAGNÓSTICO: El diagnóstico clínico del síndrome de HELLP se plantea en gestantes o puérperas con preeclampsia severa-eclampsia, excepto en el 15-20%, en las cuales esta asociación no puede ser demostrada, en tanto se cumplan los criterios de Sibai: MANIFESTACIONES CLÍNICAS:

• Malestar general, fatiga y molestias inespecíficas 90% • Cefalea 70% • Epigastralgia 64% • Vómito 22% • Fosfenos 15%

• Visión Borrosa 11% • Acùfenos 3% • Ictericia • Anemia no explicada • Oliguria

• Si se añade una HEMORRAGIA HEPÁTICA, el paciente puede quejarse de dolor en el HOMBRO DERECHO y EL CUELLO, además de las molestias abdominales.

• Equimosis en los sitios de punciones venosas, petequias en los sitios de presión del

brazo, pero pueden tener pruebas de Rumpel Leed negativas. • En casos severos se pude presentar ascitis como causa de hipertensión portal.

DIAGNÓSTICO: El diagnóstico clínico del síndrome de HELLP se plantea en gestantes o puérperas con preeclampsia severa-eclampsia, excepto en el 15-20%, en las cuales esta asociación no puede ser demostrada, en tanto se cumplan los criterios de Sibai: HEMOLISIS • Frotis periférico anormal (eritrocitos fragmentados) • Hematocrito (>24%) • Bilirrubina indirecta (>1.2mg/dL) • Deshidrogenasa láctica (>218 UI/L)

ENZIMAS HEPÁTICAS ELEVADAS • LDH >218UI/L • AST >30UI/L • ALT >37UI/L

PLAQUETAS BAJAS <100.000/mm3

BIBLIOGRAFIA Sibai baha, El síndrome HELLP. Universidad de Valencia , revista quincenal de Obstetricia clínica y ginecología, Octubre 2003. V. Cararach, Síndrome de HELLP y Repercusiones maternas. X curso intensivo de formación continuada materno fetal. Enero de 2003.

Toirac, Abelardo. Síndrome de Weistein HELLP Hospital Ginecoobstetrico Tamara Bunke. Junio 2002 De la Fuente, David. Síndrome HELLP. Medicina Universitária 2003; 5 (19): 101 -9 Andrea G. Witlin, DO, Baha M. Sibai, MD. Diagnosis and Management of women with Hemolysis Elevate Liver Enzymes, and Pletelet Count (HELLP) syndrome. Hospital Physician. Febrero 1999. CIFUENTES B, Rodrigo. Ginecología y obstetricia basadas en las evidencias. Bogotá: Distribuna, 2006. Sexta edición. 447 - 283 p. 64.- Se trata de femenino de 28 años de edad, acude por presentar retraso menstrual de 7 semanas, asintomática, con antecedentes de enfermedad pélvica inflamatoria tratada anteriormente, sin evidencia mediante ecografía de útero ocupado. Al determinar la B-HCG, se obtiene cifra de 2,500UI/l. Ante estos datos, en primer lugar habrá que pensar en:

a) Gestación de evolución normal correspondiente a amenorrea. b) Embarazo ectópica asintomático. c) Aborto precoz completo con expulsión total de restos ovulares intrauterinos. d) Embarazo molar de inicio, sin signos ecográficos intrauterinos.

FACTORES DE RIESGO

Anomalías tubarias

Enfermedad pélvica inflamatoria 30-50% Adherencias (endometriosis) Antecedente de salpingoclasia Alteraciones anatómicas

EMBARAZO ECTÓPICO

Descenso

Cuantificación seriadade HGC

USG normal

< 6500mUI

Repetir USG Laparoscopia DxDilatación y legrado

Estabilización oincremento

Aumento

02/07/201364

Diagnóstico:

HGC > 1500 U/L Sin evidencia de saco Líquido libre en fondo de

saco Sensibilidad 63% Especificidad 100%

Ankum WM., Mol Bw.,Van der Veen F.,Bossuyt PM. Risk factors for ectopic pregnancy:a meta-analysis. Fertil Steril 1996:65:1093-9. 65.- Femenino de 26 años G-3, P-1, A-1 con 39 SDG por FUR. Reporta contracciones uterinas que han sido regulares las últimas tres horas. Al examen encuentras que las contracciones son cada tres minutos y duran 50 segundos y son firmes a la palpación. Tuvo ruptura de membranas hace una hora y lo demuestras con papel de nitrazina. El examen digital cervical demuestra una dilatación de 5 cm, con borramiento del 100% y presentación en vértex en estación 0. De los siguientes criterios el más preciso para decir que se encuentra en la fase activa del trabajo de parto es : a) Borramiento cervical más de 90% b) Duración de las contracciones de más de 30 seg c) Dilatación cervical mayor de tres centímetros d) Ruptura de membranas FASES DEL TRABAJO DE PARTO El trabajo de parto se divide en tres fases: Fase 1 ó latente Es llamado así al periodo que sirve para la preparación uterina del parto, ocurre al final del embarazo y va hasta el inicio de las contracciones del trabajo de parto. Los aspectos a destacar en este lapso es el reblandecimiento cervical, el aumento importante en el número de receptores para oxitocina a nivel de las células endometriales, un aumento sustancial en los puentes de unión y el número de conexinas a nivel miometrial y por consiguiente una mayor sensibilidad a los agentes uterotónicos. Fase 2 ó activa Es el lapso que representa el trabajo de parto activo, y se acepta que se inicie cuando existen 3 cm de dilatación y las contracciones uterinas son aptas para producir un avance en el trabajo de parto; se divide en tres periodos: Primer periodo. Se inicia cuando las contracciones uterinas alcanzan la frecuencia, intensidad y duración suficientes para causar borramiento y dilatación del cuello uterino, y finaliza cuando éste se encuentra en completa dilatación. El lapso de tiempo que dura es variable, pero se acepta como normal hasta diez horas en primigrávidas y ocho horas en multigrávidas; pero independientemente de esto, se debe considerar como adecuado si el borramiento y la dilatación cervical son progresivos e ininterrumpidos. Segundo periodo. Se inicia con una dilatación cervical completa y termina con la expulsión del feto; tiene una duración variable, pero se acepta como normal una hora en pacientes primíparas y 30 minutos en multíparas; y tiene como característica que debe de ser progresivo e ininterrumpido. Tercer periodo. Este comienza inmediatamente finalizada la expulsión fetal y termina con la expulsión total de la placenta y las membranas corioamnióticas; a este periodo se le conoce también como de “alumbramiento” y es el más corto de los periodos del parto; como norma general se acepta que no debe de extenderse más allá de 10 minutos. Existen algunos autores que incluyen un “cuarto periodo” dentro del trabajo de parto, el cual

abarca aproximadamente la hora posterior al alumbramiento, y comprende el lapso de tiempo cuando ocurre la contracción y retracción de las fibras miometriales, así como la trombosis de los vasos adyacentes, lo cual es un efectivo control de la hemorragia del sitio de implantación de la placenta. Fase 3 Este periodo es el que representa el regreso de la mujer a su estado previo al embarazo, y se caracteriza por la involución uterina, la eyección láctea y por último la restauración de la fertilidad; existen estudios que involucran en esta fase a la endotelina-1 y a la oxitocina como substancias responsables de estos cambios postparto. PROGRAMA DE ACTUALIZACION CONTINUA PARA GINECOLOGÍA Y OBSTETRICIA PAC GO-1 Libro 3 Obstetricia 2005 66.- Femenino de 19 años, gesta 1, tuvo un parto con un producto único masculino de 3,600 g. la calificación de Apgar en el RN fue de 9 al primer minuto y 9 a los 5 minutos. La revisión de sus registros de trabajo de parto mostró que tuvo ruptura de membranas 7 horas antes del parto. 40 horas después del parto la paciente presenta: temperatura 38.2 ° C, FC 105 x’, TA 110/70, FR 16x’; E.F. dolor leve a la palpación del útero, las mamas se encuentran sin eritema, ni dolor a la palpación, no dolor a la compresión de las pantorrillas. De los siguientes pasos el más indicado antes de iniciar terapia con antimicrobiana ES: a) Hemocultivo b) Cultivo de secreción vaginal c) Examen general de orina y cultivo d) Espirometría por incentivo . El vaciamiento incompleto ocasiona orina residual, distensión vesical excesiva y estasis, además del cateterismo intermitente con sonda vesical durante el trabajo de parto. Por lo tanto, la vejiga en el puerperio está predispuesta a infecciones. El dolor leve a la palpación del útero puede ser normal en el puerperio y no se debe suponer de inmediato endometritis puerperal. Cuando se sospecha endometritis, los cultivos de secreción vaginal tienen poca utilidad porque se encuentran los mimos microorganismos que en mujeres puérperas sanas. Los hemocultivos son apropiados para la valoración diagnóstica de la fiebre puerperal, pero no son el paso inicial. La espirometría por incentivo se utiliza en el posoperatorio de inmediato para fomentar la expansión pulmonar y disminuir las atelectasias. El legrado uterino se utiliza para tratar la hemorragia Morgan M, Siddighi S. Ginecología y obstetricia, National Medical Series. 5° edición. Mc Graw Hill. Pp. 29.

67.- Femenino de 22 años de edad, acude al servicio de urgencias por referir malestar general, fosfenos, náusea y vómito. Antecedentes: cursa con embarazo de 36.5 SDG, G2, A1, C0. Exploración física: TA 185/110 mmHg, FC 120 lpm, FR 35 x’, T37.9°, alerta, inquieta, aprecia ictericia, cardiopulmonar sin compromiso, abdomen globoso a expensas de útero grávido, se detecta PUVI, longitudinal, cefálico, dorso a la derecha, FCF 142 x’, al tacto genital cérvix, central, formado, cerrado, extracción de guantes sin evidencia de pérdidas genitales, extremidades inferiores edema +++. El dato de laboratorio que apoya el diagnóstico en esta paciente es: a) Hematuria. b) Coluria. c) Piuria. d) Proteinuria. Referencia: La preeclampsia se define por aumento de la presión arterial y la presencia de proteinuria durante el embarazo.

3) Leve: las pacientes suelen tener unas cuantas manifestaciones, y su presión arterial diastólica es menor de 110 mmHg. En ocasiones hay edema. La cifra de plaquetas es mayor de 100 000/µl.

4) Grave: los síntomas son más notorios y persistentes. La presión arterial casi siempre es con niveles mayores de 160/110 mmHg. Puede haber trombocitopenia que avance hasta coagulación intravascular diseminada.

Se requiere hospitalización para las mujeres con preeclampsia; debe obtenerse una biometría hemática completa con cifra de plaquetas y determinación de electrolitos, que además incluya enzimas hepáticas. Se obtiene una muestra de orina de 24 horas para determinar la depuración de creatinina y proteínas totales al ingreso hospitalario. Se debe controlar la hipertensión arterial, para evitar sufrimiento fetal, así como empeoramiento de la paciente. Bibliografía: 3. McPhee S, Papadakis M, et. al. Diagnóstico Clínico y Tratamiento 2010. Lange, McGraw

Hill, 49ª edición, México, 2010.

4. Sibai BM, Diagnosis, prevention, and management of eclampsia. Obstet Gynecol. 2005. Feb; 105;: 402 – 410.

68.- Femenino de 37 años gestante, acude a ultrasonido de control. Reportando: Placenta marginal, embarazo de 37 SDG. Antecedentes: G- 5 P- 4. La complicación esperada en esta paciente es: a) Hemorragia materna. b) Desprendimiento de placenta normoinserta. c) Parto pretérmino. d) Restricción en el crecimiento intrauterino.

Placenta previa: La placenta en lugar de estar implantada en el cuerpo del útero por fuera del orificio cervical interno, se localiza por encima del orificio interno o cerca de él. En la placenta marginal el borde la placenta está en el margen del orificio. La multiparidad y la edad avanzada parecen favorecer la placenta previa. El hecho más carácterístico de la placenta previa es la hemorragia no dolorosa, la cual no suele aparecer hasta casi del final de segundo trimestre o más tarde. Complicaciones maternas: Hemorragia y muerte. • Pritchard, Williams Obstetricia, 3ªEdición, Salvat editores, pag. 395-399

69.- Se trata de femenino de 31 años nuligesta pero con actividad sexual regular, sin método de planificación familiar, con ciclos regulares, sin leucorrea, refiere dispareunia profunda, sangrado intermestrual y dismenorrea secundaria ocacionalmente presenta urgencia urinaria. Establecido su diagnóstico el tratamiento de elección en esta patología es:

a) Doxicilclina b) Ceftriaxona c) Leuprolide d) Tibolona

EndometriosisTratamiento

Análogos de  GnRH por 6  meses (Leuprolide,  Nafarelina, Triptorelina, Goserelina.

• Disminuye la secreción de FSH y LH .

Danazol

ACO por 9 meses

Laparoscopia

Radical con HTA +SOB

¿Cáncer?

Células claras

Endometroide de ovario

70.- Recibe usted los resultados histopatológicos de una paciente de 24 años de edad que acudió a revisión rutinaria, los resultados reportan imagen histológica de coilocitos lo cual sugiere infección por:

a) Herpes virus tipo 2 b) Citomegalovirus c) Virus del papiloma humano d) Vaginosis bacteriana.

El coilocito es un tipo de célula hallada en lesiones precancerosas cervicales. También es común apreciarla microscópicamente en lesiones reaccionales en la mucosa oral, debido a su similitud con la mucosa vaginal, en enfermedades como Papilomas, o en Condiloma acuminado.

El coilocito es la manifestación clásica de la infección por VPH en la célula. Fue descrito por primera vez por Koss y Durfee en 1956. Esta célula también ha sido llamada “célula en balón”.

El coilocito es una célula epitelial escamosa, más comúnmente superficial e intermedia, aunque también puede verse en células parabasales y metaplásicas. Esta célula presenta cambios típicos tanto en su núcleo como en su citoplasma, pierde los bordes angulados usuales de la célula escamosa superficial y su forma tiende a ser redondeada y ovoide. El citoplasma muestra una condensación periférica que le da un aspecto en “asa de alambre”, es opaco, denso y de aspecto céreo, anfofílico, acidofílico o de color rojo/naranja brillante. Además se observa una gran cavidad o halo con un margen muy bien definido, de forma oval o ligeramente festoneado. El núcleo de la célula se localiza de manera excéntrica, lo que lo convierte en un halo paranuclear, no perinuclear. Ocasionalmente puede encontrarse material fagocitado dentro del espacio coilocítico.

Lesión Intraepitelial de Bajo Grado. Alteraciones Celulares compatibles con Infección (Coilocitos). Schlecht, N.F., Kulaga, S., Robitaille, J., Ferreira, S., Santos, M., Miyamura, R.A., Duarte-Franco, E., Rohan, T.E., Ferenczy, A., Villa, L.L., & Franco, E.L. (2002) Persistent Human Papillomavirus Infection as a Predictor of Cervical Intraepithelial Neoplasia. JAMA, 286, 3106-3114

71.- Femenino de 32 años segunda gesta a término sin anormalidades en el transcurso de éste. Inicia trabajo de parto de forma espontánea, con evolución normal hasta que se rompe la bolsa, con una dilatación de 4 cm. A partir de entonces, comienza con hemorragia de sangre roja, en moderada cantidad y aparecen signos de sufrimiento fetal agudo. El estado general de la mujer es bueno y la dinámica uterina es normal. Este cuadro corresponde a: a) Abruptio placentae. b) Rotura uterina. c) Placenta previa central d) Rotura de vasa previa.

La vasa previa es una condición de alto riesgo obstétrico en la cual vasos fetales o placentarios cruzan el segmento uterino por debajo de presentación. Estos vasos están desprotegidos de gelatina de Wharton o tejido placentario, lo que los hace altamente vulnerables y susceptibles de ruptura o laceraciones en cualquier período del embarazo, principalmente en el momento del parto . También es frecuente la compresión de estos vasos, especialmente durante el tercer trimestre de la gestación, lo que puede condicionar asfixia y muerte fetal. Esta condición ocurre como resultado de que vasos velamentosos cruzan por el segmento uterino debido a una inserción velamentosa del cordón, situación en la cual el cordón umbilical se inserta en las membranas ovulares en vez del tejido placentario (vasa previa tipo I), o por el cruce de vasos fetales entre uno o más lóbulos accesorios de la placenta (vasa previa tipo II) (Figura 1).

Figura 1. A, Vasa previa tipo I, debida a inserción velamentosa. B, Vasa previa tipo II, debida a cotiledones aberrrantes. Reproducido de Daly-Jones y cols. Ultrasound 2008. Vasa previa se puede presentar si existe alguna (o ninguna) de las siguientes condiciones: placenta baja (que puede ser causa de abortos previos seguidos por legrado o por operaciones uterinas, que provocan cicatrices en el útero), placenta bilobada o de lóbulo succensuriado, embarazos resultado de fertilización in vitro, o embarazos múltiples (5-6). El sangrado por vasa previa no es doloroso. Otros sangrados por complicaciones o por nacimiento no necesariamente son sin dolor. Referencias: 1. Oyalese Y, Smulian JC. Placenta previa, placenta acreta, and vasa previa. Obstet Gynecol 2006; 107: 927-941. 2. Oyalesse KO, Turner M, Less C, Campbell S. Vasa previa: an avoidable obstetric tragedy. Obstet Gynecol Surv 1999; 54: 138-145. 3. Sepúlveda W, Sebire NJ, Harris R, Nyberg DA. The placenta, umbilical cord, and membranas. In Diagnostic Imaging of Fetal Anomalies, Nyberg DA, MaGahan JP, Pretorius DH, Pilu G (eds). Philadelphia, PA: Lippicont Williams & Wilkins 2003, 85-132.

4. Daly-Jones E, John A, Leahy A, McKenna C, Sepulveda W. Vasa praevia; a preventable tragedy. Ultrasound 2006; 16: 8-14. 5. Derbala Y, Grochal F, Jeanty P. Vasa previa. J Prenat Med 2007; 1: 2-13. 6. Fung TY, Lau TK. Poor perinatal outcome associated with vasa previa. It is preventable? A report of three cases and review of the literature. Ultrasound Obstet Gynecol 1998; 12: 430-433. 7. Robert JA, Sepulveda W. Fetal exsanguination from ruptured vasa previa: still a catastrophic event in modern obstetrics. J Obstet Gynaecol 2003; 23: 574. 8. Cordero DR, Helfgott AW, Landy HJ, et al. A non-hemorrhagic manifestation of vasa previa: a clinicopathologic case report. Obstet Gynecol 1993; 82: 698-700. 9. Schachter M, Tovbin Y, Arieli S, et al. In vitro fertilization as a risk factor for vasa previa. Fertil Steril 2002; 78: 642-643. 72.- Se trata de femenino de 24 años de edad que acude a consulta, cursando su 14ª semana de gestación, refiere tenesmo vesical, disuria y escalofrío. De acuerdo a su estado actual el tratamiento de elección que recomienda a la paciente es:

a) Ciprofloxacino b) Tetraciclina c) Ampicilina d) Metronidazol El Metronidazol no ha mostrado efectos tóxicos en humanos, pero es teratogénico en modelo animal. Las tetraciclinas ocasionan coloración anormal de los dientes, hepatotoxicidad y alteración en el desarrollo de huesos. Las sulfas podrían tener un efecto deletéreo en el primer trimestre dada su actividad como antimetabolitos, y en los últimos meses pueden favorecer kernicterus en el recién nacido si es que tiene alteraciones metabólicas que favorezcan anemia hemolítica. Las quinolonas se han asociado a malformaciones óseas en modelos animales, y se recomienda evitarlas si existen mejores opciones. Kasper DL, Braunwald E, Fauci AS, Hauser SL, Longo DL, Jameson JL. Harrison´s Principles of Internal Medicine. McGraw Hill. 16 Ed. 789-806 pp. 73.- Se trata de femenino de 33 años con antecedentes patológicos de hipertensión arterial crónica bien controlada tratada con IECAS , actualmente cursa con 7 semanas de gestación , signos vitales dentro del parámetro normal y exámenes de laboratorio sin alteraciones, se refiere asintomática. La conducta más adecuada a seguir es:

a) b) Mantener el tratamiento y asociar alfametildopa para disminuir los riesgos fetales de los IECAs

b) Mantener el tratamiento y asociar hidralacina para disminuir los riesgos maternos de los IECAs.

c) Suspender los IECAs dado el riesgo que presentan para el feto.

d) Mantener el tratamiento dado el buen control tensional. El uso de IECA y ARAII durante el segundo y tercer trimestre de embarazo está contraindicado, debido a que estos medicamentos inducen toxicidad fetal (descenso de la

función renal, oligohidramnios, retraso en la osificación del cráneo) y toxicidad neonatal (insufi ciencia renal, hipotensión, hiperpotasemia). En cuanto a su uso durante el primer trimestre de embarazo, un estudio publicado en el año 20061 mostraba un incremento de la incidencia de malformaciones congénitas, en particular malformaciones cardiacas, en niños nacidos de madres expuestas a IECA durante el primer trimestre de embarazo en comparación con las mujeres que no recibieron tratamiento antihipertensivo o que recibieron tratamiento con otros medicamentos antihipertensivos. Estudios posteriores realizados no han confirmado a día de hoy los resultados de este estudio. En lo referente a los ARAII, no se dispone de estudios epidemiológicos analíticos apropiados, por lo que no se puede descartar que exista el mismo riesgo que para los IECA. A pesar de estas incertidumbres, el Comité de Medicamentos de Uso Humano (CHMP) de la Agencia Europea de Medicamentos (EMEA) ha recomendado prudencialmente evitar el uso de IECA y ARAII durante el primer trimestre del embarazo. Cooper WO et al. Major congenital malformations after fi rst-trimester exposure to ACE inhibitors. N Engl J Med 2006; 354 (23): 243- 51. (ref.: 2008/10, junio

74.- Se trata de femenino de 27 años, G1 en trabajo de parto prematuro con embarazo de 30 semanas de gestación. A pesar del uso de agentes tocolíticos, estos no han dado resultado. Se puede inducir la maduración pulmonar del producto por medio de:

a) Betametasona

b) Sulfato de magnesio

c) Hidroxiprogesterona

d) Clorprocaína

La utilización de betametasona como inductor de madurez pulmonar fetal (IMPF) disminuye la morbilidad neonatal relacionada con prematurez pero su efecto diabetógeno materno ha sido poco estudiado.

La revisión Cochrane de un ciclo único de corticosteroides se actualizó en 2006. En esta actualización se incluyeron 21 estudios con un total de 3885 mujeres y 4269 lactantes.

En la revisión se descubrió que la administración de determinados corticosteroides a mujeres con riesgo de tener un parto prematuro reduce considerable los riesgos de complicaciones relacionadas con la prematurez como muerte fetal y neonatal combinada, síndrome de dificultad respiratoria, hemorragia cerebroventricular, enterocolitis necrotizante, infecciones sistémicas y retraso en el desarrollo durante la niñez. Los beneficios estaban presentes cuando el tratamiento se iniciaba entre las 26 y las 35 semanas de gestación y en los niños que nacían entre 1 y 7 días después de haber comenzado el tratamiento; también se observaron beneficios en los subgrupos de mujeres con rotura prematura de membranas y trastornos hipertensivos. La muerte fetal y neonatal combinada se redujo incluso en neonatos que nacieron a menos de las 24 horas de haber administrado la primera dosis.

No se demostraron beneficios cuando el tratamiento comenzó antes de las 26 semanas de gestación, tampoco se observaron beneficios en los recién nacidos antes de las 26 semanas

de gestación ni en los que nacieron después de 7 días o más de la administración del tratamiento.

En el caso de los neonatos que nacieron después de las 36 semanas hubo una tendencia a aumentar la muerte fetal y neonatal combinada.

Se observó una reducción en el peso al nacer en los neonatos que nacieron entre los días 1 y 7, al igual que en los que nacieron más de 7 días después del primer tratamiento.

Un estudio que reclutó mujeres con preeclampsia severa sugirió que las mujeres tratadas tenían un mayor riesgo de sufrir diabetes gestacional.

La evidencia epidemiológica y en animales sugiere que pueden haber efectos adversos a largo plazo por la exposición prenatal a los corticosteroides, entre ellos la alteración de la tolerancia a la glucosa y la hipertensión. Los estudios en animales también han sugerido que afecta el crecimiento del cerebro.

1. National Institute of Health (NIH). Consensus Conference; Effect of corticos-teroide for fetal maturation on perinatal outcomes. JAMA 1994;(12):1-19.

2. White A, Marcucci G, Andrews E, Edwards K. Antenatal steroids and neonatal outcomes in controlled clinical trials of surfactant replacement. Am J Obstet Gynecol 1995; (173):286-90.

3. Klauss MH, Fanaroff AA, Martin RJ. Problemas respiratorios. En: Asistencia del recién nacido de alto riesgo. 2 ed. La Habana: Editorial Científico-Técnica, 1981:194.

4. Avery M, Frank N, Gribetz I. The inflationary force produced by pulmonary vascular distention in excised lungs. The possible relation of this force to that needed to inflatc the lungs at birth. J Clin Invest 1959;38:456.

5. Chu J, Clements J, Cotton E. Neonatal pulmonary ischemia. Pediatrics 1965;40:733. 6. Liggins GC, Howle RN. A controlled trial of antepartum glucocorticoid treatment for

prevention of respiratory distress syndrome in premature infants. Pediatrics 1972;50: 515-25.

7. Wright LL, Verter J, Younes N. Antenatal corticosteroids administration and neonatal outcome in infants 501 to 1500 g. Am J Obstet Gynecol 1995; (173):263.

75.- Ante una paciente de 35 años que acude a consulta con antecedentes de G4 C2 A1, refiere que ha presentado durante el primero y segundo trimestres de su embarazo manchado con frecuencia intermitente ,a las 34 SDG inicia con hemorragia abundante, repentina e indolora, su principal sospecha es:

a) Coriocarcinoma b) Ruptura uterina c) Placenta previa d) Desprendimiento grave de placenta normoinserta

PLACENTA PREVIA DEFINICIÓN: Es cuando la placenta se implanta sobre o muy cerca del orificio cervical interno y una parte de la placenta precede a la parte fetal que se presenta. INCIDENCIA: Esta es difícil determinar ya que muchos casos pasan desapercibidos, sobre todo cuando ocurren los abortos en embarazos tempranos. La prevalencia varía de 1 en 100 a 1 en 850 nacidos vivos, pero solo el 20% total. Etiología: Edad avanzada, multíparas, paciente con cesáreas previas, paciente con aborto de repetición, esto debido a las gestaciones previas.

TIPOS: Inserción baja.- Es cuando el borde placentario se encuentra en el segmento inferior a menos de 6 cm del orifico cervical interno. Marginal.- Es cuando el borde placentario alcanza los márgenes del orificio cervical interno. Parcial.- Es esta la placenta cubre parcialmente el orificio cervical interno. Total.- La placenta cubre la totalidad del orificio cervical interno aún con dilatación cervical avanzada. DIAGNÓSTICO: La característica es el STV de aparición brusca en forma indolora en el segundo o tercer trimestre. Frecuentemente hay ausencia de dolor a actividad uterina que son parámetros para hacer el diagnóstico. La mayor incidencia de sangrado aparece a las 33-34 sdg. ESTUDIOS DE GABINETE: El estudio más utilizado es la ultrasonografía obstétrica. TRATAMIENTO: El manejo va a depender de factores como son: Edad gestacional, magnitud del sangrado, si hay trabajo de parto, variedad de placenta previa y complicaciones materna. BIBLIOGRAFÍA: 1.- Waxler P, Gottesfeld KR. Early diagnosis of placenta previa. Obstet Gynecol 1979;54:231-32. 2.- Cabrero-Roura L. Riesgo elevado obstétrico. Ed. Masson 1996; pp; 109-118. 3.- Patrick J, Placenta Previa, Clinical Obst and Gynecology 1990;33(3): 414-421. 4.- Chapman M, Furtenes ET, Significance of ultrasound in location of placenta in early pregnancy Br J Obst Gynecol 197;86: 846. 57 76.- Mujer de 25 años de edad, es atendida en consulta por secreción láctea bilateral hace 4 meses y menstruaciones cada 21 días, en escasa cantidad desde hace 6 meses. Antecedentes: hace 6 meses padece gastritis tratada con cimetidina durante 2 meses y posteriormente ha tomado el medicamento de forma irregular. a.g.o.: menarca 12 años, ritmo 30/7, núbil. e.f.: talla 1.63, peso 54 kg. Al efectuar compresión en glándulas mamarias se produce salida de secreción láctea bilateral, resto normal. El tratamiento de primera elección para esta paciente es:

a) Metimazol b) Bromocriptina. c) Levotiroxina S. d) omeprazol.

La hiperprolactinemia (hiperprl) es un trastorno frecuente, más en la mujer que en el hombre, y puede ser la manifestación de un adenoma hipofisario (prolactinoma). La medición de prolactina (prl) en la sangre es ahora un procedimiento de rutina en la detección de trastornos de la menstruación y galactorrea que se resuelven efectivamente con el uso de fármacos con actividad dopaminérgica (bromocriptina y cabergolina). La causa más frecuente de hiperprl es por el uso de fármacos que inhiben la acción de dopamina, pero se debe descartar inicialmente hipotiroidismo y un prolactinoma mediante resonancia magnética. Los prolactinomas son fácilmente controlados con dopaminérgicos y excepcionalmente se requiere cirugía. En caso de infertilidad por exceso de prl existe una respuestafavorable con bromocriptina o cabergolina, aunque en la actualidad se prefiere la última por su efectividad y menos efectos indeseables.

Hiperprolactinemia. Guía terapéutica y diagnóstica Arturo Zárate* * Unidad de investigación de endocrinología, diabetes y metabolismo, Centro Médico Nacional IMSS y Hospital Ángeles México. 77.- A woman presents with painless vaginal bleeding at 37 weeks´ gestation. The fetal heart rate is stable in the 150 pbm. Which of the following is indicated? a) Ultrasound examination b) Nonstress test c) Induction of labor d) Digital examination of the cervix Diagnóstico por USG.

Sangrados 2da Mitad EmbarazoDiagnóstico diferencial (1)

DPPNI PLACENTA PREVIA

DOLOR HIPOGASTRICO

TONO UTERINO

COMPROMISOFETAL

U.S.G.

PRESENTE AUSENTE

AUMENTADO RELAJADO

FRECUENTE INFRECUENTE

DIAGNOSTICA DIAGNOSTICA

78.- Se trata de paciente femenino de 55 años de edad que presenta prolapso uterino e incontinencia urinaria de esfuerzo el procedimiento de elección en esta paciente es: a) Histerectomia total abdominal.

b) Histerectomía vaginal reconstructiva c) Colpoperineoplastía. d) Uterosuspensión. El prolapso genital y su tratamiento ha sido siempre un importante capítulo de la ginecología. No existe consenso clínico del concepto que define al prolapso genital como patológico. Cierto grado de descenso y relajación de la pared vaginal es considerado normal en la gran mayoría de las mujeres, siendo más frecuente en mujeres de mayor edad. En la población general solo un 3% presenta prolapso genital severo, entendiendo como tal al de III° y IV° grado. Es

probable que alrededor de 3 a 6% de la población femenina desarrolle un prolapso de esas características en algún momento de su vida La intervención quirúrgica por excelencia para el tratamiento quirúrgico del prolapso uterino es la histerectomía vaginal. Esta intervención se clasifica como "limpia-contaminada

INDICACIONES PARA LA HISTERECTOMÍA VAGINAL

Prolapso uterino Hemorragia uterina disfuncional

Carcinoma cérvico-uterino "in situ" Miomatosis uterina poco voluminosa

Hiperplasia endometrial Piometra

VENTAJAS DE LA HISTERECTOMÍA VAGINAL

No deja cicatriz abdominal Mínimo trauma abdominal

Escasa hemorragia transoperatoria Mínima manipulación intestinal

Menos dolor postoperatorio

CUADROCONTRAINDICACIONES PARA LA HISTERECTOMÍA VAGINAL

Impericia

Útero muy voluminoso Tumores ováricos

Endometriosis Enfermedad pélvica inflamatoria

Cirugía previa en útero, trompas y ovarios Cáncer de endometrio

Histerectomía obstétrica

79.- Se trata de femenino de 26 años de edad, que acude a revisión por 8 semanas de amenorrea, asintomática. En la ecografía vaginal se observa una tumoración anexial izquierda, sin evidencia de embrión. En el útero no se visualiza saco gestacional. La actitud más correcta en éste caso es:

a) Ingreso para legrado. b) Culdocentésis. c) Salpingectomía urgente. d) Ingreso y determinación de niveles deB-HCG.

FACTORES DE RIESGO

Anomalías tubarias

Enfermedad pélvica inflamatoria 30-50% Adherencias (endometriosis) Antecedente de salpingoclasia Alteraciones anatómicas

EMBARAZO ECTÓPICO

Descenso

Cuantificación seriadade HGC

USG normal

< 6500mUI

Repetir USG Laparoscopia DxDilatación y legrado

Estabilización oincremento

Aumento

27/01/201464

Diagnóstico:

HGC > 1500 U/L Sin evidencia de saco Líquido libre en fondo de saco

Sensibilidad 63% Especificidad 100%

Ankum WM., Mol Bw.,Van der Veen F.,Bossuyt PM. Risk factors for ectopic pregnancy:a meta-analysis. Fertil Steril 1996:65:1093-9. 80.- Femenino de 20 años, atendida en sala de urgencias ginecoobstétricas, Antecedente: cursa embarazo de 38 SDG. Exploración Física: en trabajo de parto. Repentinamente presenta sangrado profuso transvaginal y dolor abdominal. La causa más probable de la sintomatología de esta paciente es:

a) Laceración vaginal por coito b) Cervicitis c) Placenta previa d) Abruptio placentae

DESPRENDIMIENTO PREMATURO DE PLACENTA NORMOINSERTA (DPPNI): Constituye la separación de la placenta de su área de inserción antes del 3º período del parto. La mortalidad fetal es muy alta (superior al 15%) y la materna es tres veces superior a la esperada. La aparición frecuente de SFA, prematuridad, anemia, etc., hace que el número de secuelas tanto sensitivas como motoras sea alto. Desde el punto de vista materno, complicaciones secundarias a la hemorragia, a las alteraciones de la coagulación o bien a la embolia pulmonar también tienen una tasa muy alta. Desde la antigüedad se identificó al cuadro clínico caracterizado con la tríada sintomática de hipertonía, metrorragia y muerte fetal, con pronóstico materno comprometido. En 1775, Ricci diferenció la separación prematura de una placenta de inserción normal, de aquella de inserción baja; a la primera llamó “hemorragia accidental”, y a la segunda “hemorragia imprevisible”. Couvelaire introdujo el término de “desprendimiento prematuro de placenta” y describió la aparición de la “apoplejía uterina”

(útero de Couvelaire)

Obstetricia. Scwarcz, Sala, Duverges. 7ª edic. Edit. El Ateneo. (Biblioteca Fac. Med. UNNE).